SlideShare una empresa de Scribd logo
1 de 50
Descargar para leer sin conexión
ENAEExamen Nacional de Enfermería
2017
Pedidos: Cel. :RPC. 969340751
Cel. :RPC. 941105128
Correo: publicacionesdc@hotmail.com
COMPENDIO DE
BANCOS DE PREGUNTAS
6ta Edición
[1]
EXAMEN NACIONAL DE ENFERMERÍA. Compendio Banco de Preguntas.
Mg. Dany Colca
Lic. Carolina Ayala
1. Hombre de 48 años que acude a Urgencias por dolor abdominal y
vómitos. Los datos analíticos iniciales son: GOT: 80 U/I. GTP 54 U/I.
Leucocitos 21.800/mm3. Amilasa 4.500 U/I. La TAC abdominal
demuestra colección peripancreática. Se inicia tratamiento con
fluidoterapia y analgésicos. Señale cuál de los siguientes
fármacos añadiría al tratamiento, en primer lugar, para mejorar el
pronóstico de este caso:
A) Metilprednisolona.
B) Antiinflamatorios no esteroideos. C) D)
Imipenem.
E) Inhibidores de la secreción pancreática.
2. ¿Cuál es el principal factor responsable de la no
cicatrización de una úlcera péptica?:
A) Estrés.
B) Infección por Helicobacter pylori.
C) Consumo de alcohol.
D) No abandono del hábito tabáquico. E)
Determinados hábitos dietéticos.
3. ¿Cuál de las siguientes recomendaciones es FALSA en relación
con el tratamiento de la peritonitis bacteriana espontánea en un
paciente cirrótico?:
A) El diagnóstico se basa en la existencia de más de
250 PMN/mm3 en el líquido ascítico.
B) El tratamiento de elección lo constituyen las
cefalosporinas de tercera generación.
C) La administración de albúmina intravenosa previene el
desarrollo de insuficiencia renal.
D) El tratamiento antibiótico debe mantenerse durante
15 días.
E) Una vez resuelta la infección, debe iniciarse tratamiento
profiláctico con norfloxacino.
4. Paciente de 72 años, que como único tratamiento toma
antidiabéticos orales, presenta anemia ferropénica crónica, con
hemorragias ocultas positivas. ¿Cual es el método diagnóstico más
indicado para localizar la lesión sangrante?:
A) Tránsito gastroduodenal.
B) Gammagrafía con hematíes marcados. C)
Tránsito intestinal.
D) Colonoscopia total. E)
Panendoscopia oral.
5. ¿Cuál de las siguientes definiciones corresponde con mayor
precisión al concepto de Metaanálisis?:
A) Es una revisión narrativa en la que la bibliografía se busca de
manera no estructurada.
B) Es una revisión narrativa en la que la bibliografía se busca de
manera estructurada.
C) Es una revisión narrativa en la que se presentan tablas
estructuradas de los resultados de los distintos estudios incluidos.
D) Es una revisión en la que se combinan
estadísticamente los resultados de los estudios incluidos.
BANCO
N°001
[2]
Mg. Dany Colca
Lic. Carolina Ayala
RESIDENTADO DE ENFERMERÍA. Compendio Banco de Preguntas.
E) Es una revisión en las que se presentan tablas estructuradas de
los resultados de los distintos estudios incluidos.
6. Le encargan el diseño de un ensayo clínico en el que es muy
importante que un factor pronóstico se distribuya por igual en los dos
grupos de tratamiento. El método de aleatorización que usted utilizaría
es:
A) Aleatorización simple.
B) Aleatorización por bloques. C)
Aleatorización estratificada. D)
Aleatorización centralizada.
E) Aleatorización ciega (ocultación de la secuencia de
aleatorización).
7. En una revista biomédica se publica un estudio, en el que los
autores notifican el resultado en coste/años de vida ganados. ¿De qué
tipo de análisis de evaluación económica se trata?:
A) Coste de la enfermedad. B)
Coste-efectividad.
C) Coste-beneficio.
D) Coste-consecuencia. E)
Coste-utilidad.
8. La mejor forma de verificar una hipótesis en
epidemiología es a través de:
A) Un estudio descriptivo.
B) Un estudio experimental. C) Un
estudio caso-control. D) Un
estudio de cohortes. E) Un
estudio transversal.
9. Todas estas situaciones, EXCEPTO una, aumentan la probabilidad
de detectar un cáncer colorrectal, señálela: A) Enfermedad
inflamatoria intestinal de larga evolución. B) Endocarditis por
Streptococcus bovis.
C) Tabaquismo de más de 35 años de duración.
D) Ureterosigmoidostomía hace 20 años, para corregir una
malformación vesical.
E) Ingesta crónica de aspirina o antiinflamatorios no
esteroideos.
10. ¿Cuál de las siguientes características NO es propia de un
ensayo clínico controlado?:
A) Intervención experimental.
B) Tratamiento asignado según el criterio del
investigador.
C) Participación voluntaria del sujeto.
D) Asignación aleatoria a las distintas posibilidades de tratamiento
en estudio.
E) Enmascaramiento de la medicación.
11. La vacuna antineumocócica está recomendada para todas,
MENOS una de las siguientes situaciones clínicas, señálela:
A) Alcoholismo crónico.
B) Contactos familiares de un paciente con neumonía
neumocócica
C) Fístula crónica de líquido cefalorraquídeo tras un
traumatismo craneal.
D) Infección VIH avanzada.
E) Insuficiencia cardíaca crónica.
12. ¿Cuál de la siguientes vacunas no se incluye en las
recomendaciones actuales de un paciente esplenectomizado?:
A) Vacuna neumocócica. B)
Vacuna meningocócica.
C) Vacuna frente a Haemophilus influenzae tipo B.
D) Vacuna frente al virus de la gripe. E)
Vacuna frente al virus hepatitis A.
13. ¿En qué patología pensaría en primer lugar en un paciente de 65
años, que presenta disminución lenta, progresiva e indolora de su
agudeza visual sin signos de inflamación ocular?:
A) Error de refracción. B)
Distrofia corneal.
C) Papilitis.
D) Glaucoma agudo.
E) Degeneración macular senil.
14. Un paciente diabético tratado mediante fotocoagulación focal con
láser de Argón tres años antes, presenta una pérdida brusca e
importante de visión, sin dolor ni alteraciones en la superficie ocular.
La causa más probable de esta disminución de agudeza visual es:
A) Hemorragia vítrea. B)
Edema corneal.
C) Glaucoma crónico simple. D)
Catarata nuclear.
E) Atrofia óptica.
15. Una de los siguientes signos o síntomas, NO es habitual en
una uveítis anterior aguda:
A) Midriasis. B)
Dolor.
C) Inyección ciliar.
D) Sinequias posteriores. E)
Fotofobia.
16. ¿Cuál es el tratamiento de la obstrucción intestinal no
quirúrgica en la fase terminal del cáncer?:
A) Sonda nasogástrica, aspiración continua,
sueroterapia intravenosa.
B) Administración de vitamina grupo B, laxante y enema de limpieza.
C) Alimentación parenteral total y esteroides i.v.
D) Morfina, buscapina y haloperidol por vía subcutánea. E) Dieta
absoluta y sueros por vía subcutánea.
17. ¿Cuál de los siguientes opioides NO es adecuado para el
tratamiento del dolor crónico de etiología cancerosa?:
A) Metadona. B)
Meperidina. C)
Morfina.
[3]
Mg. Dany Colca
Lic. Carolina Ayala
RESIDENTADO DE ENFERMERÍA. Compendio Banco de Preguntas.
D) Fentanilo. E)
Tramadol.
18. La localización ideal para hacer una traqueotomía es:
A) La membrana crico-tiroidea. B) La
membrana mio-tiroidea. C) Primer
anillo traqueal.
D) Segundo o tercer anillo traqueal.
E) Cuarto o quinto anillo traqueal.
19. ¿Cuál de estas afirmaciones es FALSA en relación con la otitis
media secretoria?:
A) Los niños con paladar hendido son más susceptibles a
padecerla.
B) En casos unilaterales en adultos, es obligada la inspección
del cavum.
C) Su período de mayor incidencia en climas templados, es de junio a
septiembre, coincidiendo con la época de baños.
D) El 66% de los cultivos de las muestras obtenidas por
miringocentesis presentan bacterias.
E) La otoscopia puede mostrar niveles líquidos o
burbujas de aire tras el tímpano íntegro.
20. Una mujer de 68 años, sin aparentes factores de riesgo
cardiovascular, ingresa en la Unidad Coronaria del Hospital por un
cuadro ¿agudo de cardiopatía isquémica. En la analítica realizada
a su llegada se objetiva una anemia (Hb 8gr/dl) previamente no
conocida. En este caso, la actitud más adecuada con respecto a la
anemia es:
A) Actitud expectante, ya que sólo se debe transfundir la anemia
sintomática.
B) Transfundir hematíes.
C) Transfundir sangre total.
D) Instaurar tratamiento con eritropoyetina. E)
Administrar hierro intravenoso.
21. Los cuerpos de Howell-Jolly, son inclusiones
eritrocitarias de fragmentos nucleares y se observan en: A) Asplenia.
B) Mielofibrosis.
C) Leucemia linfática crónica. D)
Déficit de G6PD.
E) Intoxicación por plomo.
22. ¿Cuál de los siguientes hallazgos es el más
característico de la leucemia mieloide crónica?:
A) El cromosoma Filadelfia.
B) El esplenomegalia palpable.
C) La disminución de la fosfatasa alcalina granulocitaria. D) El
aumento del ácido úrico sérico.
E) El reordenamiento del gen bcr/abl.
23. Joven de 23 años, con historia de conducta sexual de riesgo,
consulta por fiebre, malestar general y dolor en glande. La
exploración física muestra múltiples tatuajes, "piercings" y la
presencia de tres lesiones
vesiculosas en glande y adenopatías inguinales bilaterales.
¿Cuál es el diagnóstico más probable?:
A) Herpes genital.
B) Secundarismo luético. C)
Primoinfección VIH.
D) Infección por citomegalovirus. E)
Condilomas acuminados.
24. En cuanto a la neumonía por Neumococo en paciente
infectados por el virus de la inmunodeficiencia humana (VIH) es
FALSO que:
A) Es una infección frecuentemente bacteriémica.
B) Tiene mayor incidencia que en la población general. C) Junto a
Haemophilus influenzae es la causa más común de neumonía en
pacientes con SIDA.
D) Se recomienda vacuna neumocócica en aquellos pacientes
con CD4 < 100 cel/ L.
E) Esta neumonía puede ser vista en pacientes con sistema
inmune relativamente intacto.
25. La tuberculosis asociada a la infección por VIH se caracteriza
por:
A) Presentación subclínica de la enfermedad.
B) Aparición característica en los estadios de
inmunodepresión más severa (>50 CD4/mm3).
C) Elevada frecuencia de afectación extrapulmonar y
diseminada.
D) Escaso rendimiento de los métodos microbiológicos de
diagnóstico.
E) Mala respuesta al tratamiento antituberculoso.
26. NO es propio del shock tóxico estafilocócico: A) Fiebre
elevada.
B) Lesiones cutáneas. C)
Metástasis sépticas. D)
Fracaso renal.
E) Rabdomiólisis.
27. ¿Cuál de las siguientes permite diferenciar la pielonefritis
aguda de la cistitis aguda?:
A) Leucocituria. B)
Hematuria. C)
Bacteriuria.
D) Antecedentes de infección urinaria.
E) Fiebre de más de 38.5ºC.
28. Un paciente de 22 años, que acude para evaluación de una lesión
genital ulcerada, presenta un VDRL positivo a título de 8 diluciones
con un FTA-Abs negativo. La interpretación más adecuada de estos
resultados es:
A) Falso positivo de las pruebas no treponémicas. B) Falso
negativo de las pruebas treponémicas.
C) Sífilis curada (pendiente la positivización del FTA- Abs).
D) Sífilis curada (pendiente la negativización del VDRL). E) Sífilis de
larga evolución.
[4]
Mg. Dany Colca
Lic. Carolina Ayala
RESIDENTADO DE ENFERMERÍA. Compendio Banco de Preguntas.
29. Señale cuál de las siguientes asociaciones de helmintos y su
clínica característica es INCORRECTA:
A) Ascaris lumbricoides - Síndrome de Löeffler. B) Giardia
lamblia – Mala absorción intestinal.
C) Ancylostoma duodenale - Anemia megaloblástica.
D) Strongyloides stercolaris - Síndrome de
hiperinfestación en inmunodeprimidos.
E) Taenia solium - Convulsiones generalizadas.
30. Paciente de 30 años, seropositivo VIH conocido desde 5 años
antes, con antecedentes de neumonía por P. jivovecci, que consulta
por cefalea desde 10 días antes. La exploración física muestra como
datos más relevantes mínima rigidez de nuca y temperatura de
37,5ºC, fondo de ojo normal, TAC: ligera atrofia cortical. La punción
lumbar da salida a líquido claro con 40 células mononucleares,
proteínas: 90 mgrs%, glucosa:
30 mg% (glucemia: 90 mg%). Señalar, de entre las siguientes, la
causa más probable:
A) Herpesvirus tipo 8. B)
Listeria.
C) Criptococo. D)
CMV.
E) VIH.
31. En relación con Streptococcus pyogenes y la faringoamigdalitis,
¿cuál de las siguientes afirmaciones NO es correcta?:
A) En tratamiento de la faringoamigdalitis estreptocócica se efectúa
con una sola inyección i.m. de 1,200,000 UU de penicilina benzatina.
B) El tratamiento antibiótico de la faringoamigdalitis estreptocócica se
efectúa con 250,000 UU/6h oral de penicilina V durante 10 días.
C) El tratamiento antibiótico de la faringoamigdalitis estreptocócica se
efectúa con una sola inyección i.m. de
1,200,000 UU de penicilina procaína.
D) El tratamiento antibiótico de la faringoamigdalitis estreptocócica se
efectúa con amoxicilina oral 500 mg/8h durante 10 días.
E) El tratamiento antibiótico recomendado de la faringoamigdalitis
estreptocócica en los casos de alergia a la penicilina es un macrólido
oral durante 10 días.
32. A lo largo de los últimos 10 años se ha producido un cambio muy
importante en la etiología de la Endocarditis Infecciosa del adulto. El
microorganismo más frecuente en la actualidad, es:
A) Microorganismos del grupo HACEK. B)
Staphylococcus aureus.
C) Estafilococos coagulasa negativos. D)
Bacilos gram negativos.
E) Streptococcus viridans.
33. Un hombre de 74 años con un infarto agudo de miocardio es
tratado con estreptoquinasa. Seis horas después desarrolla un
cuadro de hipotensión arterial severa y obnubilación. ¿Cuál de las
siguientes complicaciones es MENOS probable que sea la causa?: A)
Infarto de ventrículo derecho.
B) Tromboembolismopulmonar. C)
Rotura del músculo papilar.
D) Rotura de la pared libre ventricular. E)
Hemorragia cerebral.
34. En el tratamiento de la hipertensión arterial, la ventaja de
los bloqueadores de los receptores de la Angiotensina II con
respecto a los inhibidores del enzima conversor de la Angiotensina
es que:
A) Son más potentes.
B) Producen menos tos.
C) No producen hiperpotasemia. D) Se
puede dar en embarazadas.
E) Se pueden dar en sujetos con estenosis de la arteria renal
bilateral.
35. El tratamiento más eficaz para prevenir recurrencias en el aleteo
o flutter auricular común es:
A) Buen control de la hipertensión arterial que con frecuencia
padecen estos enfermos.
B) Digoxina asociada a un fármaco que disminuya la conducción en
el nodo A-V (anticálcicos o betabloqueantes).
C) Ablación con catéter y radiofrecuencia del istmo cavo tricúspide.
D) Inserción de un marcapasos con capacidad
antitaquicardia.
E) Amiodarona.
36. Un paciente obnubilado con una presión arterial de
80/40 mmHg, un gasto cardíaco de 3 l/min, una presión de
enclavamiento pulmonar de 14 mmHg y una presión auricular derecha
de 14 mmHg, puede estar sufriendo cualquiera de las siguientes
situaciones patológicas con la excepción de una:
A) Taponamiento cardíaco. B)
Deshidratación.
C) Infarto de ventrículo derecho. D)
Tromboembolismo pulmonar. E)
Constricción pericárdica.
37. La campana del fonendoscopio es más adecuada que la
membrana para:
A) Distinguir un soplo diastólico de uno sistólico.
B) Oír mejor el chasquido de apertura en pacientes con estenosis
mitral.
C) Valorar la presencia de un tercer y/o cuarto tono. D) Detectar
el click mesosistólico del prolapso mitral.
E) Auscultar a los niños pequeños y bebés, en los que la membrana
no capta bien la tonalidad de los ruidos
cardíacos.
38. ¿Cuál de las siguientes respuestas sobre el Delirium
Tremens es la correcta?:
A) Se trata del estado peculiar de intoxicación producido por el
consumo de alcohol.
B) Se caracteriza por la aparición de conductas impulsivas
tras el consumo de una pequeña cantidad de
alcohol.
[5]
Mg. Dany Colca
Lic. Carolina Ayala
RESIDENTADO DE ENFERMERÍA. Compendio Banco de Preguntas.
C) Es un cuadro clínico con elevado índice de complicaciones
psiquiátricas y con pocas complicaciones orgánicas.
D) Aparece con mayor frecuencia entre los hombres alcohólicos
a partir de los 50 años.
E) Su tratamiento de elección son las benzodiacepinas.
39. ¿Cuáles son los trastornos perceptivos, de entre los que se
enumeran, más característicos de las esquizofrenias?:
A) Ilusiones hipnagógicas.
B) Alucinaciones visuales zoomórficas.
C) Alucinaciones o pseudoalucinacionesauditivas. D)
Paraeidolias.
E) Alucinosis auditivas.
40. Una paciente de 24 años acude a la Urgencia llevada por un
familiar por vómitos recidivantes. Parece bulimia nerviosa. ¿Cuál de
las pruebas de laboratorio es más útil para evaluar la gravedad de los
vómitos?:
A) Nivel de Hemoglobina.
B) Nivel plasmático de Amilasa. C)
Nivel sérico de Socio.
D) Nivel plasmático de Calcio.
E) Nivel plasmático de Creatinina.
41. Acude a urgencias un paciente de 37 años con gran postración,
fiebre elevada (38,5°C) e intensa cefalea. Presenta en miembros
inferiores, tronco y raíz de miembros superiores un exantema tenue
eritemato- violáceo, maculoso, escasamente confluente, con
afectación de palmas y plantas. Al interrogar a familiares,
reconocen haber estado hace siete días en una excursión.
Reexplorado el paciente, en cara posterior de la piera derecha
aparece una lesión necrótica-costrosa, de 0,5 cm. de diámetro,
rodeada por un halo violáceo, edematoso de 0,3-0,4 cm. El
tratamiento más indicado sería:
A) Doxiclina v.o. (200 mgr. cada 12 horas un día ó 100 mgr. cada
12 horas por 5 días).
B) Cloxacilina 1 gr. i.v. cada 6 horas durante 10 días. C)
Ceftriaxona 1 gr. i.v. cada 12 horas durante 10 días. D) TMP-SMX
500 mgr. i.v. cada 8 horas por 10 días.
E) Eritromicina 250 mgr. i.v. cada 6 horas por 7 días.
42. Un paciente de 27 años ex-ADVP desde hace tres años, con
muguet oral y antecedentes de neumonía por neumocistis carinii,
presenta en el dedo índice, en superficie dorsal de la 2.ª falange,
una lesión única, úlcero necrótica, con crecimiento serpinginoso de
aproximadamente 4-5 cms. de diámetro, bordes geográficos y
algunas áreas costrosas, hemorrágicas. Es extremadamente
dolorosa. Su 1.ª sospecha clínica es:
A) Linfoma B cutáneo.
B) Ulcera secundaria al tratamiento. C)
Infección por herpes simple.
D) Chancro luético.
E) Picadura sobreinfectada.
43. Un paciente de 67 años, comienza a presentar lesiones
eritematoedematosas en brazos y abdomen. Ocasionalmente se
observan ampollas salpicando el área afectada. No hay afectación
de mucosas. Conserva el estado general. Se realizó biopsia de piel
donde se observó las imagen de una vesícula subepidérmica con
eosinófilos. En inmunofluorescencia directa se observó una imagen
de depósito lineal en UDE de IgG. Tras usar la técnica de separación
con sal, la banda de depósito aparece tanto en el suelo, como en el
techo de la ampolla. El diagnóstico más probable será:
A) Enfermedad de During-Brocq. B)
Fogo Selvagen.
C) Enfermedad IgA lineal.
D) Epidermólisis ampollosa adquirida. E)
Penfigoide ampolloso.
44. Un paciente de 44 presenta lesiones pápulo- erosivas muy
pruriginosas en glúteo y rodillas, que comenzaron a salir hace meses,
cursando en brotes que curan espontáneamente en una semana.
Se ha realizado biopsia de piel observándose ampolla subepidérmica
en tinción de hematoxilina-eosina y un depósito de IgA linea en
UDE. La biopsia de vellosidades intestinales en normal. Los
anticuerpos antigliadina IgA son negativos. Los anticuerpos
antigliadina IgG y los anticuerpos antiendomisio son positivos.
Presenta además anticuerpos antitiroideos. El tratamiento adecuado
será:
A) Corticoides orales porque es una enfermedad IgA
lineal.
B) Corticoides orales porque es una Dermatitis
Herpetiforme con biopsia intestinal negativa.
C) Sulfonas más dieta sin gluten, porque es una
Dermatitis Herpetiforme.
D) Sulfonas sin dieta, porque es una Dermtitis
Herpetiforme con biopsia intestinal negativa. E)
Sulfonas más inmunosupresores.
45. Un varón de 6 años presenta de manera brusca un brote de
lesiones eritematoescamosas redondeadas, de pqueño tamaño,
distribuídas por todo el tegumento. La semana anterior acudió a
Urgencias por un cuadro catarral, con T°= 37,5°C y amigdalitis. En el
cultivo faríngeo se aisló un estreptococo. La patología cutánea más
probable corresponde a:
A) Pitiriasis rosada. B)
Eczema numular. C)
Psoriasis gutata. D)
Herpes circinado. E)
Vasculitis séptica.
46. Un joven de 26 años, con antecedentes familiares de psoriasis,
presenta un brote de lesiones en placas eritematosas, con centro
amarillento y halo periférico descamativo, ovaladas, bien
delimitadas, en cara anterior y posterior del tronco. La 1.ª lesión
había aparecido hacía 1 semana, era de mayor tamaño y se
localizaba en el tercio superior de la espalda. El
[6]
Mg. Dany Colca
Lic. Carolina Ayala
RESIDENTADO DE ENFERMERÍA. Compendio Banco de Preguntas.
paciente conservó un buen estado general en todo momento.
Probablemente el cuadro corresponde a:
A) Psoriasis en pequeñas placas. B)
Psoriasis tipo Von Zumbusch. C)
Pitiriasis rosada de Gibert.
D) Roséola sifilítica.
E) Eczema diseminado.
47. Un varón de 16 años comienza a presentar vesículas de
contenido claro agrupadas, de localización peribucal, que a lo largo
de 1 semana se van rompiendo formándose erosiones y costras.
Transcurrido este período, en el dorso de las manos aparecen 2
lesiones ampollosas con vesículas dispuestas periféricamente y
alguna pápula eritematosa con centro más oscuro. Se trata de:
A) Inicio de una varicela.
B) Aparición de lesiones en manos por contagio directo desde la
zona peribucal.
C) Impétigo estafilocócico.
D) Eritema multiforme minor.
E) Síndrome de Stevens-Johnson.
48. Una mujer de 65 años viene presentando desde hace 2 años
brotes de lesiones nodulares < de 2 cm., localizadas de forma bilateral
en MMII, acompañados de livedo reticularis, febrícula y artralgias. La
histología corresponde a una vasculitis leucocitoclástica.
Probablemente se trata de:
A) Síndrome de Sweet.
B) Eritema elevatum diutinum. C)
Panarteritis nodosa cutánea. D)
Vasculitis urticarial.
E) Enfermedad de Kawasaki.
49. Un paciente de 63 años de edad presenta una eritrodermia de
varios meses de evolución con adenopatías generalizadas y más
de 10% de células con núcleo cerebriforme en sangre periférica.
Tiene además intenso prurito y edemas pretibiales. Su diagnóstico
sería:
A) Eczema seborreico.
B) Exantema medicamentoso crónico. C)
Eritrodermia psoriasica.
D) Síndrome de Sezary.
E) Parapsoriasis en grandes placas.
50. Un niño sin alteraciones cutáneas al nacimiento comienza a
presentar hacia los 3 meses de edad escamas grandes y negruzcas
en tronco y extremidades, afectando pliegues axilares y poplíteos.
Tiene un hermano mayor igualmente afecto. Es probable que
se trate de:
A) Una ictiosis ligada a X. B)
Una ictiosis laminar.
C) Una ictiosis vulgar.
D) Enfermedad de Darier.
E) Ninguna de las anteriores.
51. Una mujer acude a nuestra consulta poque tras haber
estado esa mañana expuesta al sol presenta una
quemadura solar exagerada localizada en cara (respetando región
peiorbitaria, retroauricular y submentoniana), dorso de manos y en
ambas piernas hasta la altura de las rodillas. Estaba en tratamiento
con un diurético tiacidico.
A) Se trata de una reacción fototóxica.
B) Es una reacción fotoalérgica por mecanismo tipo IV. C) Debe
eliminarse el agente causante, poner tratamiento sintomático
y evitar la radiación lumínica hasta que remita el cuadro.
D) Es una erupción polimorfa lumínica. E) Son
correctas A y C.
52. Una joven de 16 años de edad presenta varias máculas de color
blanco lechoso de varios centímetros de diámetro, de distribución
simétrica sobre codos, rodillas, manos y zona peribucal. En la biopsia
cutánea hay ausencia de malanocitos. Tras fotoquimioterapia
sistémica con psoralenos han pigmentado parcialmente las lesiones.
El diagnóstico de esta paciente es:
A) Esclerosis tuberosa. B)
Lepra.
C) Hipomelanosis guttata idiopática. D)
Vitíligo.
E) Hipomelanosis de Ito.
53. Niño de 10 años de edad que acude a la consulta por amigdalitis
pultácea con adenopatias cervicales. Se pone tratamiento con
penicilina a dosis correctas. A las
72 horas acude de nuevo al no experimentar mejoria. Debemos
pensar en:
A) Posible Mononucleosis infecciosa.
B) Posible amigdalitis viral de etiología diferente al
Estreptococo beta hemolítico del grupo A. C)
Linfoma.
D) V. Parainfluenzae. E)
Rubéola.
54. La presencia de vómitos en la infancia es uno de los motivos más
frecuentes de consulta. Pueden ser debidos a enfermedades
digestivas y extradigestivas.
¿Cuál de las siguientes patologías produce menos
frecuentemente vómitos en la etapa de la lactancia?
A) Reflujo gastroesofágico.
B) Estenosis hipertrófica de píloro. C)
Apendicitis.
D) Invaginación intestinal. E)
Gastroenteritis.
55. Ante un niño que de forma brusca presenta hipotensión, vómitos
y colapso cardiovascular que no responde a la administración de
drogas vasoactivas o catecolaminas y en la analítica realizada
presenta hiponatremia e hiperpotasemia, se debe considerar como
probable diagnóstico:
A) Insuficiencia cardíaca.
B) Insuficiencia suprarrenal. C)
Diabetes juvenil.
D) Diabetes insípida.
E) Intoxicación por monóxido de carbono.
[7]
Mg. Dany Colca
Lic. Carolina Ayala
RESIDENTADO DE ENFERMERÍA. Compendio Banco de Preguntas.
56. Un varón de 17 años se encuentra mareado, con vómitos y en el
transcurso de unos minutos se halla tumbado en el suelo en coma,
con una exploración neurológicanormal. La causa más probablesería:
A) Ingestión de barbitúricos.
B) Hemorragia subaracnoidea. C)
Coma etílico.
D) Status convulsivo. E)
Tumor cerebral.
57. Un varón de 4 años tiene lesiones purpúricas palpables,
simétricas, de 3 días de evolución en las extremidades inferiores.
Los estudios hematológicos revelan:
Hemoglobina: 10 g/dl; recuento leucocitario
16.500/mm3; recuento plaquetario 240.000/mm3 y VSG
de 45 mm/hora. La etiología más probable es: A)
Maltrato infantil.
B) Púrpura de Schonlein Henoch. C)
Enfermedad de Kawasaki.
D) Meningococemia.
E) Enfermedad de Von Willebrand.
58. Durante la evaluación previa al ingreso en el colegio de un niño
de 5 años, se detecta retraso en el habla. Como antecedentes se
recogen episodios reiterados de cuadros catarrales sin control
médico. ¿Cuál de las siguientes es la casusa más probabnle
de esta situación?
A) Trastorno de déficit de atención con hiperactividad. B)
Hipoacusia de conducción.
C) Disfunción de la Trompa de Eustaquio. D)
Retraso mental.
E) Hipoacusia neurosensorial.
59. Niña de 7 años sin antecedentes de interés que acude a urgencias
por dolor abdominal generalizado y vómitos desde doce horas antes.
Deposición normal. No antecedentes quirúrgicos previos. A la
exploración presenta abdomen muy distendido y dolor en zona
periumbilical con aumento de ruidos intestinales, Blumberg (-). En la
Radiografía de abdomen en bipedestación se observa obstrucción
a nivel de intestino delgado. El diagnóstico más probable sería:
A) Invaginación intestinal. B)
Brida intestinal.
C) Malrotación intestinal. D)
Divertículo de Meckel.
E) Estenosis ileal congénita.
60. Niño de 5 meses que llevan a la consulta por tos y secreción nasal
desde hace 24 horas. Toma mal los biberones por presentar fatiga.
Se observa febrícula y retraccion intercostal, con zonas de
hipoventilación y estertores en la auscultacion respiratoria. Existen
otros familiares con cuadro catarral. Se le pone tratamiento
sintomático y se aconseja la revisión a las 24 horas. En la nueva
visita el niño ha empeorado, con importante insuficiencia respiratoria,
sibilancias, tos continua y
fiebre de 38°C. ¿Cuál de los siguientes cuadros cree que
presenta el niño?:
A) Catarro habitual descendente con evolución a asma. B)
Aspiración de cuerpo extraño.
C) Bronquiolitis. D)
Epiglotitis.
E) Laringitis.
61. Un niño nacido a término de 2.100 gr. de peso presenta
irritabilidad y temblores amplios a las 36 horas de vida. Se alimenta
mal y tiene diarrea y obstrucción nasal. ¿Cuál es el diagnostico más
probable?:
A) Hipocalcemia.
B) Hipomagnesemia.
C) Déficit de piridoxina.
D) Sindrome de abstinencia por adición materna a heroína.
E) Hipoglucemia.
62. Lactante de dos meses que presenta llanto agudo, en crisis,
desde hace 20 días, síntomas motores y heces normales para su
edad. ¿El diagnóstico más probable será?:
A) Gastroenteritis aguda.
B) Intolerancia a la lactosa. C)
Cólico del lactante.
D) Otitis media aguda. E)
Hernia inguinal.
63. Lactante de tres meses sin antecedentes previos de interés, que
en el examen de salud se le detecta un deterioro en las adquisiciones
sicomotoras, el resto de la exploración por aparatos es normal. En la
anamnesis presenta sacudidas musculares breves de cabeza y
extremidades superiores. ¿Cuál sería el diagnóstico más
probable?:
A) Cólico del lactante.
B) Hemorragia cerebral. C)
Síndrome de West.
D) Síndrome de Lenaux-Gastaut. E)
Fenilcetonuria.
64. Ante un lactante de 1,5 meses de edad que presenta ictericia
debemos pensar en todos los siguientes cuadros, excepto en:
A) Infección urinaria. B)
Lactancia materna.
C) Atresia congénita de vías biliares. D)
Ictericia fisiológica.
E) Hipotiroidismo congénito.
65. Un paciente acude por cuadro de dolor intenso epigástrico, de
aparición brusca, acompañado de vómitos en los que sólo consigue
arrojar saliva, gran distensión abdominal alta e imposibilidad para
pasar más allá del esófago distal con una sonda nasogástrica,
¿cuál sería la principal sospecha diagnóstica?: A)
Perforación gástrica.
B) Estenosis pilórica aguda.
C) Vólvulo gástrico agudo.
[8]
Mg. Dany Colca
Lic. Carolina Ayala
RESIDENTADO DE ENFERMERÍA. Compendio Banco de Preguntas.
D) Síndrome de Boerhaave. E)
Tricobezoar.
66. Un paciente de 60 años refiere dolor en epigastrio desde hace
unas tres semanas y en menor medida desde meses antes, que
se alivia con los alimentos y reaparece 2 horas después de las
comidas, con irradiación a hipocondrio derecho. En la gastroscopia
muestra una úlcera de 1 cm. en curvadura menor, con bordes netos
bien definidos, ausencia de islotes de tejido dentro del nicho
ulceroso, con mucosa de aspecto normal. Ante lo cual Ud.:
A) Decide no practicar biopsia de la úlcera por tener características
de benignidad.
B) Sólo practicará biopsia del fondo del nicho.
C) Practicaría biopsias múltiples independientemente de las
características del nicho ulceroso.
D) Algunos de los signos descritos son sugestivos de malignidad, por
lo que practicaría biopsias múltiples.
E) Por no tener características de malignidad claras, no practicaría
biopsias.
67. Paciente de 84 años que presenta cuadro de diarrea mucosa con
decaimiento generalizado y pérdida de peso. Analíticamente el
paciente presenta anemia con hipopotasemia, hiponatremia e
hipocloremia; al tacto rectal se palpa una masa homogénea de
consistencia blanda, recubierta de moco, no dolorosa. Este paciente
presentará con gran probabilidad:
A) Hemorroides.
B) Enteropatía pierde proteínas. C)
Leiomioma de recto.
D) Adenoma velloso de recto. E)
Hamartoma rectal.
68. Mujer de 42 años que acude al hospital con historia de 10 años
de disfagia, primero para líquidos y posteriormente para sólidos, y que
en la actualidad presenta regurgitaciones de carácter principalmente
nocturno. ¿Cuál sería la prueba diagnóstica esencial?:
A) Rx de tórax.
B) Tránsito digestivo. C)
Endoscopia alta.
D) Manometría esofágica. E)
TAC torácica.
69. Un varón de 60 años sin antecedentes personales ni familiares de
interés presenta sangre roja mezclada con las heces. El médico
realiza una inspección perianal y un tacto rectal encontrando
hemorroides internas grado III. La actitud más correcta sería:
A) Medidas higiénicas más pomada anti hemorroidal. B) Solicitar
endoscopia alta.
C) Solicitar un enema opaco.
D) Solicitar rectoscopia y enema opaco. E)
Solicitar colonoscopia total.
70. Paciente de 43 años en tratamiento por colitis ulcerosa con
corticoides y sulfasalazina; acude a urgencias por malestar
general, fiebre, distensión
abdominal, náuseas y vómitos. Presenta gran distensión abdominal,
intenso dolor a la palpación y signos de irritación peritoneal. En la
analítica presenta marcada leucocitosis con desviación izquierda. En
la RX simple de abdomen se observa gran dilatación del colon. El
paso siguiente es:
A) Intervención quirúrgica. B)
Rectocolonoscopia.
C) Inmunosupresores.
D) Sonda nasogástrica, sueros y corticoides iv. E) TAC
abdominal.
71. Una mujer de 52 años diagnosticada de úlcera gástrica de 2 cm.
con biopsia negativa por malignidad, a las 8 semanas de tratamiento
con ranitidina 150
mg cada 12 horas, se somete a control endoscópico encontrándose
una úlcera de 0,5 cm, con nueva biopsia negativa y totalmente
asintomática. ¿Cuál es la conducta preferida?:
A) Cambiar el medicamento a fenotidina. B)
Añadir curalfato a la ranitidina.
C) Remitir por cirugía.
D) Continuar con ranitidina 8 semanas más. E)
Suspender toda medicación.
72. Una mujer de 76 años presenta vómito en posos de café. En la
endoscopia se encontró un pólipo en cuerpo gástrico. No se identificó
ningún otro origen de la hemorragia. El hematócrito es del 28%.
¿Cuál es la mejor opción terapéutica?:
A) Biopsia con pinzas por estudio histológico, si es benigna
ninún tratamiento adicional.
B) Biopsia con pinzas, si es benigna, antagonistas H2. C)
Polipectomía endoscópica con asas.
D) Resección quirúrgica.
E) Actitud expectante por si se repitiese la hemorragia.
73. Un paciente de 45 años con anemia ferropénica y colonoscopia
normal se sometió a endoscopia alta observándose un duodeno con
mucosa testoreada sin lesiones sugerentes de hemorragia. La biopsia
mostró atrofia total de vellosidades, lo cual sugiere:
A) Glardiasis.
B) Mucosa normal. C)
Celíaca.
D) Enfermedad de Crohn. E)
Linfoma intestinal.
74. Una mujer de 65 años con artritis reumatoide deformante grave se
presenta en el hospital por dolor periumbilical de inicio nocturno, con
aumento rápido de intensidad. La exploración abdominal era casi
normal con hemocultivo positivo, recuento de leucocitos de
20.000 mm3 con desviación a la izquierda y VSG > 100 mm/h.
¿Cuál es el diagnóstico más probable?:
A) Colitis ulcerosa.
B) Enfermedad de Crohn. C)
Isquemia intestinal.
D) Parasitosis intestinal.
E) Angiodisplasia de colon.
[9]
Mg. Dany Colca
Lic. Carolina Ayala
RESIDENTADO DE ENFERMERÍA. Compendio Banco de Preguntas.
75. Varón de 78 años con debilidad, pérdida de peso, diarrea, artritis
y fiebre, durante el último año. En el examen físico: pérdida de
masa muscular, linfadenopatía y tumefacción en rodilla izquierda. En
pruebas de laboratorio: anemia ferropénica y hemault positivo. Con
Rx abdomen y enema opaco banales. El diagnóstico diferencial debe
incluir todos los siguientes, excepto:
A) SIDA.
B) Enfermedad de Crohn. C)
Vasculitis reumatoide.
D) Enfermedad de Whipple. E)
Singellosis.
76. Se hace colonoscopia en un enfermo, donde aparece un
pólipo de 8 mm en colon sigmoide que se extirpa sin observar otras
lesiones. ¿Cuál de los siguientes considera el intervalo de vigilancia
en este paciente?:
A) Seis meses. B)
Un año.
C) Dos años. D)
Tres años. E)
Cinco años.
77. Niña de 6 años remitida a consulta por hemorragia vginal, que
presenta un desarrollo mamario, en etapa III de Tanner, estatura alta,
con una edad ósea de 9 años (Rx. de mano y muñeca izda.), valores
basales de gonadotropinas y estradiol elevados para la prepubertad
con ovarios aumentados para la edad y con múltiples quistes de
diámetro igual o mayor de 14 mm. Examen neurológico clínico-
radiológico normal, sin pigmentación cutánea en mancha de café con
leche y estudio hormonal tiroideo y suprarrenal normal. El
tratamiento de elección es:
A) Laparocopia diagnóstico-terapéutica ya que su etiología es un
tumor ovárico.
B) Agonistas de GnRH, ya que estamos ante una PPV, y los
agonistas de GnRH son los únicos que retrasan el desarrollo sexual y
la maduración esquelética.
C) Danazol.
D) Acetato de medroxiprogesterona ya que producen un retraso en el
desarrollo sexual y muy buenos resultados en el control del
crecimiento.
E) No precisa tratamiento, aunque sí una vigilancia anual.
78. Una paciente de 30 años, que consulta por amenorrea secundaria,
presenta concentraciones plasmáticas basales de FSH 2 MUI/ml., LH
1,5, MUI/ml., prolactina 9 ngr./ml. Tras la administración de
gestágenos 10 mgr./día durante 5 días no se observa sangrado
vaginal. En cambio, tras la administración de estrógenos conjugados
durante 21 días y en los 5 últimos días gestágenos aparece una
menstruación. De las siguientes causas de amenorrea ¿cuál es la que
corresponde con el cuadro clínico?:
A) Síndrome de ovario poliquístico. B)
Fallo ovárico autoinmune.
C) Prolactinoma hipofisario. D)
Síndrome de Asherman. E) Tumor
cerebral.
79. Una mujer de 54 años solicita tratamiento hormonal sustitutivo por
síntomas neurovegetativos y manifestaciones genitourinarias
importantes. Refiere que la >FUR fue hace un año y medio, pero que
hace 2 meses ha empezado a sangar muy abundantemente. Presenta
exploración física y mamografía normal. ¿Cuál es la actitud más
apropiada en esta paciente?:
A) Tratamiento con gestágenos. B)
Tratamiento con estrógenos.
C) Tratamiento con terapia combinada.
D) Ecografía transvaginal para valorar línea media y si existe alguna
duda de patología endometrial realizar histología endometrial.
E) Citología (triple toma).
80. Una mujer de 45 años con el antecedente de un proceso gripal
hace dos semanas por el que fue tratado con amoxicilina presenta
ahora un cuadro de prurito vulvar y leucorrea. En la exploración se
aprecia enrojecimiento y edemas de la vulva y del introito y
secreción vaginal blanca grumosa de aspecto caseoso. En la mucosa
vaginal aparecen unas placas blanquecinas irregulares que se
desprenden con facilidad y dejan ulceraciones rojas superficiales.
¿La etiología probable es?:
A) Candidiasis o moniliasis. B)
Herpes genital.
C) Tricomaniasis.
D) Gardnerella vaginalis. E)
Clamidias.
81. Mujer de 40 años que consulta por cuadro de poliartritis simétrica
de grandes y pequeñas articulaciones de 15 días de evolución. En la
analítica destaca un factor reumatoide positivo siendo el resto del
estudio inmunológico negativo. ¿Qué diagnóstico realizaría?:
A) Artritis reumatoide.
B) Artritis paraneoplásica. C)
Artritis no filiada.
D) Enfermedad de Still del adulto. E)
Poliartritis vírica.
82. Mujer de 65 años que ingresa por cuadro de cefalea, fiebre y dolor
con limitación de ambos hombros y ambas caderas de 2 meses
de evolución. El resto de anamnesis y exploración física no aporta
datos relevantes. La analítica pone de manifiesto una gran elevación
de los reactantes de fase aguda (VSG y PCR). La conducta a
seguir sería:
A) Diagnosticar a la paciente de polimialgia reumática e iniciar
tratamiento.
B) Realizar biopsia de la arteria temporal para descartar la existencia
de arteritis pues el tratamiento difiere.
[10]
Mg. Dany Colca
Lic. Carolina Ayala
RESIDENTADO DE ENFERMERÍA. Compendio Banco de Preguntas.
C) Iniciar tratamiento con prednisona 1 mg./kg./día para “curarse en
salud” ante la posible existencia de una arteritis.
D) Realizar artrocentesis de un hombro o una cadera para descartar
primero la existencia de una artritis séptica.
E) Iniciar tratamiento con 20 mg./día de prednisona y si no mejorara
en 15 días subir a 1 mg./kg./día.
83. Varón de 25 años afecto clínica y radiológicamente de una
sacroileítis unilateral de 2 meses de evolución. La conducta a seguir
es la siguiente:
A) Solicitar analítica con HLA y dar tratamiento con AINE ante la
sospecha diagnóstica de espondiloartropatía.
B) Además de lo anterior añadir salazopirina al tratamiento pues
probablemente padecerá una espondilitis anquilosante.
C) Además de lo anterior realizar una buena anamnesis y exploración
física para descartar la existencia de psoriasis.
D) Realizar anamnesis y exploración física, solicitar analítica
con HLA así como ppD y serologías a brucella. E) Lo primero es
realizar una artrocentesis de la articulación afecta para
descartar proceso infeccioso crónico.
84. Mujer de 45 años que acude a la consulta por “dolor óseo
generalizado” desde hace varios años. La conducta a seguir es:
A) Diagnosticarla de fibromialgia y tratarla con analgésicos y
antidepresivos.
B) Sospechar la posible existencia de proceso metastásico y realizar
un completo estudio de búsqueda del tumor primario.
C) Realizar una correcta anamnesis, exploración física y solicitar
analítica completa con hormonas tiroideas y CPK.
D) Además de lo explicado en el apartado 3., solicitar una
gammagrafía ósea para asegurarse de la existencia de proceso
inflamatorio articular.
E) Realizar una anamnesis y exploración física correctas.
Solicitar un estudio analítico con VSG y PCR. Cuando exista
sospecha clínica solicitar determinación de hormonas tiroideas,
anticuerpos anti-ANA y creatin- P-quinasas.
85. Varón de 45 años que acude a la consulta por presentar dolor,
tumefacción y edema a nivel de la mano izquierda. Entre sus
antecedentes patológicos únicamente destaca la luxación del hombro
izquierdo 1 mes antes. La conducta a seguir es la siguiente:
A) Realizar artrocentesis de la muñeca para descartar artritis séptica.
B) Solicitar analítica y radiografías ante la sospecha de artritis de
muñeca.
C) Solicitar analítica, radiografías y gammagrafía ósea con tecnecio
ante la sospecha de DSR.
D) Solicitar analítica con inmunología y radiografías ante
la sospecha de inicio de proceso inflamatorio crónico.
E) Solicitar analítica, radiografías y gammagrafía con tecnecio y
galio ante la sospecha de DSR.
86. Mujer de 20 años que acude por dolor y tumefacción de ambos
tobillos junto lesiones cutáneas eritematovioláceas dolorosas en
ambas EEII de 10 días de evolución. El diagnóstico más probable es:
A) Artritis reactiva. B)
Vasculitis.
C) Sarcoidosis.
D) Enfermedad inflamatoria intestinal. E)
Artropatía no filiada.
87. Una paciente de 55 años sin antecedentes patológicos de interés
consulta por dorsalgia de inicio brusco y ritmo mecánico de dolor.
Aporta Rx simple de columna en la que se aprecian aplastamientos
vertebrales múltiples. La exploración física es normal salvo por la
presencia de debilidad muscular proximal. Ante la sospecha clínica de
osteomalacia, cuál de las siguientes exploraciones es más rentable:
A) VSG y hemograma.
B) Determinación de calcio, fosforo y fosfatasas alcalinas.
C) Densitometría ósea. D)
Gammagrafía ósea.
E) Resonancia magnética.
88. Una paciente de 23 años con antecedentes de ulcus duodenal
presenta una poliartritis simétrica con afección predominante de
manos. En la exploración física, aparte de la poliartritis, presenta aftas
orales. Se practica analítica general que muestra como únicas
alteraciones VSG: 40 mm/1.ª hora, leucocitos: 3.000/mm3 (1.200
linfocitos) y ANA + 1/320 patrón homogéneo. ¿Cuál sería su
diagnóstico?:
A) Artritis reumatoide.
B) Lupus eritematoso sistémico. C)
Gota poliarticular.
D) Condrocalcinosis. E)
Artritis reactiva.
89. ¿Cuál es el tratamiento indicado en la paciente de la pregunta
anterior?:
A) Antiinflamatorios no esteroideos. B)
Antipalúdicos.
C) Glucocorticoides orales.
D) Pulsos de metilprednisolona. E)
Inmunosupresores.
90. Un paciente de 45 años consulta porque en una analítica de rutina
se ha detectado una uricemia de 9 mg./dl. No existen antecedentes
de artritis ni cálculos urinarios. ¿Cuál es la actitud correcta?:
A) Iniciar tratamiento con uricosúricos.
B) Iniciar tratamiento con uricosúricos y colchicina. C) Iniciar
tratamiento con alopurinol.
D) Iniciar tratamiento con alopurinol y colchicina. E) No
precisa tratamiento.
[11]
Mg. Dany Colca
Lic. Carolina Ayala
RESIDENTADO DE ENFERMERÍA. Compendio Banco de Preguntas.
91. Una paciente de 15 años es remitida por sospecha de fiebre
reumática: tras un cuadro gripal con artromialgias generalizadas se
determinaron las antiestreptolisinas (ASLO), que son positivas. ¿Cuál
de las siguientes afirmaciones es verdadera?:
A) Ante el cuadro clínico de la paciente y las ASLO +, el diagnóstico
de fiebre reumática es seguro.
B) Con las ASLO +, bastaría para diagnosticar la fiebre reumática.
C) La fiebre reumática es un tipo de artritis séptica.
D) Con estos datos, no se puede realizar el diagnóstico de fiebre
reumática.
E) Debemos iniciar rápidamente tratamiento con penicilina.
92. Una paciente de 40 años presenta una gonartosis con afección
predominante del compartimento interno de la rodilla e impotencia
funcional severa. Señale la respuesta verdadera:
A) El tratamiento es reposo absoluto y esperar a que sea más
mayor para colocarle una prótesis.
B) Una osteotomía podría estar indicada.
C) Nunca se debe tratar con antiinflamatorios.
D) No debe realizar fisioterapia ya que podría dañarse más la
articulación.
E) El uso de un bastón está contraindicado.
93. Un paciente presenta latencia del sueño de menos de 10 minutos
con dificultades para despertarse y episodios de sueño de 18-20
horas, asociado al despertarse con hiperfagia, hipersexualidad,
aumento de peso, irritabilidad, depresión, comportamiento impulsivo,
disfunción vegetativa y alteraciones neurológicas. Las siestas
diurnas duran varias horas. Estos episodios duran varias semanas
intercalándose varios meses sin somnolencia. En uno de los
episodios de somnolencia el paciente tuvo un grave accidente. El
diagnóstico más probable es:
A) Epilepsia.
B) Narcolepsia.
C) Simulación o trastorno conversivo-histeria. D)
Síndrome de Klein-Levin.
E) Apnea del sueño.
94. Una chica de 17 años acude al servicio de urgencias traída por su
madre por un desmayo. Cuando la vemos está irritable y dice querer
irse de alta porque no le pasa nada. Va con ropas holgadas y algo
más abrigada que el resto de pacientes, está delgada y parece más
joven de lo que le corresponde por su edad. Dice comer normal
y niega usar laxantes o diuréticos, su madre dice que no come con
ellos porque es muy activa y está todo el día fuera. Rompió con el
novio hace un año. Dice que está harta de tener a su madre siempre
pendiente de ella. El dato que más nos ayudaría a diferenciar una
anorexia nerviosa de otros trastornos sería:
A) Presencia de amenorrea. B)
Peso inferior al 15%.
C) Que la paciente diga que tiene miedo a ganar peso o
que diga que se ve gorda.
D) Que diga que tiene frío. E)
Uñas y pelo frágil.
95. Una adolescente de apariencia física normal acude por irritabilidad
y tristeza. Comenta que decidió comer menos en las comidas porque
estaba algo gorda. Haciendo esto consigue adelgazar a temporadas
pero luego se vuelve a engordar. Ahora está en período de transición,
sigue restringiendo las comidas pero se pasa todo el día picoteando
y en ocasiones come más de lo que quisiera de forma descontrolada.
En esos descontroles efectivamente la paciente come una gran
cantidad de alimentos especialmente chocolate, galletas y dulces,
aunque en otras ocasiones son salados. Tras estos atracones se
siente muy culpable e irritada y vomita para evitar engordarse. El
diagnóstico más probable es:
A) Síndrome de Klein-Levin. B)
Depresión bipolar.
C) Bulimia.
D) Esquizofrenia.
E) Anorexia nerviosa tipo compulsivo/purgativo.
96. Una paciente de 20 años se presenta en la guardia quejándose al
internista de que tiene ataques de corazón y sensación de ahogo,
sudoración y sensación de mareo que se inicia bruscamente mientras
estaba leyendo relajadamente en su casa, le duró unos minutos,
creyó morir y sentía que lo que le ocurría no era real, temiendo
perder el control o estar volviéndose loca. Se le realizan pruebas ECG
y auscultación, detectándose frecuencia cardíaca alta y signos
sugerentes de prolapso mitral. Aun así dada la gran ansiedad de la
paciente se llama al psiquiatra ya que insiste en que se le hagan más
pruebas y en que no se quiere quedar sola en casa nunca más,
¿cuál es el posible diagnóstico?:
A) Prolapso mitral. B)
Agorafobia.
C) Angor.
D) Trastorno de angustia. E) A
y D.
97. Una paciente de 55 años acude a consultas por cansancio
especialmente por la mañana, falta de concentración desde hace 1
mes. Dice estar por las mañanas despierta antes de hora sin poder
dormir. Ha perdido apetito si bien come como siempre pero
forzándose, por lo que no ha perdido peso. Pierde el hilo de las
conversaciones y está irritable. Ha dejado de hacer las cosas que
le gustaba hacer y no sale de casa más que lo justo. Viene
acompañada por su hermana y nos ruega que no le digamos nada a
su marido ya que desde que no hace las cosas de la casa tan bien
como antes no quiere darle más disgustos. ¿Cuál es el diagnóstico
más probable?:
A) Ansiedad y depresión. B)
Demencia.
C) Depresión mayor.
D) Trastorno de personalidad.
[12]
Mg. Dany Colca
Lic. Carolina Ayala
RESIDENTADO DE ENFERMERÍA. Compendio Banco de Preguntas.
E) Neurosis.
98. El paciente, que es traído por un familiar, dice no saber porqué
está aquí y parece irritado y con agresividad contenida. El familiar nos
hace gestos con los ojos haciendo ver que el paciente no está muy
bien. Entrevistando a ambos por separado el paciente se muestra
contenido y dice estar bien, mejor que nunca. El familiar dice que
últimamente ha comprado cosas innecesarias y ha hecho algunos
regalos. Dice que quiere iniciar la carrera de psicología, se ha
apuntado a un gimnasio y quiere proponer a su jefe algunos
cambios en la empresa si bien no tiene responsabilidades directivas.
Está muy hablador y agudo en las conversaciones pero irritable,
dominante y con explosiones de mal genio. Debe dormir 4 horas pero
no está cansado al día siguiente, no hay problemas de apetito. No
ha habido problemas en su trabajo, pero pasa mucho tiempo en
la calle. ¿Cuál es el diagnóstico más correcto?:
A) Esquizofrenia. B)
Manía.
C) Depresión ansiosa.
D) Trastorno esquizoafectivo. E)
Trastorno de personalidad.
99. Paciente traído por la policía por haberlo encontrado andando
descalzo por la carretera. El paciente tiene 28 años, es de otra
ciudad y por la documentación que lleva parece que pueda ser
estudiante. Parece deshidratado, completamente desaseado y con
olor a orín. Pupilas normales. La piel está tostada sin quemaduras en
las zonas de exposición al sol. Está ausente, como absorto de manera
que al hablarle más fuerte se sobresalta y nos atiende
momentáneamente sonriéndonos de forma inapropiada hasta que
ladea la cabeza y la gira sin motivo. Murmura algo y mira al
techo, permaneciendo con los brazos extendidos. ¿Cuál es el
diagnóstico más probable?:
A) Intoxicación por LSD. B)
Esquizofrenia.
C) Demencia.
D) Cuadro maníaco. E)
Histeria.
100. Acude la madre de un presunto paciente de 18 años porque
su hijo no sale casi nada de casa, parece evitarles. Se hace su
propia comida a partir de conservas generalmente y come aparte.
Nunca usa conservas o alimentos ya empezados, ni acaba los que no
usa por completo. Alguna vez lo ha visto en el pasillo haciendo
posturas extrañas o dando golpes de kárate. Fue aficionado a las
lecturas de ovnis y ciencia ficción. Fue regular estudiante. No trabaja
ni estudia. Cierra todas las persianas y se queja de que los vecinos
están demasiado pendientes de ellos. Por las noches a veces no
duerme. Dice que puede oír al vecino del tercero (ellos viven en el
primero) hablar mal de ellos y que
siempre está comentando lo que él va haciendo. ¿Cuál es el
diagnóstico más probable?:
A) Esquizofrenia.
B) Depresión psicótica. C)
Manía.
D) Demencia. E)
Klein-Levin.
TEMA B
1. Paciente de 64 años ingresado para cirugía diferida que al día
siguiente del ingreso presenta cuadro de hiperactividad vegetativa,
temblor, sobresaltos, hiperactividad e hiperalerta pero con confusión,
alucinaciones y convulsiones. ¿Cuál es la causa más frecuente?:
A) Abstinencia a alcohol.
B) Sobredosis de neurolépticos.
C) Sobredosis de benzodiacepinas. D)
Sobredosis de anticolinérgicos. E)
Ingestión de barbitúricos.
2. Mujer de 25 años que acude por intento autolítico con
benzodiacepinas. Es el quinto intento en 7 años, en esta ocasión tras
una pelea familiar. Su madre dice que es muy voluble y caprichosa
que siempre consigue salirse con la suya mediante grandes crisis de
nervios y peleas. La madre dice estar harta ya que siempre está
regañándola mientras que su marido la perdona todo y no la castiga
lo suficiente. No tiene novio fijo ya que rompe con ellos en seguida,
es mala estudiante y no es capaz de dedicarse a nada fijo ya que se
aburre en seguida, dice sentirse en ocasiones muy vacía y en
otras muy inspirada en hacer el bien pero nada le dura mucho,
quisiera poder tener una relación con un hombre romántico. Sale
mucho por la noche y en ocasiones no llega hasta el día siguiente
por la tarde, llegando ebria en ocasiones. Ha perdido a sus
amigas, que se quejaban entre otras cosas de que las estaba
llamando continuamente. Le han diagnosticado previamente de
hepatitis. El diagnóstico de esta paciente es:
A) Esquizofrenia. B)
Manía.
C) Trastorno de personalidad límite o inestable. D)
Trastorno de personalidad por dependencia.
E) Trastorno esquizoide.
3. El paciente acude a nuestra consulta porque quisiera dejar de hacer
algunas de las cosas que actualmente hace ya que le quitan mucho
tiempo. El paciente tarda mucho tiempo en vestirse ya que ha de
pensar la ropa que le dará suerte para esa mañana. Teme que si no
coge la adecuada algo podría salir mal. No puede evitar sumar los
números que ve en las matrículas de los coches, así consigue
neutralizar los pensamientos que le vienen sobre la posibilidad de
que el coche tenga un accidente. El sabe que éstos son tonterías
suyas pero son cosas que no puede quitarse de la cabeza. Es muy
ordenado en algunos aspectos pero se le acumula el
[13]
Mg. Dany Colca
Lic. Carolina Ayala
RESIDENTADO DE ENFERMERÍA. Compendio Banco de Preguntas.
trabajo y no consigue hacer las cosas que tiene preparadas en una
lista para ese día. Le gusta corregir sus escritos hasta que estén
impecables, lo mismo le ocurre con su trabajo, es muy perfeccionista,
exigente consigo mismo y con los demás, es incapaz de delegar
trabajo. El diagnóstico del paciente es:
A) Esquizofrenia.
B) Trastorno de personalidad dependiente. C)
Trastorno obsesivo-compulsivo.
D) Fobia.
E) Depresión.
4. Paciente que acude por no poder salir de casa sola. Dice que le da
miedo salir por sitios solitarios por si le ocurre algo y que en los sitios
con mucha gente se agobia y se pone muy nerviosa teniendo crisis
de ansiedad con palpitaciones, se imagina así misma necesitada de
ayuda y sin poder salir. Si se encuentra acompañada se encuentra
mejor ya que esa persona podría ayudarla si ocurriera algo. Desde
hace unos meses tiene unas crisis parecidas pero más cortas estando
relajada, incluso acompañada, sin aviso, la primera estando en la
calle distraída viendo un escaparate. Cuando le dan en casa es capaz
de salir a la calle para coger aire o bien abre las ventanas. Desde la
primera crisis cogió algo de miedo a estar sola hasta llegar a la
situación actual. Se ha comprado un teléfono móvil para poder pedir
ayuda si fuera necesario. Si entra en un cine siempre está pendiente
de sentarse cerca de alguna salida. El cuadro clínico de la paciente es:
A) Agorafobia. B)
Hipocondría.
C) Trastorno facticio. D)
Trastorno obsesivo.
E) Agorafobia con crisis de pánico.
5. Ante un paciente con dolor epigástrico irradiado en hemicinturón,
náuseas, distensión abdominal, descenso de los ruidos hidroaéreos y
ascenso del ST en el electrocardiograma,¿cuál sería su actitud?:
A) Llamaría de inmediato a la Unidad de Cuidados
Intensivos para tratar el infarto agudo de miocardio.
B) Repetiría el electrocardiograma a las 8 horas para confirmar el
diagnóstico.
C) Lo diagnosticaría de pancreatitis aguda.
D) Su diagnóstico sería aneurisma disecante de aorta. E) El
paciente tiene una pericarditis aguda.
6. Ingresa en el Servicio de Urgencias un paciente con dolor
epigástrico irradiado en hemicinturón y amilasa elevada, que pierde
de forma brusca visión ¿qué pensaría que ha ocurrido?:
A) Retinopatía de Purtscher.
B) Accidente cerebro vascular agudo de la región occipital
posterior.
C) Desprendimiento de retina. D)
Simulación.
E) Glaucoma agudo.
7. Ante una paciente que ingresa en el Servicio de Urgencias con
dolor abdominal, elevación de la amilasa y la glucosa en suero y con
un pH de 7,1 ¿en qué pensaría primero?:
A) Pancreatitis aguda.
B) Rotura de embarazo ectópico. C)
Quiste de ovario.
D) Cetoacidosis diabética. E)
Ulcera de estómago.
8. Un paciente de 18 años consulta por ictericia sin fiebre y
coluria sin prurito. En las pruebas complementarias presenta: Ac. anti
VHA IgG positivos, Ac. antiHBs positivos, Ac. anti HBc IgG positivos,
ecografía hepática normal, hay predominio de la bilirrubina directa
con monoconjugados, la colecistografía oral es normal y al dar al
paciente fenobarbital se observa disminución de las cifras de
bilirrubina total. ¿Cuál sería su diagnóstico de sospecha?:
A) Coledocolitiasis.
B) Síndrome de Gilbert.
C) Síndrome de Dubin-Johnson.
D) Hepatitis aguda por VHA, con hepatitis aguda por
VHB curada.
E) Síndrome de Rotor.
9. Ante un paciente con cirrosis alcohólica en estadío C-
10 de Child-Pugh que ingresa por descompensación hidrópica, y
que se encuentra hipotenso, taquicárdico, oligúrico, con un sodio
sérico de 125 mEq/l. y de 5 mEq/l. en orina, con sedimento normal,
creatinina sérica
3 mg./dl. y un aclaramiento de creatinina de 40 ml., ¿en qué pensaría
usted?:
A) Deficiente tratamiento diurético.
B) Insuficiencia renal prerrenal secundaria a tercer espacio.
C) Síndrome hepatorrenal.
D) Glomerulonefritis mesangial IgA.
E) Insuficiencia renal prerrenal secundaria a deshidratación.
10. Paciente varón de 28 años, VIH(+), en tratamiento con DDI
(Dideoxinosina), que acude al hospital por dolor intenso en epigastrio
irradiado hacia la espalda, que mejora al flexionar el tronco,
acompañado de náuseas y vómitos. ¿Cuál de los siguientes
diagnósticos es el más probable?:
A) Pericarditis.
B) Obstrucción intestinal. C)
Apendicitis.
D) Pancreatitis aguda. E)
Endocarditis.
11. Tras una semana de ingreso por una pancreatitis aguda, a pesar
del tratamiento médico sintomático persiste la fiebre, leucocitosis e
hiperamilasemia. Se aprecia a la palpación una masa abdominal
localizada en hipocondrio derecho. ¿Cuál es el diagnóstico más
probable?:
[14]
Mg. Dany Colca
Lic. Carolina Ayala
RESIDENTADO DE ENFERMERÍA. Compendio Banco de Preguntas.
A) Carcinoma de páncreas. B)
Flemón pancreático.
C) Pancreatitis crónica.
D) Plastón secundario a perforación duodenal. E) Quiste
hidatídico.
12. El principal diagnóstico de presunción ante una enferma que
acude por un cuadro de diarrea acuosa, dos úlceras duodenales
resistentes al tratamiento médico e hipercalcemia es:
A) Insulinoma. B)
Gastrinoma.
C) Adenocarcinoma de páncreas. D)
Somatostatinoma.
E) Vipoma.
13. Varón de 60 años que acude por pérdida de peso de
10 kilos, dolor sordo en piso abdominal superior de 3 meses de
evolución, acompañado de ictericia mucocutánea y deposición de
color blanco desde hace 5 días. A la exploración destaca masa
palpable en hipocondrio derecho. ¿Qué patología se sospecharía en
primer lugar?:
A) Adenocarcinoma pancreático. B)
Tumor gástrico.
C) Coledocolitiasis. D)
Colecistitis.
E) Hepatitis aguda.
14. Varón de 45 años, etilismo crónico, con dolor abdominal en el
hipocondrio izquierdo, de 3 meses de evolución, que empeora con la
ingesta y se acompaña de deposiciones diarreicas pastosas muy mal
olientes. En las exploraciones complementarias destaca: glucemia
de 280, amilasemia en los límites de la normalidad y en la placa de
abdomen múltiples calcificaciones a nivel de L2:
A) Pancreatitis aguda. B)
Cólico biliar.
C) Pancreatitis crónica. D)
Hepatitis aguda.
E) Ulcus gástrico
15. Paciente de 60 años con fibrilación auricular en tratamiento con
amiodarona desde hace meses, colelitiasis diagnosticada
ecográficamente e insuficiencia renal crónica moderada. Consulta por
cuadro de anorexia, astenia, náuseas, vómitos e ictericia
mucocutánea. En los datos de laboratorio destaca un leve aumento
de las transaminasas y una creatinina de
2,2. Se realiza una biopsia hepática, observándose al microscopio
electrónico cuerpos lisosómicos lamelares cargados de fosfolípidos.
Qué proceso patológico le sugieren estos datos:
A) Hepatitis viral aguda. B)
Cólico biliar.
C) Pancreatitis aguda.
D) Hepatitis tóxica de origen medicamentoso. E)
Hepatitis viral crónica.
16. Un varón de 13 años que había presentado un cuadro febril de
vías respiratorias altas de una semana de evolución acude a
urgencias por vómitos súbitos e incoercibles. La semana previa había
consumido ácido acetilsalicílico para la sintomatología respiratoria y
tres semanas antes estuvo en contacto con un paciente con hepatitis
aguda por virus B. Dos días después del ingreso el paciente se
encuentra estuporoso con convulsiones sin signos neurológicos de
focalidad y dolor en hipocondrio derecho con hepatomegalia. ¿Cuál
sería su diagnóstico?:
A) Cuadro convulsivo en relación con la fiebre. B)
Hepatitis fulminante vírica.
C) Síndrome de Reye.
D) Reagudización del cuadro gripal.
E) Intoxicación por ácido acetilsalicílico.
17. Un hombre de 21 años recibió un golpe en el escroto dos horas
antes de ser examinado en urgencias. Su escroto está tenso,
hinhado, y equimótico. No se pueden palpar los testículos. El paso
siguiente es:
A) Hacer uretrografía retrógrada.
B) Hacerle un Doppler de flujo color. C)
Realizar una ecografía del escroto.
D) Tratarle con hielo, reposo y suspensorio. E) Hacer
exploración quirúrgica del escroto.
18. Un hombre de 64 años presenta una hinchazón indolora del
testículo derecho de tres meses de duración. Los resultados de
los análisis de orina son normales y la ecografía testicular muestra un
aumento del tamaño de dicho teste.
El diagnóstico más probable es: A)
Linfoma testicular.
B) Leucemia linfática crónica. C)
Seminoma espermatocítico. D)
Teratocarcinoma.
E) Carcinoma de células embrionarias.
19. Una mujer sana presenta de forma aguda polaquiuria y
disuria. En el sedimento urinario se observan más de 5 leucocitos
por campo y el urinocultivo revela
1.000 colonias de E. coli por ml. El diagnóstico más probable
es:
A) Uretritis por clamydia. B)
Síndrome uretral.
C) Bacteriuria por E. coli. D)
Cistitis intersticial.
E) Cistitis quística.
20. Un hombre de 74 años con cáncer de próstata metastásico
conocido presenta dolor agudo de cadera derecha. Hace dos años se
le practicó una orquiectomía pero no ha recibido ningún otro
tratamiento. Su estado general es bueno. Una gammagrafía ósea
muestra metástasis difusas y una Rx simple revela una osteólisis en
el acetábulo derecho. La siguiente medida a aplicar es:
A) Flutamida.
[15]
Mg. Dany Colca
Lic. Carolina Ayala
RESIDENTADO DE ENFERMERÍA. Compendio Banco de Preguntas.
B) Análogos de la LH-RH. C)
Radioterapia localizada. D)
Fosfato de estramustina. E)
Ketoconazol.
21. Un recién nacido presenta hematuria, proteinuria y creatinina
elevada. Las presiones de la arteria umbilical están significativamente
altas y el paciente desarrolla una insuficiencia cardíaca congestiva.
Una gammagrafía renal revela una ausencia de función en el riñón
izquierdo. El diagnóstico más probable es:
A) Trombosis de la vena renal. B)
Necrosis cortical renal.
C) Hemorragia adrenal.
D) Rotura de un nefroma mesoblástico. E)
Trombosis de la arteria renal.
22. Un hombre de 35 años presenta un cólico renal izquierdo. En la
Rx de abdomen se observa un cálculo de 3 mm. de diámetro alojado
en uréter medio. La necesidad de tratamiento quirúrgico depende de:
A) La duración de los síntomas del paciente.
B) Del número de episodios de cólicos previos.
C) Del número de intervenciones quirúrgicas previas. D) De la
presencia de fiebre e infección urinaria.
E) De una anormalidad metabólica subyacente.
23. A un paciente se le somete a una intervención de bypass en el
intestino delgado y presenta una litiasis urinaria. ¿Cuál será la
composición más probable de la litiasis?:
A) Acido úrico.
B) Urato amónico. C)
Fosfato cálcico. D)
Oxalato cálcico. E)
Estruvita.
24. El factor pronóstico más importante en los niños que presentan
un tumor de Wilms intracava es:
A) La histología.
B) El volumen del tumor.
C) La extensión atrial del tumor.
D) La afectación de ganglios linfáticos.
E) La diseminación del tumor durante la intervención quirúrgica.
25. Un muchacho de 7 años presenta de forma súbita dolor escrotal
derecho de 4 horas de duración. Se sospecha de una torsión
testicular. ¿Cuál de las siguientes observaciones es la más probable?:
A) La ausencia del reflejo cremastérico.
B) El aumento de la captación por parte del teste derecho
después de la exploración radioisotópica.
C) La presencia del reflejo cremastérico.
D) Una transiluminación correcta del compartimento escrotal
derecho.
E) La estetoscopia con Doppler será simétrica en ambos
compartimentos escrotales.
26. En un paciente asmático, ¿cuál de los siguientes fármacos está
contraindicado en el tratamiento de la incontinencia?:
A) La efedrina. B)
Sudafed.
C) El propranolol.
D) La fenilefrina.
E) Las anfetaminas.
27. Ante un paciente de 60 años con un cáncer de próstata
localizado, mal diferenciado y sin metástasis, qué tratamiento le
recomendaría para intentar aumentar su supervivencia:
A) Prostatectomía radical. B)
Análogos de la LH-RH. C)
Flutamida.
D) Estramustina. E)
Orquiectomía.
28. Ante un paciente de 55 años que presenta una tumoración vesical
que infiltra la muscular y que ocupa la mitad de la vejiga, ¿cuál
sería el tratamiento que emplearía con intención curativa?:
A) Cistectomía radical y derivación urinaria. B)
Resección transuretral vesical.
C) Quimioterapia intravesical. D)
Quimioterapia sistémica.
E) Cistectomía parcial.
29. Paciente mujer de 75 años de edad, que es traída a la consulta
por cambio progresivo de conducta en los últimos meses.
Previamente habían observado fallo de memoria reciente sin poder
precisar el momento de inicio. Destacaba a la exploración lenguaje
pobre y fallos de juicio. ¿Cuál es el diagnóstico más probable en esta
paciente?:
A) Depresión.
B) Síndrome confusional agudo. C)
Demencia tipo Alzheimer.
D) Demencia multiinfarto.
E) Trastorno de la personalidad.
30. Varón de 87 años, con amaurosis bilateral, que ingresa por
neumonía basal derecha en la Unidad de Agudos del hospital. La
noche del ingreso presenta cuadro de agitación psicomotriz, con
alteración del nivel de conciencia y desorientación temporoespacial.
¿Cuál es el diagnóstico más probable?:
A) Demencia.
B) Depresión delirante. C)
ACVA.
D) Síndrome confusional agudo. E)
Ninguno.
31. Paciente de 85 años de edad con antecedentes de insuficiencia
cardíaca en tratamiento con diuréticos, e insomnio que trataba con
lorazepam. Presenta nicturia
2-3 veces. Sufre caída al levantarse bruscamente durante la noche
para ir al retrete. ¿Qué factores pueden haber contribuido e la
caída?:
[16]
Mg. Dany Colca
Lic. Carolina Ayala
RESIDENTADO DE ENFERMERÍA. Compendio Banco de Preguntas.
A) Fármacos.
B) Hipotensión postural. C)
Factores ambientales. D)
Ninguno.
E) Todos ellos.
32. Paciente de 75 años de edad, varón, que vive solo. Es encontrado
la mañana del día 15 de enero caído en el suelo. A su llegada al
hospital se evidencia afasia motora y hemiplejía derecha
diagnosticándose de ACVA. ¿Cuál de las siguientes patologías debe
ser descartada en la valoración inicial?:
A) Rabdomiólisis. B)
Hipertiroidismo. C)
Depresión.
D) Hipotermia. E)
A y D.
33. Paciente mujer de 85 años que ingresa por fractura pertrocantérea
de cadera derecha colocándose tracción. Se evidencia neumonía
basal derecha que obliga a retrasar la intervención. La paciente sufre
de incontinencia urinaria que se maneja con catéter. A los cinco días
se observa enrojecimiento en región sacra que no palidece con la
presión, diagnosticándose de úlcera por presión grado I. ¿Cuál es el
principal factor de riesgo para esta complicación?:
A) Incontinencia urinaria. B)
Hipoxemia.
C) Edad avanzada.
D) Inmovilidad. E)
Todos.
34. Paciente de 70 años que sufre caída al suelo golpeándose en la
cabeza. Al cabo de unas semanas sufre trastornos de conducta,
pérdida de memoria y posteriormente alteración del nivel de
conciencia. ¿Qué diagnóstico debe descartarse en primer lugar?:
A) Hipotiroidismo.
B) Demencia tipo Alzheimer. C)
Pseudodemencia.
D) Hematoma subdural. E)
Tumor cerebral.
35. Mujer de 75 años que consulta por incontinencia urinaria, en la
que predomina la urgencia-miccional y que no sigue ningún
tratamiento farmacológico habitual. A través de la exploración física
no se objetiva patología orgánica, siendo el residuo postmiccional
normal. El estudio analítico es normal. El tratamiento médico de
elección sería:
A) Sondaje vesical intermitente. B)
Calcioantagonistas.
C) Cirugía.
D) Antocolinérgicos. E)
Colector externo.
36. Durante los últimos 3 meses un varón de 80 años presenta una
rápida progresión de una demencia acompañada de signos
extrapiramidales y mioclonías. El diagnóstico de sospecha inicial
sería:
A) Demencia senil tipo Alzheimer.
B) Enfermedad de Creutzfeldt-Jakob. C)
Enfermedad de Huntington.
D) Hidrocefalia a presión normal. E)
Enfermedad de Parkinson.
37. ¿Cuál de las lesiones cutáneas siguientes se asocia más
frecuentemente con neoplasia oculta en un paciente anciano?:
A) Penfigoide bulloso. B)
Dermatomiositis.
C) Eritema multiforme. D)
Herpes zoster.
E) Pénfigo vulgar.
38. Anciano de 70 años con un melanoma maligno de reciente
diagnóstico y sin otra patología médica. Su pronóstico vital estará más
estrechamente relacionado con una de las siguientes características:
A) Ausencia de regresión.
B) Tipo histológico de la lesión. C)
Grado de invasión.
D) Presencia de ulceración. E)
Lugar de la lesión.
39. Una mujer de 75 años con historia de diabetes mellitus no
insulindependiente y epilepsía secundaria a enfermedad cerebro-
vascular padece una inflamación con retracción gingival. Se
encuentra en tratamiento con glipizida y fenobarbital. La causa más
probable de su proceso gingival será:
A) Caries dental. B)
Edentulismo.
C) Tratamiento con fenobarbital. D)
Déficit de cinc.
E) Pobre higiene oral.
40. Un varón de 76 años con historia de diabetes mellitus de
larga evolución y con datos clínicos de polineuropatía periférica,
empieza a tomar amitriptilina por prescripción médica para las
parestesias en miembros inferiores. De forma rogresiva nota
disminución del volumen de diuresis y ocasionalmente escapes
involuntarios de orina. En la revisión médica siguiente se objetiva
deterioro del estado general con insuficiencia renal. ¿Cuál es la causa
más lógica de su deterioro clínico?:
A) Nefrotoxicidad por amitriptilina. B)
Pielonefritis aguda.
C) Infección urinaria de vías bajas.
D) Retención urinaria con fracaso renal secundario. E)
Glomeruloesclerosisdiabética.
[17]
Mg. Dany Colca
Lic. Carolina Ayala
RESIDENTADO DE ENFERMERÍA. Compendio Banco de Preguntas.
41. Varón de 45 años con otalgia derecha y sensación de
taponamiento auditivo, sin otorrea. A los dos días presenta aumento
del dolor, que se hace retroauricular, y fiebre en agujas. El Rinne es
negativo en oído derecho y el Weber lo lateraliza a la derecha. El
diagnóstico más probable es:
A) Colesteatoma antral invasivo. B)
Petrositis.
C) Tromboflebitis del seno lateral. D)
Otitis externa maligna.
E) Carcinoma de oído derecho.
42. Mujer de 22 años que presenta parálisis facial periférica derecha.
A la exploración presenta otoscopia normal y lengua geográfica
fisurada. La paciente comenta haber tenido otro episodio
anteriormente, aquella vez asociado a edema de labio inferior. El
diagnóstico más probable es:
A) Síndrome de Guillain-Barré.
B) Síndrome de Heerfordt-Waaldenström. C)
Déficit de C1-inhibidor.
D) Síndrome de Melkerson-Rosenthal. E)
Parálisis facial de Bell.
43. Paciente que presenta hipoacusia neurosensorial izquierda de
larga evolución con caída en agudo, y acúfeno persistente. La actitud
correcta en este caso sería:
A) No hacer nada, pues se trata de un traumatismo acústico
crónico.
B) Potenciales evocados para descartar neurinoma del
VIII.
C) Instaurar terapia vasodilatadora endovenosa, pues se trata de una
hipoacusia súbita.
D) Intervenir el posible colesteatoma.
E) Administrar sedantes vestibulares ante la posibilidad de vértigo
de Ménière.
44. Paciente de 65 años con rinorrea unilateral purulenta,
dolor hemifacial y epistaxis ocasionales. En Rx se observan lesiones
osteolíticas en maxilar. Probablemente se trate de:
A) Cuerpo extraño nasal. B)
Ocena.
C) Granuloma sangrante de tabique. D)
Carcinoma de fosa nasal.
E) Coriza.
45. Un varón de 60 años consulta por tumoración indolora en raíz
nasal, que desplaza la órbita produciéndole diplopia. El cuadro se
debe probablemente a:
A) Mucocele etmoidal.
B) Quiste de retención en seno frontal. C)
Enfermedad de Woakes.
D) Papiloma invertido en techo de fosa nasal. E)
Plasmocitoma solitario en seno frontal.
46. Una mujer de 45 años acude a la urgencia con un síndrome
de disnea y estridor importantes, con tiraje
supraclavicular. A la exploración presenta taquicardia y leve
cianosis, un tiroides agrandado y, en la laringoscopia,
parálisis de ambas cuerdas vocales en posición paramediana. La
actitud que debe seguirse es: A) Bolo de corticoides endovenosos,
ante la posibilidad de carcinoma subglótico.
B) Tiroidectomía de urgencia, pues probablemente el tiroides
agrandado comprime la tráquea.
C) Intubación y observación.
D) Traqueostomía de urgencia, pues se trata de una parálisis
recurrencial bilateral secundaria a patología tiroidea.
E) Administración endovenosa de espasmolíticos.
47. Un niño de 3 años es traído a urgencias con un cuadro de fiebre
y tos irritativa "perruna", a lo que se asocia disfonía y cierto grado de
disnea. El cuadro es compatible con todas menos:
A) Cuerpo extraño en vías aéreas. B)
Laringotraqueítis aguda.
C) Epiglotitis aguda.
D) Adenoamigdalitis aguda obstructiva. E)
Edema alérgico.
48. Paciente mujer de 25 años que consulta por pérdida de audición
de comienzo insidioso. Su madre era sorda y ganó audición tras
operarse del oído. Otoscopia normal. Rinne negativo oído izdo.,
positivo oído dcho. Weber a la izda. Audiometría: hipoacusia
transmisiva izquierda. Reflejo estapedial abolido en oído izdo. y
presente en derecho. Timpanometría normal. El diagnóstico probable
es:
A) Otosclerosis oído izquierdo. B)
Colesteatoma izquierdo.
C) Fijación de cadena osicular derecha. D)
Timpanosclerosis izquierda.
E) Luxación de cadena osicular izquierda.
49. Un varón de 30 años acude a urgencias por vértigo periférico
intenso con nistagmus a la derecha y acúfeno en oído izquierdo.
Recuerda haber tenido déficit auditivo izquierdo previo, que le
desapareció. Audiometría: hipoacusia neurosensorial izda. leve.
Reflejo estapedial: derecho en 70 dB; izquierdo en 50 dB. Diagnóstico
probable:
A) Neuronitis vestibular.
B) Neurinoma del VIII par con reclutamiento positivo. C) Tumor
de tronco cerebral.
D) Otitis media secretora con fístula perilinfática. E)
Síndrome de Ménière.
50. Un niños de 2 años presenta rinorrea purulenta de larga evolución
por fosa nasal derecha, con mala ventilación nasal. Son diagnósticos
posibles todos menos:
A) Tuberculosis nasal.
B) Cuerpo extraño intranasal. C)
Sinusitis maxilar.
D) Coriza común.
E) Rinitis alérgica sobreinfectada.
[18]
Mg. Dany Colca
Lic. Carolina Ayala
RESIDENTADO DE ENFERMERÍA. Compendio Banco de Preguntas.
51. Paciente de 30 años que acude a urgencias presentando una
ulceración amigdalar unilateral. Son diagnósticos probables todos
menos:
A) Amigdalitis tifoidea de Duguet. B)
Angina de Ludwig.
C) Carcinoma escamoso de amígdala.
D) Angina de Plaut-Vincent. E)
Agranulocitosis.
52. Un niños de 12 años acude a urgencias por obstrucción nasal
bilateral crónica y otitis seromucosa bilateral. Recientemente ha
tenido epistaxis importante por ambas fosas nasales. El diagnóstico
más probable es:
A) Hipertrofia adenoidea.
B) Ototubaritis asociada a rinitis alérgica. C)
Poliposis nasal bilateral.
D) Adenocarcinoma de etmoides. E)
Angiofibroma nasofaríngeo.
53. Mujer de 75 años con historia de cefalea presenta pérdida súbita
de visión unilateral con edema de papila ipsilateral. ¿Qué medida
tomaríamos en primer lugar?:
A) TAC.
B) VSG.
C) Ingreso hospitalario paratratamiento antibiótico iv. D)
Observación.
E) Radiografía centrada en el agujero óptico.
54. Mujer de 20 años con ojo rojo bilateral, acompañado de quemosis,
folículos conjuntivales tarsales, adenopatía preauricular, sin pérdida
de visión. La etiología más frecuente será:
A) Adenovirus. B)
S. aureus.
C) H. influenzae.
D) Queratitis herpética. E)
Parainfluenzae virus.
55. Mujer de 23 años que presenta pérdida brusca de agudeza visual
indolora en ojo derecho, en la exploración se observa defecto
pupilar aferente relativo en ojo derecho, segmento anterior normal y
F.O. normal. El diagnóstico más probable es:
A) Neuritis óptica retrobulbar. B)
Histeria.
C) Compresión quiasmática por tumor hipofisario. D)
Compresión del globo ocular por tumor orbitario. E) Glaucoma
agudo de ángulo cerrado.
56. Varón de 50 años con ojo rojo y doloroso, midriasis arreactiva y
cámara anterior estrecha. ¿Cuál de las siguientes respuestas es
falsa?:
A) No sería raro que fuera hipermétrope.
B) Puede presentar cefalea con náuseas y vómitos.
C) Evitar tomar la PIO por posible etiología infecciosa. D) El
tratamiento definitivo es láser o cirugía.
E) Puede presentar visión en halos de colores.
57. Ante un varón de 32 años con síndrome febril y pérdida visual
unilateral que presenta en fondo de ojo lesiones en queso y tomate.
¿Cuál de las siguientes afirmaciones es falsa?:
A) Sería aconsejable realizar serología HIV. B) El
pronóstico vital del enfermo es malo.
C) No necesita tratamiento por ser con frecuencia una alteración
transitoria.
D) Con bastante probabilidad se deberá a CMV. E) La
afectación suele ser bilateral.
58. Paciente de 70 años que presenta metamorfosias, micropsia y
disminución de la agudeza visual de varias semanas de evolución,
el diagnóstico más probable es: A) Catarata nuclear.
B) Catarata subcapsular posterior. C)
Degeneración macular senil.
D) Pars planitis.
E) Hemorragia vítrea.
59. Ante un niño de 5 años con endotropía que ha seguido
tratamiento con corrección óptica y colusiones y cuya agudeza visual
aún no es normal. La pauta a seguir será:
A) Corrección quirúrgica de la endotropía.
B) Continuar las oclusiones sobre ojo con mejor visión. C)
Continuar las oclusiones sobre ojo con peor visión. D) No ocluir
más y pasar a otro tratamiento.
E) Observación.
60. Varón de 70 años que presenta pérdida visual progresiva
unilateral, que precisa cambios sucesivos de corrección óptica
miópica. La causa más probable es:
A) Degeneración macular senil. B)
Glaucoma crónico simple.
C) Vitritis senil.
D) Coriorretinosis senil. E)
Catarata nuclear.
61. Varón de 51 años que sufre súbita pérdida visual total e
indolora en ojo derecho, apreciándose en el F.O. una retina de color
blanquecino con mácula rojo-cereza. El diagnóstico más probable es:
A) Desprendimiento de retina con afectación macular. B)
Enfermedad de Tay-Sacks.
C) Enfermedad de Newman-Pick.
D) Obstrucción de arteria central de la retina. E)
Obstrucción de vena central de la retina.
62. Varón de 10 años de edad que presenta estrabismo, disminución
de agudeza visual y leucocoria en ojo derecho, sin malformaciones
oculares. ¿Qué patología debemos descartar como más probable?:
A) Retinoblastoma.
B) Catarata congénita.
C) Vítreo primario hiperplásico persistente. D)
Enfermedad de Coats.
E) Fibroplasia retrolental.
[19]
Mg. Dany Colca
Lic. Carolina Ayala
RESIDENTADO DE ENFERMERÍA. Compendio Banco de Preguntas.
63. Paciente de 65 años con Diabetes mellitus tipo II de
10 años de evolución con mal control metabólico presenta
disminución de visión brusca unilateral. La causa más probable será:
A) Isquemia macular.
B) Desprendimiento de retina traccional. C)
Hemorragia vítrea.
D) Edema macular.
E) Obstrucción de arteria central de la retina.
64. Varón de 35 años sano de carácter nervioso comienza con
metamorfopsias y escotoma central unilateral, disminución de visión
moderada y patrón angiográfico en chimenea. El diagnóstico más
probable será:
A) Distrofia viteliforme de Best. B)
Retinosis pigmentaria.
C) Degeneración macular ligada a la edad. D)
Enfermedad de Coats.
E) Coriorretinopatía central serosa.
65. Una mujer de 35 años, no fumadora, atleta de fondo y sin
antecedentes personales de interés refiere en los últimos 3 meses
una menor respuesta al ejercicio habitual, con disnea de moderados
esfuerzos. Tos no productiva pero niega fiebre. En una ocasión
reciente presentó la emisión de varios esputos hemoptoicos, que no
se han vuelto a repetir. La Rx de tórax muestra un patrón reticular fino
difuso bilateral y un mínimo derrame pleural derecho. En las pruebas
funcionales llama la atención el incremento de los volúmenes
pulmonares. De los siguientes, ¿cuál le parece el diagnóstico más
probable?:
A) Lupus eritematoso sistémico. B)
Tuberculosis.
C) Sarcoidosis.
D) Linfangioleiomatosis.
E) Enfermedad de Hamman-Rich.
66. Varón de 50 años, no fumador, que refiere disnea de moderados
esfuerzos de unos 4 meses de evolución con tos no productiva.
Ha recibido tratamiento con diuréticos de asa (furosemida) en el
último mes tras realizarse una Rx de tórax (que no aporta). Acude a
urgencias por incremento de la disnea y expectoración de esputos
claros. Refiere febrícula de predominio vespertino y pérdida de unos
6 Kg de peso. Presenta hipoxemia con hipocapnia (insuficiencia
respiratoria parcial) y la Rx de tórax muestra densidades difusas
bilaterales, confluentes, mal definidas de predominio parahiliar con un
índice cardiotorácico en el límite de la normalidad. FVC: 65%, FEV1:
70%, FEV1/FVC: 75%. DLCO: 60%.
A) Insuficiencia cardiaca. Edema agudo de pulmón. B)
Enfermedad de Hamman-Rich.
C) Hemorragia pulmonar. D)
Neumonía por CMV.
E) Carcinoma bronquioalveolar.
67. Varón de 64 años, veterinario de profesión, bebedor esporádico
que presenta fiebre, cefalea y artromialgias desde hace 7 días.
Durante las últimas 48 horas refiere tos productiva, dolor pleurítico en
el costado derecho y disnea progresiva. Ha sido tratado con
eritromicina, pese a lo que se encuentra febril y desorientado. Se
evidencian estertores crepitantes en la base derecha y una
hepatomegalia a 4 cm del reborde costal. En las pruebas
complementarias efectuadas, destaca Hb de
14.8 mg/dL, leucocitosis (17.000/μL) con desviación izquierda, GOT
(AST): 106, GPT (ALT): 82. En la Rx de tórax se aprecia un infiltrado
intersticial en lóbulo inferior derecho. Su diagnóstico de presunción
debe ser:
A) Neumonía neumocócica.
B) Neumonitis por hipersensibilidad. C)
Fiebre Q.
D) Legionella.
E) Tuberculosis.
68. Varón de 37 años que viene presentando durante los tres
últimos meses astenia, esputos hemoptoicos y disnea progresiva
hasta hacerse de mínimos esfuerzos, con intolerancia al ejercicio.
Salvo una TA de 90/60, los datos exploratorios son anodinos. Sin
embargo, los estudios complementarios nos sorprenden: pH: 7.37,
PaO2: 62, PaCO2: 37, HCO3: 27. Hb: 9.2 y creatinina de 2.3. La
Rx de tórax muestra infiltrados difusos parahiliares bilaterales. Ante
los hallazgos reseñados, se añade la petición de un sedimento de
orina, que muestra microhematuria y proteinuria. De las siguientes,
¿qué prueba diagnóstica le parece prioritaria en la evaluación del
enfermo?:
A) Test de difusión (DLCO).
B) Examen citológico (esputo o lavado broncoalveolar). C)
Anticuerpos anti membrana basal glomerular, c- ANCA.
D) Biopsia pulmonar. E)
Biopsia renal.
69. En el caso anterior, la capacidad de difusión del CO está
aumentada y el título de anticuerpos antimembrana basal glomerular
es de 1:128. ¿Cuál le parece el diagnóstico más probable?:
A) Granulomatosis de Wegener. B)
Síndrome de Goodpasture.
C) Tuberculosis.
D) Tromboembolismopulmonar.
E) Granulomatosis de Churg-Strauss.
70. Mujer de 78 años que en el curso de un postoperatorio por fractura
de cadera comienza con un cuadro brusco de disnea y febrícula.
Exploración: taquipnea a 30 r.p.m., taquicardia a 130 l.p.m., refuerzo
del segundo tono, abolición del murmullo vesicular en base de pulmón
derecho y extremidades sin edemas, no dolorosas, sin signos
flogóticos. Complementarios: GAB: pH: 7.52, PaO2: 56, PaCO2: 30,
HCO3: 25. 13.000 leucocitos con desviación izquierda.
Rx de tórax: pinzamiento del seno costodiafragmático derecho.
ECG: Taquicardia sinusal con bloqueo
[20]
Mg. Dany Colca
Lic. Carolina Ayala
RESIDENTADO DE ENFERMERÍA. Compendio Banco de Preguntas.
incompleto de rama derecha. Señale la actitud más adecuada:
A) Diuréticos.
B) Toracocentesis. C)
Antibióticos.
D) Corticoides. E)
Heparina.
71. Mujer de 37 años, fumadora, que consulta por presentar durante
el último mes fiebre, malestar general, artralgias y una erupción
cutánea dolorosa en ambas piernas. Complementarios. Rx de tórax:
adenopatías hiliares bilaterales, sin afectación del parénquima
pulmonar. Mantoux negativo. Se realizó una FBC con lavado
broncoalveolar (LBA). Líquido del LBA: 22% de linfocitos con cociente
CD4/CD8 de 5.2. El diagnóstico más probable es:
A) Tuberculosis. B)
Sarcoidosis. C)
Linfoma.
D) Cáncer de pulmón . E)
Asbestosis.
72. Varón de 47 años que acude a urgencias por fiebre, tos, artralgias
y rinorrea purulenta con ulceraciones de la mucosa nasal de dos
semanas de evolución. Inició tratamiento antibiótico 7 días antes, al
ser diagnosticado por su médico de cabecera de sinisitis
(opacificación de ambos senos maxilares), sin obtener una mejoría
clínica. La Rx de tórax presenta múltiples nódulos pulmonares
bilaterales, algunos de ellos cavitados. En los análisis efectuados
destaca un sedimento de orina con 8 hematíes por campo con
algún cilindro eritrocitario. La biopsia de la mucosa nasal mostró
inflamación granulomatosa con necrosis. El diagnóstico más probable
es:
A) Granulomatosis de Wegener.
B) Granulomatosis de Churg-Strauss.
C) Cáncer de cavum con metástasis pulmonares. D)
Granulomatosis linfomatoide.
E) Tuberculosis.
73. Un grave problema de las unidades de cuidados intensivos son
las infecciones (neumonías y sepsis) por gérmenes gramnegativos
multirresistentes (pseudomonas, serratias, citrobacter, morganella,
acinetobacter, etc.). Ya se han identificado previamente cepas
multirresistentes en nuestra UCI. A falta de un antibiograma, ¿cuál
sería el tratamiento empírico de elección?
A) Ceftazidima, amicamicina y vancomicina. B)
Ceftriaxona y tobramicina.
C) Imipenem o ciprofloxacino. D)
Imipenem y amikamicina.
E) Esperar hasta los resultados del antibiograma.
74. Un varón de 30 años, fumador de 20 cigarrillos/día desde los
20 a los 25 años y ex fumador desde entonces, presenta,
en un reconocimiento laboral, un nódulo pulmonar solitario
(NPS) de unos 2 cm. de diametro en la periferia del LSD. La Rx
de tórax muestra un mediastino normal y no permite
identificar calcificaciones en el NPS. El paciente se encuentra
asintomático y niega la posibilidad de recuperar radiografías
anteriores antes de 6 meses (por cambio de domicilio). ¿Qué actitud
mantendría ante este enfermo? A) Informar de la baja probabilidad
de malignidad y Rx de tórax en 3 meses.
B) Fibrobroncoscopia.
C) Realización de una TAC torácica.
D) Realización preferente de una PAAF con control de
TAC.
E) Insistir en la recuperación de las Rx previas y nueva cita en la
consulta entonces (6 meses).
75. En el caso anterior, la TAC no muestra nuevos datos (confirma la
ausencia de calcificaciones, no adenopatías ni afectación
mediastínica y no existe afectación pleural). La familia del enfermo
ha localizado las Rx de tórax previas (14 meses antes) en la que se
identifica el mismo NPS con un diámetro de 1,4 cm. ¿Cuál sería la
actitud más adecuada?
A) Actitud expectante y repetir pruebas de imagen en 2 meses.
B) Fibrobroncoscopia con citología en las muestras obtenidas.
C) Fibrobroncoscopia y biopsia transbronquial (BTB). D) PAAF
con control de TAC.
E) Toracotomía.
76. En caso de que la PAAF obtenga material suficiente y el resultado
sea de malignidad, ¿qué tipo histológico le parece el más probable?
A) Ca. epidermoide. B)
Adenocarcinoma.
C) Ca células pequeñas (CCP). D) Ca.
bronquioalveolar.
E) Carcinoide.
77. Paciente de 36 años con amenorrea de 10 semanas. Tiene un
antecedente de infertilidad por factor tubárico. Refiere episodios de
dolor cólico hipogástrico desde hace aproximadamente un mes. El
test de embarazo en orina es positivo.
La prueba diagnóstica que solicita a continuación es: A)
Amniocentesis precoz.
B) Ecografía.
C) Laparoscopia.
D) Triple screening.
E) Biopsia de vellosidades coriónicas.
[21]
Mg. Dany Colca
Lic. Carolina Ayala
RESIDENTADO DE ENFERMERÍA. Compendio Banco de Preguntas.
78. Gestante de 16 semanas sin antecedentes de interés que
presenta los siguientes resultados en la analítica de triple screening:
alfafetoproteína 0,3 MM (disminuido), betahcg 1,7 MM (normal),
riesgo estimado de T21 1/43. A continuación se le realiza:
A) Ecografía.
B) Amniocentesis.
C) Biopsia de vellosidades coriónicas. D)
Funiculocentesis.
E) Fetoscopia.
79. Gestante de 26 semanas que consulta por fiebre de
39° C y dolor lumbar unilateral. La analítica de sangre presenta
16.000 leucocitos y desviación izquierda. El tratamiento indicado es:
A) Abundante ingesta de líquidos.
B) Analgésicos orales y abundante ingesta de líquidos. C)
Analgésicos endovenosos y forzar diuresis.
D) Antibióticos orales y reposo domiciliario.
E) Antibióticos endovenosos intrahospitalarios.
80. Gestante de 30 semanas con aumento excesivo de peso
(ganancia de 18 kg. hasta la actualidad) a la que se realiza un test
de O’Sullivan que resulta patológico. La actitud médica ha de ser a
continuación:
A) Vigilancia fetal estricta, con registros semanales de la frecuencia
cardíaca fetal.
B) Controles de glucemia capilar (BMtest) en desayuno, comida y
cena.
C) Ecografías seriadas para diagnosticar a la mayor
brevedad posible un hidramnios o macrosomía fetal.
D) Confirmar el diagnóstico mediante una prueba de tolerancia
oral a la glucosa.
E) Tratar con insulina rápida según los resultados del
test de O’Sullivan.
81. Gestante de 9 semanas que consulta por metrorragia
menor que una regla y dolor abdominal. Todavía no ha acudido a
ninguna visita de control por su tocólogo. En la exploración se obseva
un útero de aproximadamente 8 semanas de gestación, abdomen
blando y depresible y cérvix permeable a un dedo. Poco después la
paciente empieza a sangrar abundantemente, mucho más que una
regla. El tratamiento indicado es:
A) Ingreso y observación. B)
Laparoscopia.
C) Legrado.
D) Laparotomía.
E) Venoclisis de oxitocina.
82. Primigesta de 27 años, sin antecedentes médicos de interés, que
consulta por metrorragia insidiosa y recurrente en semana 36 de
embarazo. No presenta dolor abdominal, el útero está relajado y el
latido fetal se escucha vigoroso.
El diagnóstico más probable será: A)
Vasa praevia.
B) Desprendimiento prematuro de placenta.
C) Expulsión del tapón mucoso.
D) Placenta previa.
E) Pérdida de líquido amniótico hemático.
83. Tercigesta isoinmunizada que presenta test de Coombs indirecto
de 1/10 en semana 30 de embarazo. Se realiza un amniocentesis en
que se determina la madurez fetal (se confirma la presencia de
fosfatidil glicerol en líquido amniótico) y el nomograma de Liley, que
se encuentra en la zona II. La actitud indicada es:
A) Nueva amniocentesis en una semana.
B) Seguimiento ecográfico con la paciente hospitalizada. C)
Administración de corticoides para inducir la madurez fetal.
D) Extracción fetal.
E) Administración endovenosa materna de IgG contra
Ac anti D.
84. Gestante de 33 semanas que consulta por dinámica uterina,
disminución de movimientos fetales y febrícula. Al ingreso presenta
una analítica de sangre con leucocitosis moderada y el resto de los
parámetros normales. En la
ecografìa se observa un oligoamnios y un perfil biofísico fetal de 9. El
tratamiento consiste en:
A) Antibióticos endovenosos. B)
Antibióticos intracavitarios. C)
Inducción del parto.
D) Cultivo del líquido amniótico y tratamiento según
antibiograma.
E) Administración de corticoides para favorecer la madurez fetal e
inducción del parto en semana 36.
85. Gestante de embarazo gemelar que presenta dinámica
espontánea en semana 36. Ambos gemelos se encuentran en
presentación cefálica. El parto del primero se produce sin
complicaciones, pero el segundo se encuentra en posición occipito
iliaca derecha transversa al cabo de 25 minutos desde el nacimiento
del primero, sin progresar desde un III plano de Hodge. Para finalizar
el parto está indicada la realización de:
A) Vacuum. B)
Fórceps. C)
Cesarea.
D) Maniobra de Kristeller.
E) Cualquiera de las anteriores según el estado fetal.
86. Primigesta de 36 años que presenta un aumento de la tensión
diastólica de 30 mmHg respecto a tomas iniciales, y albuminuria con
edemas generalizados. Actualmente se encuentra en la 34 semana
de embarazo. Súbitamente inicia un cuadro de cefalea y transtornos
visuales. Vd. le inicia tratamiento médico con:
A) Nifedipina.
B) Dihidralacina. C)
Alfametildopa.
D) Sulfato de magnesio. E)
Labetalol.
[22]
Mg. Dany Colca
Lic. Carolina Ayala
RESIDENTADO DE ENFERMERÍA. Compendio Banco de Preguntas.
87. En la paciente del caso anterior el estudio fetal descubre un
feto afecto de crecimiento intrauterino retardado, en el que la relación
entre el área cefálica y el área abdominal es mayor de uno. El feto
se encuentra en situación cefálica.
La conducta obstétrica adecuada es:
A) Expectante, con parto vaginal como via de elección.
B) Controles de bienestar fetal (perfil biofísico)
semanales hasta la semana 40.
C) Valoración de la madurez fetal e inducción del parto, si el feto es
maduro.
D) Cesárea inmediata.
E) Evaluación de la funcionalidad placentaria mediante ecografía
Doppler.
88. Puérpera que acaba de parir mediante parto eutócico un varón
de 3,450 kg. a los 30 minutos no ha alumbrado, por lo que se realiza
una maniobra de Credé para extraer la placenta. Transcurridos unos
minutos presenta un cuadro de disnea y hemorragia profusa. El útero
está bien contraído, pero el sangrado no cesa. El diagnóstico más
probable es:
A) Ruptura uterina.
B) Desgarro de cérvix. C)
Hipotonía uterina.
D) Retención de restos placentarios.
E) Coagulación intravascular diseminada.
89. Un paciente de 26 años de edad, adicto a drogas por vía
parenteral, consulta por malestar general, fiebre de 39°con tiritona,
escalofríos y dolor y tumefacción en rodilla derecha. En la exploración
llama la atención un soplo cardíco panfocal que previamente no
estaba en la historia del enfermo y artritis de rodilla derecha. No se
pudo realizar artrocentesis diagnóstica. La cobertura empírica
antibiótica más segura sería:
A) Vancomicina.
B) Vancomicina y Gentamicina. C)
Ciprofloxacina.
D) Eritromicina y cefuloxima. E)
Ceftriaxona.
90. Un joven de 18 años sin antecedentes de interés, consulta por
tos, fiebre y otalgia. En la Rx de tórax presenta infiltrado
intersticial derecho. La exploración ORL demuestra miringitis
ampollosa. Iniciaríamos tratamiento con:
A) Ceftriaxona. B)
Vancomicina. C)
Norfloxacina.
D) Isoniazida, rifampicina y pirazinamida. E)
Eritromicina.
91. Un paciente al que le detecta una infección urinaria por
Pseudomona inicia tratamiento con ceftacidima. Tras 2 días de
tratamiento el paciente comienza a encontrarse peor, la fiebre
aumenta hasta los 39° y desarrolla hipotensión. En el hemograma
destaca leucopenia e importante trombopenia que previamente
no estaban. Bioquímicamente presenta hiperglucemia de 198
g./dl. Probablementenos encontramos ante:
A) Efecto secundario de la ceftacidima. B)
Evolución natural del proceso.
C) Situación de shock séptico.
D) Probable asociación de una hemopatía. E)
Cetoacidosis diabética.
92. Un varón de 27 años consulta por presenta en la región
balanoprepucial una úlcera de 1 cm. de tamaño de borde indurado,
no doloroso y que secreta serosidad. Presenta adicionalmente
adenopatías inguinales bilaterales y fiebre. Estaríamos obligados a
solicitarle en el estudio:
A) Aglutinaciones a salmonella. B)
Aglutinaciones a brucella.
C) Serología de VIH. D)
Serología luética.
E) Serología de micoplasma.
93. Una paciente de 19 años ha sido diagnosticada de mononucleosis
infecciosa, confirmada mediante Paul- Bunnel. Ha sido tratada con
paracetamol a dosis de 2,5 g./día. A los cinco días del diagnóstico
comienza de forma brusca con un cuadro de abdomen agudo y
shock. El proceso más probable es:
A) Rotura espontánea del bazo.
B) Hepatitis por virus de Epstein-Barr.
C) Complicación infecciosa intraabdominal. D)
Complicación del tratamiento.
E) Otra cualquier causa de abdomen agudo: p. ej. apendicitis
aguda.
94. Un lactante de 7 meses de edad, con Tetralogía de Fallot
intervenida, desarrolla un cuadro de dificultad respiratoria, sibilancias
y tos. No se termometra fiebre. Además de las medidas de soporte,
el tratamiento de elección sería:
A) Reintervención quirúrgica de su cardiopatía. B)
Esteroides a dosis plenas.
C) Ribavirina en aerosol.
D) Eritromicina intravenosa. E)
Ceftriaxona intravenosa.
95. Una paciente de 49 años consulta en un Servicio de Urgencias
por dolor, enrojecimiento y tumefacción de la parte distal de su
miembro inferior derecho. Por sospecha de trombosis venosa
profunda se le realiza flebografía que resulta ser negativa. La actitud
terapéutica a seguir es:
A) Tratamiento antiinflamatorio.
B) Heparina de bajo peso molecular.
C) Cobertura empírica antibiótica con Oxacilina. D)
Reposo del miembro sin más.
E) Repetir flebografía pasadas 48 horas.
[23]
Mg. Dany Colca
Lic. Carolina Ayala
RESIDENTADO DE ENFERMERÍA. Compendio Banco de Preguntas.
96. Un adicto a drogas por vía parenteral consulta por tumoración
fluctuante y con signos inflamatorios en la flexura del codo donde ha
realizado inyecciones intravenosas. Se debe realizar:
A) Tratamiento quirúrgico de drenaje. B)
Cobertura con ceftriaxona.
C) Cobertura con vancomicina. D)
Cobertura con Oxacilina.
E) Medidas antiinflamatorias.
97. Una pareja de turistas que han regresado a Lima, luego de estar
en la Selva Central del Perú, comienzan a presentar deposiciones en
cantidad abundante de características líquidas, similares al agua, en
número de
20-25 al día. No se acompaña de dolor abdominal ni fiebre. El cuadro
obliga a una rehidratación y ésta se consigue con dificultad dada la
gran pérdida de agua y electrólitos. El cuadro clínico más probable es:
A) Gastroenteritis por Salmonella. B)
Disentería bacilar.
C) Disentería amebiana. D)
Cólera.
E) Gastroenteritis viral.
98. Un enfermo leucémico, muy inmunodeprimido, desarrolla tras un
tratamiento antibacteriano de amplio espectro, cuadro importante de
insuficiencia respiratoria y fiebre. En la Rx. de tórax se aprecia una
masa densa, cubierta por un menisco delgado de aire en el interior de
una cavidad. El diagnóstico más probable es:
A) Tuberculosis.
B) Neumonía bacteriana.
C) Masa tumoral sobreinfectada. D)
Neumonía por cándidas.
E) Aspergiloma.
99. Un niño de 3 años desarrolla un cuadro de infección respiratoria
de vías altas con fiebre elevada. Posteriormente aparece tos
paroxística con gallo inspiratorio. El tratamiento de elección sería:
A) Ceftriaxona. B)
Cefonicid.
C) Cefalotina. D)
Eritromicina.
E) Amoxicilina-clavulánico.
100. Un paciente de 56 años acude a su médico de cabecera por
presentar en el último mes fiebre diaria, con una distribución de dos
picos, matutino y vespertino. A la exploración llama la atención una
hepatoesplenomegalia muy importante. No se objetivan adenopatías
a ningún nivel. El hemograma muestra pancitopenia. La prueba
diagnóstica que se debería realizar sería:
A) Estudio de médula ósea. B)
Ecografía abdominal.
C) Marcadores tumorales. D)
Biopsia hepática.
E) Marcadores de hepatitis.
ENAE Examen Nacional de Enfermería compendio banco preguntas
ENAE Examen Nacional de Enfermería compendio banco preguntas
ENAE Examen Nacional de Enfermería compendio banco preguntas
ENAE Examen Nacional de Enfermería compendio banco preguntas
ENAE Examen Nacional de Enfermería compendio banco preguntas
ENAE Examen Nacional de Enfermería compendio banco preguntas
ENAE Examen Nacional de Enfermería compendio banco preguntas
ENAE Examen Nacional de Enfermería compendio banco preguntas
ENAE Examen Nacional de Enfermería compendio banco preguntas
ENAE Examen Nacional de Enfermería compendio banco preguntas
ENAE Examen Nacional de Enfermería compendio banco preguntas
ENAE Examen Nacional de Enfermería compendio banco preguntas
ENAE Examen Nacional de Enfermería compendio banco preguntas
ENAE Examen Nacional de Enfermería compendio banco preguntas
ENAE Examen Nacional de Enfermería compendio banco preguntas
ENAE Examen Nacional de Enfermería compendio banco preguntas
ENAE Examen Nacional de Enfermería compendio banco preguntas
ENAE Examen Nacional de Enfermería compendio banco preguntas
ENAE Examen Nacional de Enfermería compendio banco preguntas
ENAE Examen Nacional de Enfermería compendio banco preguntas
ENAE Examen Nacional de Enfermería compendio banco preguntas
ENAE Examen Nacional de Enfermería compendio banco preguntas
ENAE Examen Nacional de Enfermería compendio banco preguntas
ENAE Examen Nacional de Enfermería compendio banco preguntas
ENAE Examen Nacional de Enfermería compendio banco preguntas
ENAE Examen Nacional de Enfermería compendio banco preguntas

Más contenido relacionado

La actualidad más candente

Pae de ascitis
Pae de ascitisPae de ascitis
Pae de ascitisSu Vega
 
Modelo cuidados enfermero programas madre niño - CICAT-SALUD
Modelo cuidados enfermero programas madre niño - CICAT-SALUDModelo cuidados enfermero programas madre niño - CICAT-SALUD
Modelo cuidados enfermero programas madre niño - CICAT-SALUDCICAT SALUD
 
Plan de atención de enfermería - Deterioro del intercambio gaseoso (NIC - NOC)
Plan de atención de enfermería - Deterioro del intercambio gaseoso (NIC - NOC)Plan de atención de enfermería - Deterioro del intercambio gaseoso (NIC - NOC)
Plan de atención de enfermería - Deterioro del intercambio gaseoso (NIC - NOC)angiemandy
 
Proceso de enfermería en paciente con encefalopatía hepática en el servicio d...
Proceso de enfermería en paciente con encefalopatía hepática en el servicio d...Proceso de enfermería en paciente con encefalopatía hepática en el servicio d...
Proceso de enfermería en paciente con encefalopatía hepática en el servicio d...Alejandra Centeno
 
Oxigenoterapia alto y bajo flujo
Oxigenoterapia alto y bajo flujoOxigenoterapia alto y bajo flujo
Oxigenoterapia alto y bajo flujoTernura Garcia
 
Asistencia de Enfermería en los Trastornos Acido-básicos
Asistencia de Enfermería en los Trastornos Acido-básicosAsistencia de Enfermería en los Trastornos Acido-básicos
Asistencia de Enfermería en los Trastornos Acido-básicosSergio Enrique Castillo Vega
 
AUTOCUIDADO DE DOROTHEA OREM
AUTOCUIDADO DE DOROTHEA OREMAUTOCUIDADO DE DOROTHEA OREM
AUTOCUIDADO DE DOROTHEA OREMModelos09
 
Análisis del diagnóstico riesgo de deterioro de la integridad cutánea en la p...
Análisis del diagnóstico riesgo de deterioro de la integridad cutánea en la p...Análisis del diagnóstico riesgo de deterioro de la integridad cutánea en la p...
Análisis del diagnóstico riesgo de deterioro de la integridad cutánea en la p...GNEAUPP.
 
Estudio de caso anemia
Estudio de caso anemiaEstudio de caso anemia
Estudio de caso anemialuis carvajal
 
Proceso de atención en enfermería en paciente renal
Proceso de atención en enfermería en paciente renalProceso de atención en enfermería en paciente renal
Proceso de atención en enfermería en paciente renalnatorabet
 
Proceso diagnostico de enfermería
Proceso diagnostico de enfermeríaProceso diagnostico de enfermería
Proceso diagnostico de enfermeríaGRUPO8UCV
 
Ley de consultorios de enfermeria 1
Ley  de consultorios de enfermeria 1Ley  de consultorios de enfermeria 1
Ley de consultorios de enfermeria 1ELUYOV
 
PAE: Úlceras por presión
PAE: Úlceras por presiónPAE: Úlceras por presión
PAE: Úlceras por presiónBerire94
 

La actualidad más candente (20)

(2021 06-24) ESCALAS DE VALORACION (DOC)
(2021 06-24) ESCALAS DE VALORACION (DOC)(2021 06-24) ESCALAS DE VALORACION (DOC)
(2021 06-24) ESCALAS DE VALORACION (DOC)
 
Pae de ascitis
Pae de ascitisPae de ascitis
Pae de ascitis
 
Modelo cuidados enfermero programas madre niño - CICAT-SALUD
Modelo cuidados enfermero programas madre niño - CICAT-SALUDModelo cuidados enfermero programas madre niño - CICAT-SALUD
Modelo cuidados enfermero programas madre niño - CICAT-SALUD
 
Listtooo pae
Listtooo paeListtooo pae
Listtooo pae
 
Plan de atención de enfermería - Deterioro del intercambio gaseoso (NIC - NOC)
Plan de atención de enfermería - Deterioro del intercambio gaseoso (NIC - NOC)Plan de atención de enfermería - Deterioro del intercambio gaseoso (NIC - NOC)
Plan de atención de enfermería - Deterioro del intercambio gaseoso (NIC - NOC)
 
Proceso de enfermería en paciente con encefalopatía hepática en el servicio d...
Proceso de enfermería en paciente con encefalopatía hepática en el servicio d...Proceso de enfermería en paciente con encefalopatía hepática en el servicio d...
Proceso de enfermería en paciente con encefalopatía hepática en el servicio d...
 
Oxigenoterapia alto y bajo flujo
Oxigenoterapia alto y bajo flujoOxigenoterapia alto y bajo flujo
Oxigenoterapia alto y bajo flujo
 
Asistencia de Enfermería en los Trastornos Acido-básicos
Asistencia de Enfermería en los Trastornos Acido-básicosAsistencia de Enfermería en los Trastornos Acido-básicos
Asistencia de Enfermería en los Trastornos Acido-básicos
 
AUTOCUIDADO DE DOROTHEA OREM
AUTOCUIDADO DE DOROTHEA OREMAUTOCUIDADO DE DOROTHEA OREM
AUTOCUIDADO DE DOROTHEA OREM
 
Análisis del diagnóstico riesgo de deterioro de la integridad cutánea en la p...
Análisis del diagnóstico riesgo de deterioro de la integridad cutánea en la p...Análisis del diagnóstico riesgo de deterioro de la integridad cutánea en la p...
Análisis del diagnóstico riesgo de deterioro de la integridad cutánea en la p...
 
PAE EPOC
PAE EPOC  PAE EPOC
PAE EPOC
 
Introduccion unidad de paciente critico
Introduccion  unidad de paciente criticoIntroduccion  unidad de paciente critico
Introduccion unidad de paciente critico
 
Facultad de enfermeria norma
Facultad de enfermeria normaFacultad de enfermeria norma
Facultad de enfermeria norma
 
PAE Hipertensión arterial
PAE Hipertensión arterialPAE Hipertensión arterial
PAE Hipertensión arterial
 
PAE ANEMIA CASO CLINICO
PAE ANEMIA CASO CLINICO PAE ANEMIA CASO CLINICO
PAE ANEMIA CASO CLINICO
 
Estudio de caso anemia
Estudio de caso anemiaEstudio de caso anemia
Estudio de caso anemia
 
Proceso de atención en enfermería en paciente renal
Proceso de atención en enfermería en paciente renalProceso de atención en enfermería en paciente renal
Proceso de atención en enfermería en paciente renal
 
Proceso diagnostico de enfermería
Proceso diagnostico de enfermeríaProceso diagnostico de enfermería
Proceso diagnostico de enfermería
 
Ley de consultorios de enfermeria 1
Ley  de consultorios de enfermeria 1Ley  de consultorios de enfermeria 1
Ley de consultorios de enfermeria 1
 
PAE: Úlceras por presión
PAE: Úlceras por presiónPAE: Úlceras por presión
PAE: Úlceras por presión
 

Destacado (8)

CALIENTITAS ENAE 2015
CALIENTITAS ENAE 2015CALIENTITAS ENAE 2015
CALIENTITAS ENAE 2015
 
ENAE 2015
ENAE 2015ENAE 2015
ENAE 2015
 
Banco de preguntas protesis fija
Banco de preguntas protesis fijaBanco de preguntas protesis fija
Banco de preguntas protesis fija
 
COMBO ENAE
COMBO ENAECOMBO ENAE
COMBO ENAE
 
Temario Desarrollado ENAO
Temario Desarrollado ENAOTemario Desarrollado ENAO
Temario Desarrollado ENAO
 
Etapasdelparto 110329115534-phpapp02
Etapasdelparto 110329115534-phpapp02Etapasdelparto 110329115534-phpapp02
Etapasdelparto 110329115534-phpapp02
 
Enao Odontologia
Enao OdontologiaEnao Odontologia
Enao Odontologia
 
Banco de preguntas_adulto[1]
Banco de preguntas_adulto[1]Banco de preguntas_adulto[1]
Banco de preguntas_adulto[1]
 

Similar a ENAE Examen Nacional de Enfermería compendio banco preguntas

Examen_Simulacro_Tema_A_y_B_Sabado_4_de.pdf
Examen_Simulacro_Tema_A_y_B_Sabado_4_de.pdfExamen_Simulacro_Tema_A_y_B_Sabado_4_de.pdf
Examen_Simulacro_Tema_A_y_B_Sabado_4_de.pdfIsusKonIntiAi
 
Rm19 1 v simulacro 5 parte b sin claves
Rm19 1 v simulacro 5  parte b sin clavesRm19 1 v simulacro 5  parte b sin claves
Rm19 1 v simulacro 5 parte b sin clavesLUISFUERTESV
 
Parte 1 de 3 essalud 2017
Parte 1 de 3 essalud 2017Parte 1 de 3 essalud 2017
Parte 1 de 3 essalud 2017Bruno Alcalde
 
Caso clinico para entregar
Caso clinico para entregarCaso clinico para entregar
Caso clinico para entregarJdso Otero
 
129956794 examen-essalud-moyo07
129956794 examen-essalud-moyo07129956794 examen-essalud-moyo07
129956794 examen-essalud-moyo07xixel britos
 
100615 final-examen-parte-a-estudios myc
100615 final-examen-parte-a-estudios myc100615 final-examen-parte-a-estudios myc
100615 final-examen-parte-a-estudios mycehpt
 
ENAM 2021 - 20 DE MARZO.pdf
ENAM 2021 - 20 DE MARZO.pdfENAM 2021 - 20 DE MARZO.pdf
ENAM 2021 - 20 DE MARZO.pdfEdward leyva
 
C. DIFFICILE.pptx
C. DIFFICILE.pptxC. DIFFICILE.pptx
C. DIFFICILE.pptxAnaAranda42
 
30788305 preguntas-medicina-interna
30788305 preguntas-medicina-interna30788305 preguntas-medicina-interna
30788305 preguntas-medicina-internaxixel britos
 
Preguntas y respuestas medicina interna
Preguntas y respuestas medicina internaPreguntas y respuestas medicina interna
Preguntas y respuestas medicina internaddcl
 

Similar a ENAE Examen Nacional de Enfermería compendio banco preguntas (20)

Examen_Simulacro_Tema_A_y_B_Sabado_4_de.pdf
Examen_Simulacro_Tema_A_y_B_Sabado_4_de.pdfExamen_Simulacro_Tema_A_y_B_Sabado_4_de.pdf
Examen_Simulacro_Tema_A_y_B_Sabado_4_de.pdf
 
Rm19 1 v simulacro 5 parte b sin claves
Rm19 1 v simulacro 5  parte b sin clavesRm19 1 v simulacro 5  parte b sin claves
Rm19 1 v simulacro 5 parte b sin claves
 
Simulacro 21.pdf
Simulacro 21.pdfSimulacro 21.pdf
Simulacro 21.pdf
 
Parte 1 de 3 essalud 2017
Parte 1 de 3 essalud 2017Parte 1 de 3 essalud 2017
Parte 1 de 3 essalud 2017
 
2do simulacrorm 21-tema a
2do simulacrorm 21-tema a2do simulacrorm 21-tema a
2do simulacrorm 21-tema a
 
Examen 1 de_la_salle
Examen 1 de_la_salleExamen 1 de_la_salle
Examen 1 de_la_salle
 
Caso clinico para entregar
Caso clinico para entregarCaso clinico para entregar
Caso clinico para entregar
 
Enfermería SES 2008 - Supuesto 1
Enfermería SES 2008 - Supuesto 1Enfermería SES 2008 - Supuesto 1
Enfermería SES 2008 - Supuesto 1
 
Simulacro 1.pdf
Simulacro 1.pdfSimulacro 1.pdf
Simulacro 1.pdf
 
30705227 medicina-general
30705227 medicina-general30705227 medicina-general
30705227 medicina-general
 
Multiple choice
Multiple choiceMultiple choice
Multiple choice
 
129956794 examen-essalud-moyo07
129956794 examen-essalud-moyo07129956794 examen-essalud-moyo07
129956794 examen-essalud-moyo07
 
100615 final-examen-parte-a-estudios myc
100615 final-examen-parte-a-estudios myc100615 final-examen-parte-a-estudios myc
100615 final-examen-parte-a-estudios myc
 
Examen RM 2014B
Examen RM 2014BExamen RM 2014B
Examen RM 2014B
 
ENAM 2021 - 20 DE MARZO.pdf
ENAM 2021 - 20 DE MARZO.pdfENAM 2021 - 20 DE MARZO.pdf
ENAM 2021 - 20 DE MARZO.pdf
 
EVALUACIÓN DIAGNÓSTICA.pdf
EVALUACIÓN DIAGNÓSTICA.pdfEVALUACIÓN DIAGNÓSTICA.pdf
EVALUACIÓN DIAGNÓSTICA.pdf
 
C. DIFFICILE.pptx
C. DIFFICILE.pptxC. DIFFICILE.pptx
C. DIFFICILE.pptx
 
30788305 preguntas-medicina-interna
30788305 preguntas-medicina-interna30788305 preguntas-medicina-interna
30788305 preguntas-medicina-interna
 
Microbiología
MicrobiologíaMicrobiología
Microbiología
 
Preguntas y respuestas medicina interna
Preguntas y respuestas medicina internaPreguntas y respuestas medicina interna
Preguntas y respuestas medicina interna
 

Último

ADITAMENTOS PROTESIS REMOVIBLE DENTALES.
ADITAMENTOS PROTESIS REMOVIBLE DENTALES.ADITAMENTOS PROTESIS REMOVIBLE DENTALES.
ADITAMENTOS PROTESIS REMOVIBLE DENTALES.EstefaniRomeroGarcia
 
DICTAMEN PERITAJE SICOLOGICO PERITO..pdf
DICTAMEN PERITAJE SICOLOGICO PERITO..pdfDICTAMEN PERITAJE SICOLOGICO PERITO..pdf
DICTAMEN PERITAJE SICOLOGICO PERITO..pdfkixasam181
 
ETAPA DOMESTICA DEL CUIDADO(HISTORIA DE LA ENFERMERIA)
ETAPA DOMESTICA DEL CUIDADO(HISTORIA DE LA ENFERMERIA)ETAPA DOMESTICA DEL CUIDADO(HISTORIA DE LA ENFERMERIA)
ETAPA DOMESTICA DEL CUIDADO(HISTORIA DE LA ENFERMERIA)David762496
 
ePóster. Reducciones c-LDL el primer año de tratamiento con inclisirán
ePóster. Reducciones c-LDL el primer año de tratamiento con inclisiránePóster. Reducciones c-LDL el primer año de tratamiento con inclisirán
ePóster. Reducciones c-LDL el primer año de tratamiento con inclisiránSociedad Española de Cardiología
 
Conferencia acerca del sistema nervioso autónomo
Conferencia acerca del sistema nervioso autónomoConferencia acerca del sistema nervioso autónomo
Conferencia acerca del sistema nervioso autónomoosvaldomorfagutierre
 
Clase 7 Torax, Costillas y Esternon Osteologia 2024.pdf
Clase 7 Torax, Costillas y Esternon Osteologia 2024.pdfClase 7 Torax, Costillas y Esternon Osteologia 2024.pdf
Clase 7 Torax, Costillas y Esternon Osteologia 2024.pdfgarrotamara01
 
terminologia NIÑO Y ADOLESCENTE Y SAKUD.pptx
terminologia NIÑO Y ADOLESCENTE Y SAKUD.pptxterminologia NIÑO Y ADOLESCENTE Y SAKUD.pptx
terminologia NIÑO Y ADOLESCENTE Y SAKUD.pptxrosi339302
 
Paludismo o Malaria- Medicina tropical.pptx
Paludismo o Malaria- Medicina tropical.pptxPaludismo o Malaria- Medicina tropical.pptx
Paludismo o Malaria- Medicina tropical.pptx Estefania Recalde Mejia
 

Último (20)

ADITAMENTOS PROTESIS REMOVIBLE DENTALES.
ADITAMENTOS PROTESIS REMOVIBLE DENTALES.ADITAMENTOS PROTESIS REMOVIBLE DENTALES.
ADITAMENTOS PROTESIS REMOVIBLE DENTALES.
 
(2024-04-10) PACIENTE POLIMEDICADO (doc).pdf
(2024-04-10) PACIENTE POLIMEDICADO (doc).pdf(2024-04-10) PACIENTE POLIMEDICADO (doc).pdf
(2024-04-10) PACIENTE POLIMEDICADO (doc).pdf
 
(2024-04-10) PACIENTE POLIMEDICADO (ppt).pdf
(2024-04-10) PACIENTE POLIMEDICADO (ppt).pdf(2024-04-10) PACIENTE POLIMEDICADO (ppt).pdf
(2024-04-10) PACIENTE POLIMEDICADO (ppt).pdf
 
DICTAMEN PERITAJE SICOLOGICO PERITO..pdf
DICTAMEN PERITAJE SICOLOGICO PERITO..pdfDICTAMEN PERITAJE SICOLOGICO PERITO..pdf
DICTAMEN PERITAJE SICOLOGICO PERITO..pdf
 
Estudio KARDIA-2
Estudio KARDIA-2Estudio KARDIA-2
Estudio KARDIA-2
 
(2024-04-10) TÉCNICA ROVIRALTA (ppt).pdf
(2024-04-10) TÉCNICA ROVIRALTA (ppt).pdf(2024-04-10) TÉCNICA ROVIRALTA (ppt).pdf
(2024-04-10) TÉCNICA ROVIRALTA (ppt).pdf
 
(2024-11-04) Patologia anorectal (ptt).pptx
(2024-11-04) Patologia anorectal (ptt).pptx(2024-11-04) Patologia anorectal (ptt).pptx
(2024-11-04) Patologia anorectal (ptt).pptx
 
ETAPA DOMESTICA DEL CUIDADO(HISTORIA DE LA ENFERMERIA)
ETAPA DOMESTICA DEL CUIDADO(HISTORIA DE LA ENFERMERIA)ETAPA DOMESTICA DEL CUIDADO(HISTORIA DE LA ENFERMERIA)
ETAPA DOMESTICA DEL CUIDADO(HISTORIA DE LA ENFERMERIA)
 
Estudio ORBITA-COSMIC
Estudio ORBITA-COSMICEstudio ORBITA-COSMIC
Estudio ORBITA-COSMIC
 
Estudio MINT
Estudio MINTEstudio MINT
Estudio MINT
 
Estudio ULTIMATE DAPT
Estudio ULTIMATE DAPTEstudio ULTIMATE DAPT
Estudio ULTIMATE DAPT
 
Estudio SMART
Estudio SMARTEstudio SMART
Estudio SMART
 
Estudio DANGER
Estudio DANGEREstudio DANGER
Estudio DANGER
 
ePóster. Reducciones c-LDL el primer año de tratamiento con inclisirán
ePóster. Reducciones c-LDL el primer año de tratamiento con inclisiránePóster. Reducciones c-LDL el primer año de tratamiento con inclisirán
ePóster. Reducciones c-LDL el primer año de tratamiento con inclisirán
 
Estudio DEDICATE-DZHK6
Estudio DEDICATE-DZHK6Estudio DEDICATE-DZHK6
Estudio DEDICATE-DZHK6
 
Estudio PREVENT
Estudio PREVENTEstudio PREVENT
Estudio PREVENT
 
Conferencia acerca del sistema nervioso autónomo
Conferencia acerca del sistema nervioso autónomoConferencia acerca del sistema nervioso autónomo
Conferencia acerca del sistema nervioso autónomo
 
Clase 7 Torax, Costillas y Esternon Osteologia 2024.pdf
Clase 7 Torax, Costillas y Esternon Osteologia 2024.pdfClase 7 Torax, Costillas y Esternon Osteologia 2024.pdf
Clase 7 Torax, Costillas y Esternon Osteologia 2024.pdf
 
terminologia NIÑO Y ADOLESCENTE Y SAKUD.pptx
terminologia NIÑO Y ADOLESCENTE Y SAKUD.pptxterminologia NIÑO Y ADOLESCENTE Y SAKUD.pptx
terminologia NIÑO Y ADOLESCENTE Y SAKUD.pptx
 
Paludismo o Malaria- Medicina tropical.pptx
Paludismo o Malaria- Medicina tropical.pptxPaludismo o Malaria- Medicina tropical.pptx
Paludismo o Malaria- Medicina tropical.pptx
 

ENAE Examen Nacional de Enfermería compendio banco preguntas

  • 1. ENAEExamen Nacional de Enfermería 2017 Pedidos: Cel. :RPC. 969340751 Cel. :RPC. 941105128 Correo: publicacionesdc@hotmail.com COMPENDIO DE BANCOS DE PREGUNTAS 6ta Edición
  • 2. [1] EXAMEN NACIONAL DE ENFERMERÍA. Compendio Banco de Preguntas. Mg. Dany Colca Lic. Carolina Ayala 1. Hombre de 48 años que acude a Urgencias por dolor abdominal y vómitos. Los datos analíticos iniciales son: GOT: 80 U/I. GTP 54 U/I. Leucocitos 21.800/mm3. Amilasa 4.500 U/I. La TAC abdominal demuestra colección peripancreática. Se inicia tratamiento con fluidoterapia y analgésicos. Señale cuál de los siguientes fármacos añadiría al tratamiento, en primer lugar, para mejorar el pronóstico de este caso: A) Metilprednisolona. B) Antiinflamatorios no esteroideos. C) D) Imipenem. E) Inhibidores de la secreción pancreática. 2. ¿Cuál es el principal factor responsable de la no cicatrización de una úlcera péptica?: A) Estrés. B) Infección por Helicobacter pylori. C) Consumo de alcohol. D) No abandono del hábito tabáquico. E) Determinados hábitos dietéticos. 3. ¿Cuál de las siguientes recomendaciones es FALSA en relación con el tratamiento de la peritonitis bacteriana espontánea en un paciente cirrótico?: A) El diagnóstico se basa en la existencia de más de 250 PMN/mm3 en el líquido ascítico. B) El tratamiento de elección lo constituyen las cefalosporinas de tercera generación. C) La administración de albúmina intravenosa previene el desarrollo de insuficiencia renal. D) El tratamiento antibiótico debe mantenerse durante 15 días. E) Una vez resuelta la infección, debe iniciarse tratamiento profiláctico con norfloxacino. 4. Paciente de 72 años, que como único tratamiento toma antidiabéticos orales, presenta anemia ferropénica crónica, con hemorragias ocultas positivas. ¿Cual es el método diagnóstico más indicado para localizar la lesión sangrante?: A) Tránsito gastroduodenal. B) Gammagrafía con hematíes marcados. C) Tránsito intestinal. D) Colonoscopia total. E) Panendoscopia oral. 5. ¿Cuál de las siguientes definiciones corresponde con mayor precisión al concepto de Metaanálisis?: A) Es una revisión narrativa en la que la bibliografía se busca de manera no estructurada. B) Es una revisión narrativa en la que la bibliografía se busca de manera estructurada. C) Es una revisión narrativa en la que se presentan tablas estructuradas de los resultados de los distintos estudios incluidos. D) Es una revisión en la que se combinan estadísticamente los resultados de los estudios incluidos. BANCO N°001
  • 3. [2] Mg. Dany Colca Lic. Carolina Ayala RESIDENTADO DE ENFERMERÍA. Compendio Banco de Preguntas. E) Es una revisión en las que se presentan tablas estructuradas de los resultados de los distintos estudios incluidos. 6. Le encargan el diseño de un ensayo clínico en el que es muy importante que un factor pronóstico se distribuya por igual en los dos grupos de tratamiento. El método de aleatorización que usted utilizaría es: A) Aleatorización simple. B) Aleatorización por bloques. C) Aleatorización estratificada. D) Aleatorización centralizada. E) Aleatorización ciega (ocultación de la secuencia de aleatorización). 7. En una revista biomédica se publica un estudio, en el que los autores notifican el resultado en coste/años de vida ganados. ¿De qué tipo de análisis de evaluación económica se trata?: A) Coste de la enfermedad. B) Coste-efectividad. C) Coste-beneficio. D) Coste-consecuencia. E) Coste-utilidad. 8. La mejor forma de verificar una hipótesis en epidemiología es a través de: A) Un estudio descriptivo. B) Un estudio experimental. C) Un estudio caso-control. D) Un estudio de cohortes. E) Un estudio transversal. 9. Todas estas situaciones, EXCEPTO una, aumentan la probabilidad de detectar un cáncer colorrectal, señálela: A) Enfermedad inflamatoria intestinal de larga evolución. B) Endocarditis por Streptococcus bovis. C) Tabaquismo de más de 35 años de duración. D) Ureterosigmoidostomía hace 20 años, para corregir una malformación vesical. E) Ingesta crónica de aspirina o antiinflamatorios no esteroideos. 10. ¿Cuál de las siguientes características NO es propia de un ensayo clínico controlado?: A) Intervención experimental. B) Tratamiento asignado según el criterio del investigador. C) Participación voluntaria del sujeto. D) Asignación aleatoria a las distintas posibilidades de tratamiento en estudio. E) Enmascaramiento de la medicación. 11. La vacuna antineumocócica está recomendada para todas, MENOS una de las siguientes situaciones clínicas, señálela: A) Alcoholismo crónico. B) Contactos familiares de un paciente con neumonía neumocócica C) Fístula crónica de líquido cefalorraquídeo tras un traumatismo craneal. D) Infección VIH avanzada. E) Insuficiencia cardíaca crónica. 12. ¿Cuál de la siguientes vacunas no se incluye en las recomendaciones actuales de un paciente esplenectomizado?: A) Vacuna neumocócica. B) Vacuna meningocócica. C) Vacuna frente a Haemophilus influenzae tipo B. D) Vacuna frente al virus de la gripe. E) Vacuna frente al virus hepatitis A. 13. ¿En qué patología pensaría en primer lugar en un paciente de 65 años, que presenta disminución lenta, progresiva e indolora de su agudeza visual sin signos de inflamación ocular?: A) Error de refracción. B) Distrofia corneal. C) Papilitis. D) Glaucoma agudo. E) Degeneración macular senil. 14. Un paciente diabético tratado mediante fotocoagulación focal con láser de Argón tres años antes, presenta una pérdida brusca e importante de visión, sin dolor ni alteraciones en la superficie ocular. La causa más probable de esta disminución de agudeza visual es: A) Hemorragia vítrea. B) Edema corneal. C) Glaucoma crónico simple. D) Catarata nuclear. E) Atrofia óptica. 15. Una de los siguientes signos o síntomas, NO es habitual en una uveítis anterior aguda: A) Midriasis. B) Dolor. C) Inyección ciliar. D) Sinequias posteriores. E) Fotofobia. 16. ¿Cuál es el tratamiento de la obstrucción intestinal no quirúrgica en la fase terminal del cáncer?: A) Sonda nasogástrica, aspiración continua, sueroterapia intravenosa. B) Administración de vitamina grupo B, laxante y enema de limpieza. C) Alimentación parenteral total y esteroides i.v. D) Morfina, buscapina y haloperidol por vía subcutánea. E) Dieta absoluta y sueros por vía subcutánea. 17. ¿Cuál de los siguientes opioides NO es adecuado para el tratamiento del dolor crónico de etiología cancerosa?: A) Metadona. B) Meperidina. C) Morfina.
  • 4. [3] Mg. Dany Colca Lic. Carolina Ayala RESIDENTADO DE ENFERMERÍA. Compendio Banco de Preguntas. D) Fentanilo. E) Tramadol. 18. La localización ideal para hacer una traqueotomía es: A) La membrana crico-tiroidea. B) La membrana mio-tiroidea. C) Primer anillo traqueal. D) Segundo o tercer anillo traqueal. E) Cuarto o quinto anillo traqueal. 19. ¿Cuál de estas afirmaciones es FALSA en relación con la otitis media secretoria?: A) Los niños con paladar hendido son más susceptibles a padecerla. B) En casos unilaterales en adultos, es obligada la inspección del cavum. C) Su período de mayor incidencia en climas templados, es de junio a septiembre, coincidiendo con la época de baños. D) El 66% de los cultivos de las muestras obtenidas por miringocentesis presentan bacterias. E) La otoscopia puede mostrar niveles líquidos o burbujas de aire tras el tímpano íntegro. 20. Una mujer de 68 años, sin aparentes factores de riesgo cardiovascular, ingresa en la Unidad Coronaria del Hospital por un cuadro ¿agudo de cardiopatía isquémica. En la analítica realizada a su llegada se objetiva una anemia (Hb 8gr/dl) previamente no conocida. En este caso, la actitud más adecuada con respecto a la anemia es: A) Actitud expectante, ya que sólo se debe transfundir la anemia sintomática. B) Transfundir hematíes. C) Transfundir sangre total. D) Instaurar tratamiento con eritropoyetina. E) Administrar hierro intravenoso. 21. Los cuerpos de Howell-Jolly, son inclusiones eritrocitarias de fragmentos nucleares y se observan en: A) Asplenia. B) Mielofibrosis. C) Leucemia linfática crónica. D) Déficit de G6PD. E) Intoxicación por plomo. 22. ¿Cuál de los siguientes hallazgos es el más característico de la leucemia mieloide crónica?: A) El cromosoma Filadelfia. B) El esplenomegalia palpable. C) La disminución de la fosfatasa alcalina granulocitaria. D) El aumento del ácido úrico sérico. E) El reordenamiento del gen bcr/abl. 23. Joven de 23 años, con historia de conducta sexual de riesgo, consulta por fiebre, malestar general y dolor en glande. La exploración física muestra múltiples tatuajes, "piercings" y la presencia de tres lesiones vesiculosas en glande y adenopatías inguinales bilaterales. ¿Cuál es el diagnóstico más probable?: A) Herpes genital. B) Secundarismo luético. C) Primoinfección VIH. D) Infección por citomegalovirus. E) Condilomas acuminados. 24. En cuanto a la neumonía por Neumococo en paciente infectados por el virus de la inmunodeficiencia humana (VIH) es FALSO que: A) Es una infección frecuentemente bacteriémica. B) Tiene mayor incidencia que en la población general. C) Junto a Haemophilus influenzae es la causa más común de neumonía en pacientes con SIDA. D) Se recomienda vacuna neumocócica en aquellos pacientes con CD4 < 100 cel/ L. E) Esta neumonía puede ser vista en pacientes con sistema inmune relativamente intacto. 25. La tuberculosis asociada a la infección por VIH se caracteriza por: A) Presentación subclínica de la enfermedad. B) Aparición característica en los estadios de inmunodepresión más severa (>50 CD4/mm3). C) Elevada frecuencia de afectación extrapulmonar y diseminada. D) Escaso rendimiento de los métodos microbiológicos de diagnóstico. E) Mala respuesta al tratamiento antituberculoso. 26. NO es propio del shock tóxico estafilocócico: A) Fiebre elevada. B) Lesiones cutáneas. C) Metástasis sépticas. D) Fracaso renal. E) Rabdomiólisis. 27. ¿Cuál de las siguientes permite diferenciar la pielonefritis aguda de la cistitis aguda?: A) Leucocituria. B) Hematuria. C) Bacteriuria. D) Antecedentes de infección urinaria. E) Fiebre de más de 38.5ºC. 28. Un paciente de 22 años, que acude para evaluación de una lesión genital ulcerada, presenta un VDRL positivo a título de 8 diluciones con un FTA-Abs negativo. La interpretación más adecuada de estos resultados es: A) Falso positivo de las pruebas no treponémicas. B) Falso negativo de las pruebas treponémicas. C) Sífilis curada (pendiente la positivización del FTA- Abs). D) Sífilis curada (pendiente la negativización del VDRL). E) Sífilis de larga evolución.
  • 5. [4] Mg. Dany Colca Lic. Carolina Ayala RESIDENTADO DE ENFERMERÍA. Compendio Banco de Preguntas. 29. Señale cuál de las siguientes asociaciones de helmintos y su clínica característica es INCORRECTA: A) Ascaris lumbricoides - Síndrome de Löeffler. B) Giardia lamblia – Mala absorción intestinal. C) Ancylostoma duodenale - Anemia megaloblástica. D) Strongyloides stercolaris - Síndrome de hiperinfestación en inmunodeprimidos. E) Taenia solium - Convulsiones generalizadas. 30. Paciente de 30 años, seropositivo VIH conocido desde 5 años antes, con antecedentes de neumonía por P. jivovecci, que consulta por cefalea desde 10 días antes. La exploración física muestra como datos más relevantes mínima rigidez de nuca y temperatura de 37,5ºC, fondo de ojo normal, TAC: ligera atrofia cortical. La punción lumbar da salida a líquido claro con 40 células mononucleares, proteínas: 90 mgrs%, glucosa: 30 mg% (glucemia: 90 mg%). Señalar, de entre las siguientes, la causa más probable: A) Herpesvirus tipo 8. B) Listeria. C) Criptococo. D) CMV. E) VIH. 31. En relación con Streptococcus pyogenes y la faringoamigdalitis, ¿cuál de las siguientes afirmaciones NO es correcta?: A) En tratamiento de la faringoamigdalitis estreptocócica se efectúa con una sola inyección i.m. de 1,200,000 UU de penicilina benzatina. B) El tratamiento antibiótico de la faringoamigdalitis estreptocócica se efectúa con 250,000 UU/6h oral de penicilina V durante 10 días. C) El tratamiento antibiótico de la faringoamigdalitis estreptocócica se efectúa con una sola inyección i.m. de 1,200,000 UU de penicilina procaína. D) El tratamiento antibiótico de la faringoamigdalitis estreptocócica se efectúa con amoxicilina oral 500 mg/8h durante 10 días. E) El tratamiento antibiótico recomendado de la faringoamigdalitis estreptocócica en los casos de alergia a la penicilina es un macrólido oral durante 10 días. 32. A lo largo de los últimos 10 años se ha producido un cambio muy importante en la etiología de la Endocarditis Infecciosa del adulto. El microorganismo más frecuente en la actualidad, es: A) Microorganismos del grupo HACEK. B) Staphylococcus aureus. C) Estafilococos coagulasa negativos. D) Bacilos gram negativos. E) Streptococcus viridans. 33. Un hombre de 74 años con un infarto agudo de miocardio es tratado con estreptoquinasa. Seis horas después desarrolla un cuadro de hipotensión arterial severa y obnubilación. ¿Cuál de las siguientes complicaciones es MENOS probable que sea la causa?: A) Infarto de ventrículo derecho. B) Tromboembolismopulmonar. C) Rotura del músculo papilar. D) Rotura de la pared libre ventricular. E) Hemorragia cerebral. 34. En el tratamiento de la hipertensión arterial, la ventaja de los bloqueadores de los receptores de la Angiotensina II con respecto a los inhibidores del enzima conversor de la Angiotensina es que: A) Son más potentes. B) Producen menos tos. C) No producen hiperpotasemia. D) Se puede dar en embarazadas. E) Se pueden dar en sujetos con estenosis de la arteria renal bilateral. 35. El tratamiento más eficaz para prevenir recurrencias en el aleteo o flutter auricular común es: A) Buen control de la hipertensión arterial que con frecuencia padecen estos enfermos. B) Digoxina asociada a un fármaco que disminuya la conducción en el nodo A-V (anticálcicos o betabloqueantes). C) Ablación con catéter y radiofrecuencia del istmo cavo tricúspide. D) Inserción de un marcapasos con capacidad antitaquicardia. E) Amiodarona. 36. Un paciente obnubilado con una presión arterial de 80/40 mmHg, un gasto cardíaco de 3 l/min, una presión de enclavamiento pulmonar de 14 mmHg y una presión auricular derecha de 14 mmHg, puede estar sufriendo cualquiera de las siguientes situaciones patológicas con la excepción de una: A) Taponamiento cardíaco. B) Deshidratación. C) Infarto de ventrículo derecho. D) Tromboembolismo pulmonar. E) Constricción pericárdica. 37. La campana del fonendoscopio es más adecuada que la membrana para: A) Distinguir un soplo diastólico de uno sistólico. B) Oír mejor el chasquido de apertura en pacientes con estenosis mitral. C) Valorar la presencia de un tercer y/o cuarto tono. D) Detectar el click mesosistólico del prolapso mitral. E) Auscultar a los niños pequeños y bebés, en los que la membrana no capta bien la tonalidad de los ruidos cardíacos. 38. ¿Cuál de las siguientes respuestas sobre el Delirium Tremens es la correcta?: A) Se trata del estado peculiar de intoxicación producido por el consumo de alcohol. B) Se caracteriza por la aparición de conductas impulsivas tras el consumo de una pequeña cantidad de alcohol.
  • 6. [5] Mg. Dany Colca Lic. Carolina Ayala RESIDENTADO DE ENFERMERÍA. Compendio Banco de Preguntas. C) Es un cuadro clínico con elevado índice de complicaciones psiquiátricas y con pocas complicaciones orgánicas. D) Aparece con mayor frecuencia entre los hombres alcohólicos a partir de los 50 años. E) Su tratamiento de elección son las benzodiacepinas. 39. ¿Cuáles son los trastornos perceptivos, de entre los que se enumeran, más característicos de las esquizofrenias?: A) Ilusiones hipnagógicas. B) Alucinaciones visuales zoomórficas. C) Alucinaciones o pseudoalucinacionesauditivas. D) Paraeidolias. E) Alucinosis auditivas. 40. Una paciente de 24 años acude a la Urgencia llevada por un familiar por vómitos recidivantes. Parece bulimia nerviosa. ¿Cuál de las pruebas de laboratorio es más útil para evaluar la gravedad de los vómitos?: A) Nivel de Hemoglobina. B) Nivel plasmático de Amilasa. C) Nivel sérico de Socio. D) Nivel plasmático de Calcio. E) Nivel plasmático de Creatinina. 41. Acude a urgencias un paciente de 37 años con gran postración, fiebre elevada (38,5°C) e intensa cefalea. Presenta en miembros inferiores, tronco y raíz de miembros superiores un exantema tenue eritemato- violáceo, maculoso, escasamente confluente, con afectación de palmas y plantas. Al interrogar a familiares, reconocen haber estado hace siete días en una excursión. Reexplorado el paciente, en cara posterior de la piera derecha aparece una lesión necrótica-costrosa, de 0,5 cm. de diámetro, rodeada por un halo violáceo, edematoso de 0,3-0,4 cm. El tratamiento más indicado sería: A) Doxiclina v.o. (200 mgr. cada 12 horas un día ó 100 mgr. cada 12 horas por 5 días). B) Cloxacilina 1 gr. i.v. cada 6 horas durante 10 días. C) Ceftriaxona 1 gr. i.v. cada 12 horas durante 10 días. D) TMP-SMX 500 mgr. i.v. cada 8 horas por 10 días. E) Eritromicina 250 mgr. i.v. cada 6 horas por 7 días. 42. Un paciente de 27 años ex-ADVP desde hace tres años, con muguet oral y antecedentes de neumonía por neumocistis carinii, presenta en el dedo índice, en superficie dorsal de la 2.ª falange, una lesión única, úlcero necrótica, con crecimiento serpinginoso de aproximadamente 4-5 cms. de diámetro, bordes geográficos y algunas áreas costrosas, hemorrágicas. Es extremadamente dolorosa. Su 1.ª sospecha clínica es: A) Linfoma B cutáneo. B) Ulcera secundaria al tratamiento. C) Infección por herpes simple. D) Chancro luético. E) Picadura sobreinfectada. 43. Un paciente de 67 años, comienza a presentar lesiones eritematoedematosas en brazos y abdomen. Ocasionalmente se observan ampollas salpicando el área afectada. No hay afectación de mucosas. Conserva el estado general. Se realizó biopsia de piel donde se observó las imagen de una vesícula subepidérmica con eosinófilos. En inmunofluorescencia directa se observó una imagen de depósito lineal en UDE de IgG. Tras usar la técnica de separación con sal, la banda de depósito aparece tanto en el suelo, como en el techo de la ampolla. El diagnóstico más probable será: A) Enfermedad de During-Brocq. B) Fogo Selvagen. C) Enfermedad IgA lineal. D) Epidermólisis ampollosa adquirida. E) Penfigoide ampolloso. 44. Un paciente de 44 presenta lesiones pápulo- erosivas muy pruriginosas en glúteo y rodillas, que comenzaron a salir hace meses, cursando en brotes que curan espontáneamente en una semana. Se ha realizado biopsia de piel observándose ampolla subepidérmica en tinción de hematoxilina-eosina y un depósito de IgA linea en UDE. La biopsia de vellosidades intestinales en normal. Los anticuerpos antigliadina IgA son negativos. Los anticuerpos antigliadina IgG y los anticuerpos antiendomisio son positivos. Presenta además anticuerpos antitiroideos. El tratamiento adecuado será: A) Corticoides orales porque es una enfermedad IgA lineal. B) Corticoides orales porque es una Dermatitis Herpetiforme con biopsia intestinal negativa. C) Sulfonas más dieta sin gluten, porque es una Dermatitis Herpetiforme. D) Sulfonas sin dieta, porque es una Dermtitis Herpetiforme con biopsia intestinal negativa. E) Sulfonas más inmunosupresores. 45. Un varón de 6 años presenta de manera brusca un brote de lesiones eritematoescamosas redondeadas, de pqueño tamaño, distribuídas por todo el tegumento. La semana anterior acudió a Urgencias por un cuadro catarral, con T°= 37,5°C y amigdalitis. En el cultivo faríngeo se aisló un estreptococo. La patología cutánea más probable corresponde a: A) Pitiriasis rosada. B) Eczema numular. C) Psoriasis gutata. D) Herpes circinado. E) Vasculitis séptica. 46. Un joven de 26 años, con antecedentes familiares de psoriasis, presenta un brote de lesiones en placas eritematosas, con centro amarillento y halo periférico descamativo, ovaladas, bien delimitadas, en cara anterior y posterior del tronco. La 1.ª lesión había aparecido hacía 1 semana, era de mayor tamaño y se localizaba en el tercio superior de la espalda. El
  • 7. [6] Mg. Dany Colca Lic. Carolina Ayala RESIDENTADO DE ENFERMERÍA. Compendio Banco de Preguntas. paciente conservó un buen estado general en todo momento. Probablemente el cuadro corresponde a: A) Psoriasis en pequeñas placas. B) Psoriasis tipo Von Zumbusch. C) Pitiriasis rosada de Gibert. D) Roséola sifilítica. E) Eczema diseminado. 47. Un varón de 16 años comienza a presentar vesículas de contenido claro agrupadas, de localización peribucal, que a lo largo de 1 semana se van rompiendo formándose erosiones y costras. Transcurrido este período, en el dorso de las manos aparecen 2 lesiones ampollosas con vesículas dispuestas periféricamente y alguna pápula eritematosa con centro más oscuro. Se trata de: A) Inicio de una varicela. B) Aparición de lesiones en manos por contagio directo desde la zona peribucal. C) Impétigo estafilocócico. D) Eritema multiforme minor. E) Síndrome de Stevens-Johnson. 48. Una mujer de 65 años viene presentando desde hace 2 años brotes de lesiones nodulares < de 2 cm., localizadas de forma bilateral en MMII, acompañados de livedo reticularis, febrícula y artralgias. La histología corresponde a una vasculitis leucocitoclástica. Probablemente se trata de: A) Síndrome de Sweet. B) Eritema elevatum diutinum. C) Panarteritis nodosa cutánea. D) Vasculitis urticarial. E) Enfermedad de Kawasaki. 49. Un paciente de 63 años de edad presenta una eritrodermia de varios meses de evolución con adenopatías generalizadas y más de 10% de células con núcleo cerebriforme en sangre periférica. Tiene además intenso prurito y edemas pretibiales. Su diagnóstico sería: A) Eczema seborreico. B) Exantema medicamentoso crónico. C) Eritrodermia psoriasica. D) Síndrome de Sezary. E) Parapsoriasis en grandes placas. 50. Un niño sin alteraciones cutáneas al nacimiento comienza a presentar hacia los 3 meses de edad escamas grandes y negruzcas en tronco y extremidades, afectando pliegues axilares y poplíteos. Tiene un hermano mayor igualmente afecto. Es probable que se trate de: A) Una ictiosis ligada a X. B) Una ictiosis laminar. C) Una ictiosis vulgar. D) Enfermedad de Darier. E) Ninguna de las anteriores. 51. Una mujer acude a nuestra consulta poque tras haber estado esa mañana expuesta al sol presenta una quemadura solar exagerada localizada en cara (respetando región peiorbitaria, retroauricular y submentoniana), dorso de manos y en ambas piernas hasta la altura de las rodillas. Estaba en tratamiento con un diurético tiacidico. A) Se trata de una reacción fototóxica. B) Es una reacción fotoalérgica por mecanismo tipo IV. C) Debe eliminarse el agente causante, poner tratamiento sintomático y evitar la radiación lumínica hasta que remita el cuadro. D) Es una erupción polimorfa lumínica. E) Son correctas A y C. 52. Una joven de 16 años de edad presenta varias máculas de color blanco lechoso de varios centímetros de diámetro, de distribución simétrica sobre codos, rodillas, manos y zona peribucal. En la biopsia cutánea hay ausencia de malanocitos. Tras fotoquimioterapia sistémica con psoralenos han pigmentado parcialmente las lesiones. El diagnóstico de esta paciente es: A) Esclerosis tuberosa. B) Lepra. C) Hipomelanosis guttata idiopática. D) Vitíligo. E) Hipomelanosis de Ito. 53. Niño de 10 años de edad que acude a la consulta por amigdalitis pultácea con adenopatias cervicales. Se pone tratamiento con penicilina a dosis correctas. A las 72 horas acude de nuevo al no experimentar mejoria. Debemos pensar en: A) Posible Mononucleosis infecciosa. B) Posible amigdalitis viral de etiología diferente al Estreptococo beta hemolítico del grupo A. C) Linfoma. D) V. Parainfluenzae. E) Rubéola. 54. La presencia de vómitos en la infancia es uno de los motivos más frecuentes de consulta. Pueden ser debidos a enfermedades digestivas y extradigestivas. ¿Cuál de las siguientes patologías produce menos frecuentemente vómitos en la etapa de la lactancia? A) Reflujo gastroesofágico. B) Estenosis hipertrófica de píloro. C) Apendicitis. D) Invaginación intestinal. E) Gastroenteritis. 55. Ante un niño que de forma brusca presenta hipotensión, vómitos y colapso cardiovascular que no responde a la administración de drogas vasoactivas o catecolaminas y en la analítica realizada presenta hiponatremia e hiperpotasemia, se debe considerar como probable diagnóstico: A) Insuficiencia cardíaca. B) Insuficiencia suprarrenal. C) Diabetes juvenil. D) Diabetes insípida. E) Intoxicación por monóxido de carbono.
  • 8. [7] Mg. Dany Colca Lic. Carolina Ayala RESIDENTADO DE ENFERMERÍA. Compendio Banco de Preguntas. 56. Un varón de 17 años se encuentra mareado, con vómitos y en el transcurso de unos minutos se halla tumbado en el suelo en coma, con una exploración neurológicanormal. La causa más probablesería: A) Ingestión de barbitúricos. B) Hemorragia subaracnoidea. C) Coma etílico. D) Status convulsivo. E) Tumor cerebral. 57. Un varón de 4 años tiene lesiones purpúricas palpables, simétricas, de 3 días de evolución en las extremidades inferiores. Los estudios hematológicos revelan: Hemoglobina: 10 g/dl; recuento leucocitario 16.500/mm3; recuento plaquetario 240.000/mm3 y VSG de 45 mm/hora. La etiología más probable es: A) Maltrato infantil. B) Púrpura de Schonlein Henoch. C) Enfermedad de Kawasaki. D) Meningococemia. E) Enfermedad de Von Willebrand. 58. Durante la evaluación previa al ingreso en el colegio de un niño de 5 años, se detecta retraso en el habla. Como antecedentes se recogen episodios reiterados de cuadros catarrales sin control médico. ¿Cuál de las siguientes es la casusa más probabnle de esta situación? A) Trastorno de déficit de atención con hiperactividad. B) Hipoacusia de conducción. C) Disfunción de la Trompa de Eustaquio. D) Retraso mental. E) Hipoacusia neurosensorial. 59. Niña de 7 años sin antecedentes de interés que acude a urgencias por dolor abdominal generalizado y vómitos desde doce horas antes. Deposición normal. No antecedentes quirúrgicos previos. A la exploración presenta abdomen muy distendido y dolor en zona periumbilical con aumento de ruidos intestinales, Blumberg (-). En la Radiografía de abdomen en bipedestación se observa obstrucción a nivel de intestino delgado. El diagnóstico más probable sería: A) Invaginación intestinal. B) Brida intestinal. C) Malrotación intestinal. D) Divertículo de Meckel. E) Estenosis ileal congénita. 60. Niño de 5 meses que llevan a la consulta por tos y secreción nasal desde hace 24 horas. Toma mal los biberones por presentar fatiga. Se observa febrícula y retraccion intercostal, con zonas de hipoventilación y estertores en la auscultacion respiratoria. Existen otros familiares con cuadro catarral. Se le pone tratamiento sintomático y se aconseja la revisión a las 24 horas. En la nueva visita el niño ha empeorado, con importante insuficiencia respiratoria, sibilancias, tos continua y fiebre de 38°C. ¿Cuál de los siguientes cuadros cree que presenta el niño?: A) Catarro habitual descendente con evolución a asma. B) Aspiración de cuerpo extraño. C) Bronquiolitis. D) Epiglotitis. E) Laringitis. 61. Un niño nacido a término de 2.100 gr. de peso presenta irritabilidad y temblores amplios a las 36 horas de vida. Se alimenta mal y tiene diarrea y obstrucción nasal. ¿Cuál es el diagnostico más probable?: A) Hipocalcemia. B) Hipomagnesemia. C) Déficit de piridoxina. D) Sindrome de abstinencia por adición materna a heroína. E) Hipoglucemia. 62. Lactante de dos meses que presenta llanto agudo, en crisis, desde hace 20 días, síntomas motores y heces normales para su edad. ¿El diagnóstico más probable será?: A) Gastroenteritis aguda. B) Intolerancia a la lactosa. C) Cólico del lactante. D) Otitis media aguda. E) Hernia inguinal. 63. Lactante de tres meses sin antecedentes previos de interés, que en el examen de salud se le detecta un deterioro en las adquisiciones sicomotoras, el resto de la exploración por aparatos es normal. En la anamnesis presenta sacudidas musculares breves de cabeza y extremidades superiores. ¿Cuál sería el diagnóstico más probable?: A) Cólico del lactante. B) Hemorragia cerebral. C) Síndrome de West. D) Síndrome de Lenaux-Gastaut. E) Fenilcetonuria. 64. Ante un lactante de 1,5 meses de edad que presenta ictericia debemos pensar en todos los siguientes cuadros, excepto en: A) Infección urinaria. B) Lactancia materna. C) Atresia congénita de vías biliares. D) Ictericia fisiológica. E) Hipotiroidismo congénito. 65. Un paciente acude por cuadro de dolor intenso epigástrico, de aparición brusca, acompañado de vómitos en los que sólo consigue arrojar saliva, gran distensión abdominal alta e imposibilidad para pasar más allá del esófago distal con una sonda nasogástrica, ¿cuál sería la principal sospecha diagnóstica?: A) Perforación gástrica. B) Estenosis pilórica aguda. C) Vólvulo gástrico agudo.
  • 9. [8] Mg. Dany Colca Lic. Carolina Ayala RESIDENTADO DE ENFERMERÍA. Compendio Banco de Preguntas. D) Síndrome de Boerhaave. E) Tricobezoar. 66. Un paciente de 60 años refiere dolor en epigastrio desde hace unas tres semanas y en menor medida desde meses antes, que se alivia con los alimentos y reaparece 2 horas después de las comidas, con irradiación a hipocondrio derecho. En la gastroscopia muestra una úlcera de 1 cm. en curvadura menor, con bordes netos bien definidos, ausencia de islotes de tejido dentro del nicho ulceroso, con mucosa de aspecto normal. Ante lo cual Ud.: A) Decide no practicar biopsia de la úlcera por tener características de benignidad. B) Sólo practicará biopsia del fondo del nicho. C) Practicaría biopsias múltiples independientemente de las características del nicho ulceroso. D) Algunos de los signos descritos son sugestivos de malignidad, por lo que practicaría biopsias múltiples. E) Por no tener características de malignidad claras, no practicaría biopsias. 67. Paciente de 84 años que presenta cuadro de diarrea mucosa con decaimiento generalizado y pérdida de peso. Analíticamente el paciente presenta anemia con hipopotasemia, hiponatremia e hipocloremia; al tacto rectal se palpa una masa homogénea de consistencia blanda, recubierta de moco, no dolorosa. Este paciente presentará con gran probabilidad: A) Hemorroides. B) Enteropatía pierde proteínas. C) Leiomioma de recto. D) Adenoma velloso de recto. E) Hamartoma rectal. 68. Mujer de 42 años que acude al hospital con historia de 10 años de disfagia, primero para líquidos y posteriormente para sólidos, y que en la actualidad presenta regurgitaciones de carácter principalmente nocturno. ¿Cuál sería la prueba diagnóstica esencial?: A) Rx de tórax. B) Tránsito digestivo. C) Endoscopia alta. D) Manometría esofágica. E) TAC torácica. 69. Un varón de 60 años sin antecedentes personales ni familiares de interés presenta sangre roja mezclada con las heces. El médico realiza una inspección perianal y un tacto rectal encontrando hemorroides internas grado III. La actitud más correcta sería: A) Medidas higiénicas más pomada anti hemorroidal. B) Solicitar endoscopia alta. C) Solicitar un enema opaco. D) Solicitar rectoscopia y enema opaco. E) Solicitar colonoscopia total. 70. Paciente de 43 años en tratamiento por colitis ulcerosa con corticoides y sulfasalazina; acude a urgencias por malestar general, fiebre, distensión abdominal, náuseas y vómitos. Presenta gran distensión abdominal, intenso dolor a la palpación y signos de irritación peritoneal. En la analítica presenta marcada leucocitosis con desviación izquierda. En la RX simple de abdomen se observa gran dilatación del colon. El paso siguiente es: A) Intervención quirúrgica. B) Rectocolonoscopia. C) Inmunosupresores. D) Sonda nasogástrica, sueros y corticoides iv. E) TAC abdominal. 71. Una mujer de 52 años diagnosticada de úlcera gástrica de 2 cm. con biopsia negativa por malignidad, a las 8 semanas de tratamiento con ranitidina 150 mg cada 12 horas, se somete a control endoscópico encontrándose una úlcera de 0,5 cm, con nueva biopsia negativa y totalmente asintomática. ¿Cuál es la conducta preferida?: A) Cambiar el medicamento a fenotidina. B) Añadir curalfato a la ranitidina. C) Remitir por cirugía. D) Continuar con ranitidina 8 semanas más. E) Suspender toda medicación. 72. Una mujer de 76 años presenta vómito en posos de café. En la endoscopia se encontró un pólipo en cuerpo gástrico. No se identificó ningún otro origen de la hemorragia. El hematócrito es del 28%. ¿Cuál es la mejor opción terapéutica?: A) Biopsia con pinzas por estudio histológico, si es benigna ninún tratamiento adicional. B) Biopsia con pinzas, si es benigna, antagonistas H2. C) Polipectomía endoscópica con asas. D) Resección quirúrgica. E) Actitud expectante por si se repitiese la hemorragia. 73. Un paciente de 45 años con anemia ferropénica y colonoscopia normal se sometió a endoscopia alta observándose un duodeno con mucosa testoreada sin lesiones sugerentes de hemorragia. La biopsia mostró atrofia total de vellosidades, lo cual sugiere: A) Glardiasis. B) Mucosa normal. C) Celíaca. D) Enfermedad de Crohn. E) Linfoma intestinal. 74. Una mujer de 65 años con artritis reumatoide deformante grave se presenta en el hospital por dolor periumbilical de inicio nocturno, con aumento rápido de intensidad. La exploración abdominal era casi normal con hemocultivo positivo, recuento de leucocitos de 20.000 mm3 con desviación a la izquierda y VSG > 100 mm/h. ¿Cuál es el diagnóstico más probable?: A) Colitis ulcerosa. B) Enfermedad de Crohn. C) Isquemia intestinal. D) Parasitosis intestinal. E) Angiodisplasia de colon.
  • 10. [9] Mg. Dany Colca Lic. Carolina Ayala RESIDENTADO DE ENFERMERÍA. Compendio Banco de Preguntas. 75. Varón de 78 años con debilidad, pérdida de peso, diarrea, artritis y fiebre, durante el último año. En el examen físico: pérdida de masa muscular, linfadenopatía y tumefacción en rodilla izquierda. En pruebas de laboratorio: anemia ferropénica y hemault positivo. Con Rx abdomen y enema opaco banales. El diagnóstico diferencial debe incluir todos los siguientes, excepto: A) SIDA. B) Enfermedad de Crohn. C) Vasculitis reumatoide. D) Enfermedad de Whipple. E) Singellosis. 76. Se hace colonoscopia en un enfermo, donde aparece un pólipo de 8 mm en colon sigmoide que se extirpa sin observar otras lesiones. ¿Cuál de los siguientes considera el intervalo de vigilancia en este paciente?: A) Seis meses. B) Un año. C) Dos años. D) Tres años. E) Cinco años. 77. Niña de 6 años remitida a consulta por hemorragia vginal, que presenta un desarrollo mamario, en etapa III de Tanner, estatura alta, con una edad ósea de 9 años (Rx. de mano y muñeca izda.), valores basales de gonadotropinas y estradiol elevados para la prepubertad con ovarios aumentados para la edad y con múltiples quistes de diámetro igual o mayor de 14 mm. Examen neurológico clínico- radiológico normal, sin pigmentación cutánea en mancha de café con leche y estudio hormonal tiroideo y suprarrenal normal. El tratamiento de elección es: A) Laparocopia diagnóstico-terapéutica ya que su etiología es un tumor ovárico. B) Agonistas de GnRH, ya que estamos ante una PPV, y los agonistas de GnRH son los únicos que retrasan el desarrollo sexual y la maduración esquelética. C) Danazol. D) Acetato de medroxiprogesterona ya que producen un retraso en el desarrollo sexual y muy buenos resultados en el control del crecimiento. E) No precisa tratamiento, aunque sí una vigilancia anual. 78. Una paciente de 30 años, que consulta por amenorrea secundaria, presenta concentraciones plasmáticas basales de FSH 2 MUI/ml., LH 1,5, MUI/ml., prolactina 9 ngr./ml. Tras la administración de gestágenos 10 mgr./día durante 5 días no se observa sangrado vaginal. En cambio, tras la administración de estrógenos conjugados durante 21 días y en los 5 últimos días gestágenos aparece una menstruación. De las siguientes causas de amenorrea ¿cuál es la que corresponde con el cuadro clínico?: A) Síndrome de ovario poliquístico. B) Fallo ovárico autoinmune. C) Prolactinoma hipofisario. D) Síndrome de Asherman. E) Tumor cerebral. 79. Una mujer de 54 años solicita tratamiento hormonal sustitutivo por síntomas neurovegetativos y manifestaciones genitourinarias importantes. Refiere que la >FUR fue hace un año y medio, pero que hace 2 meses ha empezado a sangar muy abundantemente. Presenta exploración física y mamografía normal. ¿Cuál es la actitud más apropiada en esta paciente?: A) Tratamiento con gestágenos. B) Tratamiento con estrógenos. C) Tratamiento con terapia combinada. D) Ecografía transvaginal para valorar línea media y si existe alguna duda de patología endometrial realizar histología endometrial. E) Citología (triple toma). 80. Una mujer de 45 años con el antecedente de un proceso gripal hace dos semanas por el que fue tratado con amoxicilina presenta ahora un cuadro de prurito vulvar y leucorrea. En la exploración se aprecia enrojecimiento y edemas de la vulva y del introito y secreción vaginal blanca grumosa de aspecto caseoso. En la mucosa vaginal aparecen unas placas blanquecinas irregulares que se desprenden con facilidad y dejan ulceraciones rojas superficiales. ¿La etiología probable es?: A) Candidiasis o moniliasis. B) Herpes genital. C) Tricomaniasis. D) Gardnerella vaginalis. E) Clamidias. 81. Mujer de 40 años que consulta por cuadro de poliartritis simétrica de grandes y pequeñas articulaciones de 15 días de evolución. En la analítica destaca un factor reumatoide positivo siendo el resto del estudio inmunológico negativo. ¿Qué diagnóstico realizaría?: A) Artritis reumatoide. B) Artritis paraneoplásica. C) Artritis no filiada. D) Enfermedad de Still del adulto. E) Poliartritis vírica. 82. Mujer de 65 años que ingresa por cuadro de cefalea, fiebre y dolor con limitación de ambos hombros y ambas caderas de 2 meses de evolución. El resto de anamnesis y exploración física no aporta datos relevantes. La analítica pone de manifiesto una gran elevación de los reactantes de fase aguda (VSG y PCR). La conducta a seguir sería: A) Diagnosticar a la paciente de polimialgia reumática e iniciar tratamiento. B) Realizar biopsia de la arteria temporal para descartar la existencia de arteritis pues el tratamiento difiere.
  • 11. [10] Mg. Dany Colca Lic. Carolina Ayala RESIDENTADO DE ENFERMERÍA. Compendio Banco de Preguntas. C) Iniciar tratamiento con prednisona 1 mg./kg./día para “curarse en salud” ante la posible existencia de una arteritis. D) Realizar artrocentesis de un hombro o una cadera para descartar primero la existencia de una artritis séptica. E) Iniciar tratamiento con 20 mg./día de prednisona y si no mejorara en 15 días subir a 1 mg./kg./día. 83. Varón de 25 años afecto clínica y radiológicamente de una sacroileítis unilateral de 2 meses de evolución. La conducta a seguir es la siguiente: A) Solicitar analítica con HLA y dar tratamiento con AINE ante la sospecha diagnóstica de espondiloartropatía. B) Además de lo anterior añadir salazopirina al tratamiento pues probablemente padecerá una espondilitis anquilosante. C) Además de lo anterior realizar una buena anamnesis y exploración física para descartar la existencia de psoriasis. D) Realizar anamnesis y exploración física, solicitar analítica con HLA así como ppD y serologías a brucella. E) Lo primero es realizar una artrocentesis de la articulación afecta para descartar proceso infeccioso crónico. 84. Mujer de 45 años que acude a la consulta por “dolor óseo generalizado” desde hace varios años. La conducta a seguir es: A) Diagnosticarla de fibromialgia y tratarla con analgésicos y antidepresivos. B) Sospechar la posible existencia de proceso metastásico y realizar un completo estudio de búsqueda del tumor primario. C) Realizar una correcta anamnesis, exploración física y solicitar analítica completa con hormonas tiroideas y CPK. D) Además de lo explicado en el apartado 3., solicitar una gammagrafía ósea para asegurarse de la existencia de proceso inflamatorio articular. E) Realizar una anamnesis y exploración física correctas. Solicitar un estudio analítico con VSG y PCR. Cuando exista sospecha clínica solicitar determinación de hormonas tiroideas, anticuerpos anti-ANA y creatin- P-quinasas. 85. Varón de 45 años que acude a la consulta por presentar dolor, tumefacción y edema a nivel de la mano izquierda. Entre sus antecedentes patológicos únicamente destaca la luxación del hombro izquierdo 1 mes antes. La conducta a seguir es la siguiente: A) Realizar artrocentesis de la muñeca para descartar artritis séptica. B) Solicitar analítica y radiografías ante la sospecha de artritis de muñeca. C) Solicitar analítica, radiografías y gammagrafía ósea con tecnecio ante la sospecha de DSR. D) Solicitar analítica con inmunología y radiografías ante la sospecha de inicio de proceso inflamatorio crónico. E) Solicitar analítica, radiografías y gammagrafía con tecnecio y galio ante la sospecha de DSR. 86. Mujer de 20 años que acude por dolor y tumefacción de ambos tobillos junto lesiones cutáneas eritematovioláceas dolorosas en ambas EEII de 10 días de evolución. El diagnóstico más probable es: A) Artritis reactiva. B) Vasculitis. C) Sarcoidosis. D) Enfermedad inflamatoria intestinal. E) Artropatía no filiada. 87. Una paciente de 55 años sin antecedentes patológicos de interés consulta por dorsalgia de inicio brusco y ritmo mecánico de dolor. Aporta Rx simple de columna en la que se aprecian aplastamientos vertebrales múltiples. La exploración física es normal salvo por la presencia de debilidad muscular proximal. Ante la sospecha clínica de osteomalacia, cuál de las siguientes exploraciones es más rentable: A) VSG y hemograma. B) Determinación de calcio, fosforo y fosfatasas alcalinas. C) Densitometría ósea. D) Gammagrafía ósea. E) Resonancia magnética. 88. Una paciente de 23 años con antecedentes de ulcus duodenal presenta una poliartritis simétrica con afección predominante de manos. En la exploración física, aparte de la poliartritis, presenta aftas orales. Se practica analítica general que muestra como únicas alteraciones VSG: 40 mm/1.ª hora, leucocitos: 3.000/mm3 (1.200 linfocitos) y ANA + 1/320 patrón homogéneo. ¿Cuál sería su diagnóstico?: A) Artritis reumatoide. B) Lupus eritematoso sistémico. C) Gota poliarticular. D) Condrocalcinosis. E) Artritis reactiva. 89. ¿Cuál es el tratamiento indicado en la paciente de la pregunta anterior?: A) Antiinflamatorios no esteroideos. B) Antipalúdicos. C) Glucocorticoides orales. D) Pulsos de metilprednisolona. E) Inmunosupresores. 90. Un paciente de 45 años consulta porque en una analítica de rutina se ha detectado una uricemia de 9 mg./dl. No existen antecedentes de artritis ni cálculos urinarios. ¿Cuál es la actitud correcta?: A) Iniciar tratamiento con uricosúricos. B) Iniciar tratamiento con uricosúricos y colchicina. C) Iniciar tratamiento con alopurinol. D) Iniciar tratamiento con alopurinol y colchicina. E) No precisa tratamiento.
  • 12. [11] Mg. Dany Colca Lic. Carolina Ayala RESIDENTADO DE ENFERMERÍA. Compendio Banco de Preguntas. 91. Una paciente de 15 años es remitida por sospecha de fiebre reumática: tras un cuadro gripal con artromialgias generalizadas se determinaron las antiestreptolisinas (ASLO), que son positivas. ¿Cuál de las siguientes afirmaciones es verdadera?: A) Ante el cuadro clínico de la paciente y las ASLO +, el diagnóstico de fiebre reumática es seguro. B) Con las ASLO +, bastaría para diagnosticar la fiebre reumática. C) La fiebre reumática es un tipo de artritis séptica. D) Con estos datos, no se puede realizar el diagnóstico de fiebre reumática. E) Debemos iniciar rápidamente tratamiento con penicilina. 92. Una paciente de 40 años presenta una gonartosis con afección predominante del compartimento interno de la rodilla e impotencia funcional severa. Señale la respuesta verdadera: A) El tratamiento es reposo absoluto y esperar a que sea más mayor para colocarle una prótesis. B) Una osteotomía podría estar indicada. C) Nunca se debe tratar con antiinflamatorios. D) No debe realizar fisioterapia ya que podría dañarse más la articulación. E) El uso de un bastón está contraindicado. 93. Un paciente presenta latencia del sueño de menos de 10 minutos con dificultades para despertarse y episodios de sueño de 18-20 horas, asociado al despertarse con hiperfagia, hipersexualidad, aumento de peso, irritabilidad, depresión, comportamiento impulsivo, disfunción vegetativa y alteraciones neurológicas. Las siestas diurnas duran varias horas. Estos episodios duran varias semanas intercalándose varios meses sin somnolencia. En uno de los episodios de somnolencia el paciente tuvo un grave accidente. El diagnóstico más probable es: A) Epilepsia. B) Narcolepsia. C) Simulación o trastorno conversivo-histeria. D) Síndrome de Klein-Levin. E) Apnea del sueño. 94. Una chica de 17 años acude al servicio de urgencias traída por su madre por un desmayo. Cuando la vemos está irritable y dice querer irse de alta porque no le pasa nada. Va con ropas holgadas y algo más abrigada que el resto de pacientes, está delgada y parece más joven de lo que le corresponde por su edad. Dice comer normal y niega usar laxantes o diuréticos, su madre dice que no come con ellos porque es muy activa y está todo el día fuera. Rompió con el novio hace un año. Dice que está harta de tener a su madre siempre pendiente de ella. El dato que más nos ayudaría a diferenciar una anorexia nerviosa de otros trastornos sería: A) Presencia de amenorrea. B) Peso inferior al 15%. C) Que la paciente diga que tiene miedo a ganar peso o que diga que se ve gorda. D) Que diga que tiene frío. E) Uñas y pelo frágil. 95. Una adolescente de apariencia física normal acude por irritabilidad y tristeza. Comenta que decidió comer menos en las comidas porque estaba algo gorda. Haciendo esto consigue adelgazar a temporadas pero luego se vuelve a engordar. Ahora está en período de transición, sigue restringiendo las comidas pero se pasa todo el día picoteando y en ocasiones come más de lo que quisiera de forma descontrolada. En esos descontroles efectivamente la paciente come una gran cantidad de alimentos especialmente chocolate, galletas y dulces, aunque en otras ocasiones son salados. Tras estos atracones se siente muy culpable e irritada y vomita para evitar engordarse. El diagnóstico más probable es: A) Síndrome de Klein-Levin. B) Depresión bipolar. C) Bulimia. D) Esquizofrenia. E) Anorexia nerviosa tipo compulsivo/purgativo. 96. Una paciente de 20 años se presenta en la guardia quejándose al internista de que tiene ataques de corazón y sensación de ahogo, sudoración y sensación de mareo que se inicia bruscamente mientras estaba leyendo relajadamente en su casa, le duró unos minutos, creyó morir y sentía que lo que le ocurría no era real, temiendo perder el control o estar volviéndose loca. Se le realizan pruebas ECG y auscultación, detectándose frecuencia cardíaca alta y signos sugerentes de prolapso mitral. Aun así dada la gran ansiedad de la paciente se llama al psiquiatra ya que insiste en que se le hagan más pruebas y en que no se quiere quedar sola en casa nunca más, ¿cuál es el posible diagnóstico?: A) Prolapso mitral. B) Agorafobia. C) Angor. D) Trastorno de angustia. E) A y D. 97. Una paciente de 55 años acude a consultas por cansancio especialmente por la mañana, falta de concentración desde hace 1 mes. Dice estar por las mañanas despierta antes de hora sin poder dormir. Ha perdido apetito si bien come como siempre pero forzándose, por lo que no ha perdido peso. Pierde el hilo de las conversaciones y está irritable. Ha dejado de hacer las cosas que le gustaba hacer y no sale de casa más que lo justo. Viene acompañada por su hermana y nos ruega que no le digamos nada a su marido ya que desde que no hace las cosas de la casa tan bien como antes no quiere darle más disgustos. ¿Cuál es el diagnóstico más probable?: A) Ansiedad y depresión. B) Demencia. C) Depresión mayor. D) Trastorno de personalidad.
  • 13. [12] Mg. Dany Colca Lic. Carolina Ayala RESIDENTADO DE ENFERMERÍA. Compendio Banco de Preguntas. E) Neurosis. 98. El paciente, que es traído por un familiar, dice no saber porqué está aquí y parece irritado y con agresividad contenida. El familiar nos hace gestos con los ojos haciendo ver que el paciente no está muy bien. Entrevistando a ambos por separado el paciente se muestra contenido y dice estar bien, mejor que nunca. El familiar dice que últimamente ha comprado cosas innecesarias y ha hecho algunos regalos. Dice que quiere iniciar la carrera de psicología, se ha apuntado a un gimnasio y quiere proponer a su jefe algunos cambios en la empresa si bien no tiene responsabilidades directivas. Está muy hablador y agudo en las conversaciones pero irritable, dominante y con explosiones de mal genio. Debe dormir 4 horas pero no está cansado al día siguiente, no hay problemas de apetito. No ha habido problemas en su trabajo, pero pasa mucho tiempo en la calle. ¿Cuál es el diagnóstico más correcto?: A) Esquizofrenia. B) Manía. C) Depresión ansiosa. D) Trastorno esquizoafectivo. E) Trastorno de personalidad. 99. Paciente traído por la policía por haberlo encontrado andando descalzo por la carretera. El paciente tiene 28 años, es de otra ciudad y por la documentación que lleva parece que pueda ser estudiante. Parece deshidratado, completamente desaseado y con olor a orín. Pupilas normales. La piel está tostada sin quemaduras en las zonas de exposición al sol. Está ausente, como absorto de manera que al hablarle más fuerte se sobresalta y nos atiende momentáneamente sonriéndonos de forma inapropiada hasta que ladea la cabeza y la gira sin motivo. Murmura algo y mira al techo, permaneciendo con los brazos extendidos. ¿Cuál es el diagnóstico más probable?: A) Intoxicación por LSD. B) Esquizofrenia. C) Demencia. D) Cuadro maníaco. E) Histeria. 100. Acude la madre de un presunto paciente de 18 años porque su hijo no sale casi nada de casa, parece evitarles. Se hace su propia comida a partir de conservas generalmente y come aparte. Nunca usa conservas o alimentos ya empezados, ni acaba los que no usa por completo. Alguna vez lo ha visto en el pasillo haciendo posturas extrañas o dando golpes de kárate. Fue aficionado a las lecturas de ovnis y ciencia ficción. Fue regular estudiante. No trabaja ni estudia. Cierra todas las persianas y se queja de que los vecinos están demasiado pendientes de ellos. Por las noches a veces no duerme. Dice que puede oír al vecino del tercero (ellos viven en el primero) hablar mal de ellos y que siempre está comentando lo que él va haciendo. ¿Cuál es el diagnóstico más probable?: A) Esquizofrenia. B) Depresión psicótica. C) Manía. D) Demencia. E) Klein-Levin. TEMA B 1. Paciente de 64 años ingresado para cirugía diferida que al día siguiente del ingreso presenta cuadro de hiperactividad vegetativa, temblor, sobresaltos, hiperactividad e hiperalerta pero con confusión, alucinaciones y convulsiones. ¿Cuál es la causa más frecuente?: A) Abstinencia a alcohol. B) Sobredosis de neurolépticos. C) Sobredosis de benzodiacepinas. D) Sobredosis de anticolinérgicos. E) Ingestión de barbitúricos. 2. Mujer de 25 años que acude por intento autolítico con benzodiacepinas. Es el quinto intento en 7 años, en esta ocasión tras una pelea familiar. Su madre dice que es muy voluble y caprichosa que siempre consigue salirse con la suya mediante grandes crisis de nervios y peleas. La madre dice estar harta ya que siempre está regañándola mientras que su marido la perdona todo y no la castiga lo suficiente. No tiene novio fijo ya que rompe con ellos en seguida, es mala estudiante y no es capaz de dedicarse a nada fijo ya que se aburre en seguida, dice sentirse en ocasiones muy vacía y en otras muy inspirada en hacer el bien pero nada le dura mucho, quisiera poder tener una relación con un hombre romántico. Sale mucho por la noche y en ocasiones no llega hasta el día siguiente por la tarde, llegando ebria en ocasiones. Ha perdido a sus amigas, que se quejaban entre otras cosas de que las estaba llamando continuamente. Le han diagnosticado previamente de hepatitis. El diagnóstico de esta paciente es: A) Esquizofrenia. B) Manía. C) Trastorno de personalidad límite o inestable. D) Trastorno de personalidad por dependencia. E) Trastorno esquizoide. 3. El paciente acude a nuestra consulta porque quisiera dejar de hacer algunas de las cosas que actualmente hace ya que le quitan mucho tiempo. El paciente tarda mucho tiempo en vestirse ya que ha de pensar la ropa que le dará suerte para esa mañana. Teme que si no coge la adecuada algo podría salir mal. No puede evitar sumar los números que ve en las matrículas de los coches, así consigue neutralizar los pensamientos que le vienen sobre la posibilidad de que el coche tenga un accidente. El sabe que éstos son tonterías suyas pero son cosas que no puede quitarse de la cabeza. Es muy ordenado en algunos aspectos pero se le acumula el
  • 14. [13] Mg. Dany Colca Lic. Carolina Ayala RESIDENTADO DE ENFERMERÍA. Compendio Banco de Preguntas. trabajo y no consigue hacer las cosas que tiene preparadas en una lista para ese día. Le gusta corregir sus escritos hasta que estén impecables, lo mismo le ocurre con su trabajo, es muy perfeccionista, exigente consigo mismo y con los demás, es incapaz de delegar trabajo. El diagnóstico del paciente es: A) Esquizofrenia. B) Trastorno de personalidad dependiente. C) Trastorno obsesivo-compulsivo. D) Fobia. E) Depresión. 4. Paciente que acude por no poder salir de casa sola. Dice que le da miedo salir por sitios solitarios por si le ocurre algo y que en los sitios con mucha gente se agobia y se pone muy nerviosa teniendo crisis de ansiedad con palpitaciones, se imagina así misma necesitada de ayuda y sin poder salir. Si se encuentra acompañada se encuentra mejor ya que esa persona podría ayudarla si ocurriera algo. Desde hace unos meses tiene unas crisis parecidas pero más cortas estando relajada, incluso acompañada, sin aviso, la primera estando en la calle distraída viendo un escaparate. Cuando le dan en casa es capaz de salir a la calle para coger aire o bien abre las ventanas. Desde la primera crisis cogió algo de miedo a estar sola hasta llegar a la situación actual. Se ha comprado un teléfono móvil para poder pedir ayuda si fuera necesario. Si entra en un cine siempre está pendiente de sentarse cerca de alguna salida. El cuadro clínico de la paciente es: A) Agorafobia. B) Hipocondría. C) Trastorno facticio. D) Trastorno obsesivo. E) Agorafobia con crisis de pánico. 5. Ante un paciente con dolor epigástrico irradiado en hemicinturón, náuseas, distensión abdominal, descenso de los ruidos hidroaéreos y ascenso del ST en el electrocardiograma,¿cuál sería su actitud?: A) Llamaría de inmediato a la Unidad de Cuidados Intensivos para tratar el infarto agudo de miocardio. B) Repetiría el electrocardiograma a las 8 horas para confirmar el diagnóstico. C) Lo diagnosticaría de pancreatitis aguda. D) Su diagnóstico sería aneurisma disecante de aorta. E) El paciente tiene una pericarditis aguda. 6. Ingresa en el Servicio de Urgencias un paciente con dolor epigástrico irradiado en hemicinturón y amilasa elevada, que pierde de forma brusca visión ¿qué pensaría que ha ocurrido?: A) Retinopatía de Purtscher. B) Accidente cerebro vascular agudo de la región occipital posterior. C) Desprendimiento de retina. D) Simulación. E) Glaucoma agudo. 7. Ante una paciente que ingresa en el Servicio de Urgencias con dolor abdominal, elevación de la amilasa y la glucosa en suero y con un pH de 7,1 ¿en qué pensaría primero?: A) Pancreatitis aguda. B) Rotura de embarazo ectópico. C) Quiste de ovario. D) Cetoacidosis diabética. E) Ulcera de estómago. 8. Un paciente de 18 años consulta por ictericia sin fiebre y coluria sin prurito. En las pruebas complementarias presenta: Ac. anti VHA IgG positivos, Ac. antiHBs positivos, Ac. anti HBc IgG positivos, ecografía hepática normal, hay predominio de la bilirrubina directa con monoconjugados, la colecistografía oral es normal y al dar al paciente fenobarbital se observa disminución de las cifras de bilirrubina total. ¿Cuál sería su diagnóstico de sospecha?: A) Coledocolitiasis. B) Síndrome de Gilbert. C) Síndrome de Dubin-Johnson. D) Hepatitis aguda por VHA, con hepatitis aguda por VHB curada. E) Síndrome de Rotor. 9. Ante un paciente con cirrosis alcohólica en estadío C- 10 de Child-Pugh que ingresa por descompensación hidrópica, y que se encuentra hipotenso, taquicárdico, oligúrico, con un sodio sérico de 125 mEq/l. y de 5 mEq/l. en orina, con sedimento normal, creatinina sérica 3 mg./dl. y un aclaramiento de creatinina de 40 ml., ¿en qué pensaría usted?: A) Deficiente tratamiento diurético. B) Insuficiencia renal prerrenal secundaria a tercer espacio. C) Síndrome hepatorrenal. D) Glomerulonefritis mesangial IgA. E) Insuficiencia renal prerrenal secundaria a deshidratación. 10. Paciente varón de 28 años, VIH(+), en tratamiento con DDI (Dideoxinosina), que acude al hospital por dolor intenso en epigastrio irradiado hacia la espalda, que mejora al flexionar el tronco, acompañado de náuseas y vómitos. ¿Cuál de los siguientes diagnósticos es el más probable?: A) Pericarditis. B) Obstrucción intestinal. C) Apendicitis. D) Pancreatitis aguda. E) Endocarditis. 11. Tras una semana de ingreso por una pancreatitis aguda, a pesar del tratamiento médico sintomático persiste la fiebre, leucocitosis e hiperamilasemia. Se aprecia a la palpación una masa abdominal localizada en hipocondrio derecho. ¿Cuál es el diagnóstico más probable?:
  • 15. [14] Mg. Dany Colca Lic. Carolina Ayala RESIDENTADO DE ENFERMERÍA. Compendio Banco de Preguntas. A) Carcinoma de páncreas. B) Flemón pancreático. C) Pancreatitis crónica. D) Plastón secundario a perforación duodenal. E) Quiste hidatídico. 12. El principal diagnóstico de presunción ante una enferma que acude por un cuadro de diarrea acuosa, dos úlceras duodenales resistentes al tratamiento médico e hipercalcemia es: A) Insulinoma. B) Gastrinoma. C) Adenocarcinoma de páncreas. D) Somatostatinoma. E) Vipoma. 13. Varón de 60 años que acude por pérdida de peso de 10 kilos, dolor sordo en piso abdominal superior de 3 meses de evolución, acompañado de ictericia mucocutánea y deposición de color blanco desde hace 5 días. A la exploración destaca masa palpable en hipocondrio derecho. ¿Qué patología se sospecharía en primer lugar?: A) Adenocarcinoma pancreático. B) Tumor gástrico. C) Coledocolitiasis. D) Colecistitis. E) Hepatitis aguda. 14. Varón de 45 años, etilismo crónico, con dolor abdominal en el hipocondrio izquierdo, de 3 meses de evolución, que empeora con la ingesta y se acompaña de deposiciones diarreicas pastosas muy mal olientes. En las exploraciones complementarias destaca: glucemia de 280, amilasemia en los límites de la normalidad y en la placa de abdomen múltiples calcificaciones a nivel de L2: A) Pancreatitis aguda. B) Cólico biliar. C) Pancreatitis crónica. D) Hepatitis aguda. E) Ulcus gástrico 15. Paciente de 60 años con fibrilación auricular en tratamiento con amiodarona desde hace meses, colelitiasis diagnosticada ecográficamente e insuficiencia renal crónica moderada. Consulta por cuadro de anorexia, astenia, náuseas, vómitos e ictericia mucocutánea. En los datos de laboratorio destaca un leve aumento de las transaminasas y una creatinina de 2,2. Se realiza una biopsia hepática, observándose al microscopio electrónico cuerpos lisosómicos lamelares cargados de fosfolípidos. Qué proceso patológico le sugieren estos datos: A) Hepatitis viral aguda. B) Cólico biliar. C) Pancreatitis aguda. D) Hepatitis tóxica de origen medicamentoso. E) Hepatitis viral crónica. 16. Un varón de 13 años que había presentado un cuadro febril de vías respiratorias altas de una semana de evolución acude a urgencias por vómitos súbitos e incoercibles. La semana previa había consumido ácido acetilsalicílico para la sintomatología respiratoria y tres semanas antes estuvo en contacto con un paciente con hepatitis aguda por virus B. Dos días después del ingreso el paciente se encuentra estuporoso con convulsiones sin signos neurológicos de focalidad y dolor en hipocondrio derecho con hepatomegalia. ¿Cuál sería su diagnóstico?: A) Cuadro convulsivo en relación con la fiebre. B) Hepatitis fulminante vírica. C) Síndrome de Reye. D) Reagudización del cuadro gripal. E) Intoxicación por ácido acetilsalicílico. 17. Un hombre de 21 años recibió un golpe en el escroto dos horas antes de ser examinado en urgencias. Su escroto está tenso, hinhado, y equimótico. No se pueden palpar los testículos. El paso siguiente es: A) Hacer uretrografía retrógrada. B) Hacerle un Doppler de flujo color. C) Realizar una ecografía del escroto. D) Tratarle con hielo, reposo y suspensorio. E) Hacer exploración quirúrgica del escroto. 18. Un hombre de 64 años presenta una hinchazón indolora del testículo derecho de tres meses de duración. Los resultados de los análisis de orina son normales y la ecografía testicular muestra un aumento del tamaño de dicho teste. El diagnóstico más probable es: A) Linfoma testicular. B) Leucemia linfática crónica. C) Seminoma espermatocítico. D) Teratocarcinoma. E) Carcinoma de células embrionarias. 19. Una mujer sana presenta de forma aguda polaquiuria y disuria. En el sedimento urinario se observan más de 5 leucocitos por campo y el urinocultivo revela 1.000 colonias de E. coli por ml. El diagnóstico más probable es: A) Uretritis por clamydia. B) Síndrome uretral. C) Bacteriuria por E. coli. D) Cistitis intersticial. E) Cistitis quística. 20. Un hombre de 74 años con cáncer de próstata metastásico conocido presenta dolor agudo de cadera derecha. Hace dos años se le practicó una orquiectomía pero no ha recibido ningún otro tratamiento. Su estado general es bueno. Una gammagrafía ósea muestra metástasis difusas y una Rx simple revela una osteólisis en el acetábulo derecho. La siguiente medida a aplicar es: A) Flutamida.
  • 16. [15] Mg. Dany Colca Lic. Carolina Ayala RESIDENTADO DE ENFERMERÍA. Compendio Banco de Preguntas. B) Análogos de la LH-RH. C) Radioterapia localizada. D) Fosfato de estramustina. E) Ketoconazol. 21. Un recién nacido presenta hematuria, proteinuria y creatinina elevada. Las presiones de la arteria umbilical están significativamente altas y el paciente desarrolla una insuficiencia cardíaca congestiva. Una gammagrafía renal revela una ausencia de función en el riñón izquierdo. El diagnóstico más probable es: A) Trombosis de la vena renal. B) Necrosis cortical renal. C) Hemorragia adrenal. D) Rotura de un nefroma mesoblástico. E) Trombosis de la arteria renal. 22. Un hombre de 35 años presenta un cólico renal izquierdo. En la Rx de abdomen se observa un cálculo de 3 mm. de diámetro alojado en uréter medio. La necesidad de tratamiento quirúrgico depende de: A) La duración de los síntomas del paciente. B) Del número de episodios de cólicos previos. C) Del número de intervenciones quirúrgicas previas. D) De la presencia de fiebre e infección urinaria. E) De una anormalidad metabólica subyacente. 23. A un paciente se le somete a una intervención de bypass en el intestino delgado y presenta una litiasis urinaria. ¿Cuál será la composición más probable de la litiasis?: A) Acido úrico. B) Urato amónico. C) Fosfato cálcico. D) Oxalato cálcico. E) Estruvita. 24. El factor pronóstico más importante en los niños que presentan un tumor de Wilms intracava es: A) La histología. B) El volumen del tumor. C) La extensión atrial del tumor. D) La afectación de ganglios linfáticos. E) La diseminación del tumor durante la intervención quirúrgica. 25. Un muchacho de 7 años presenta de forma súbita dolor escrotal derecho de 4 horas de duración. Se sospecha de una torsión testicular. ¿Cuál de las siguientes observaciones es la más probable?: A) La ausencia del reflejo cremastérico. B) El aumento de la captación por parte del teste derecho después de la exploración radioisotópica. C) La presencia del reflejo cremastérico. D) Una transiluminación correcta del compartimento escrotal derecho. E) La estetoscopia con Doppler será simétrica en ambos compartimentos escrotales. 26. En un paciente asmático, ¿cuál de los siguientes fármacos está contraindicado en el tratamiento de la incontinencia?: A) La efedrina. B) Sudafed. C) El propranolol. D) La fenilefrina. E) Las anfetaminas. 27. Ante un paciente de 60 años con un cáncer de próstata localizado, mal diferenciado y sin metástasis, qué tratamiento le recomendaría para intentar aumentar su supervivencia: A) Prostatectomía radical. B) Análogos de la LH-RH. C) Flutamida. D) Estramustina. E) Orquiectomía. 28. Ante un paciente de 55 años que presenta una tumoración vesical que infiltra la muscular y que ocupa la mitad de la vejiga, ¿cuál sería el tratamiento que emplearía con intención curativa?: A) Cistectomía radical y derivación urinaria. B) Resección transuretral vesical. C) Quimioterapia intravesical. D) Quimioterapia sistémica. E) Cistectomía parcial. 29. Paciente mujer de 75 años de edad, que es traída a la consulta por cambio progresivo de conducta en los últimos meses. Previamente habían observado fallo de memoria reciente sin poder precisar el momento de inicio. Destacaba a la exploración lenguaje pobre y fallos de juicio. ¿Cuál es el diagnóstico más probable en esta paciente?: A) Depresión. B) Síndrome confusional agudo. C) Demencia tipo Alzheimer. D) Demencia multiinfarto. E) Trastorno de la personalidad. 30. Varón de 87 años, con amaurosis bilateral, que ingresa por neumonía basal derecha en la Unidad de Agudos del hospital. La noche del ingreso presenta cuadro de agitación psicomotriz, con alteración del nivel de conciencia y desorientación temporoespacial. ¿Cuál es el diagnóstico más probable?: A) Demencia. B) Depresión delirante. C) ACVA. D) Síndrome confusional agudo. E) Ninguno. 31. Paciente de 85 años de edad con antecedentes de insuficiencia cardíaca en tratamiento con diuréticos, e insomnio que trataba con lorazepam. Presenta nicturia 2-3 veces. Sufre caída al levantarse bruscamente durante la noche para ir al retrete. ¿Qué factores pueden haber contribuido e la caída?:
  • 17. [16] Mg. Dany Colca Lic. Carolina Ayala RESIDENTADO DE ENFERMERÍA. Compendio Banco de Preguntas. A) Fármacos. B) Hipotensión postural. C) Factores ambientales. D) Ninguno. E) Todos ellos. 32. Paciente de 75 años de edad, varón, que vive solo. Es encontrado la mañana del día 15 de enero caído en el suelo. A su llegada al hospital se evidencia afasia motora y hemiplejía derecha diagnosticándose de ACVA. ¿Cuál de las siguientes patologías debe ser descartada en la valoración inicial?: A) Rabdomiólisis. B) Hipertiroidismo. C) Depresión. D) Hipotermia. E) A y D. 33. Paciente mujer de 85 años que ingresa por fractura pertrocantérea de cadera derecha colocándose tracción. Se evidencia neumonía basal derecha que obliga a retrasar la intervención. La paciente sufre de incontinencia urinaria que se maneja con catéter. A los cinco días se observa enrojecimiento en región sacra que no palidece con la presión, diagnosticándose de úlcera por presión grado I. ¿Cuál es el principal factor de riesgo para esta complicación?: A) Incontinencia urinaria. B) Hipoxemia. C) Edad avanzada. D) Inmovilidad. E) Todos. 34. Paciente de 70 años que sufre caída al suelo golpeándose en la cabeza. Al cabo de unas semanas sufre trastornos de conducta, pérdida de memoria y posteriormente alteración del nivel de conciencia. ¿Qué diagnóstico debe descartarse en primer lugar?: A) Hipotiroidismo. B) Demencia tipo Alzheimer. C) Pseudodemencia. D) Hematoma subdural. E) Tumor cerebral. 35. Mujer de 75 años que consulta por incontinencia urinaria, en la que predomina la urgencia-miccional y que no sigue ningún tratamiento farmacológico habitual. A través de la exploración física no se objetiva patología orgánica, siendo el residuo postmiccional normal. El estudio analítico es normal. El tratamiento médico de elección sería: A) Sondaje vesical intermitente. B) Calcioantagonistas. C) Cirugía. D) Antocolinérgicos. E) Colector externo. 36. Durante los últimos 3 meses un varón de 80 años presenta una rápida progresión de una demencia acompañada de signos extrapiramidales y mioclonías. El diagnóstico de sospecha inicial sería: A) Demencia senil tipo Alzheimer. B) Enfermedad de Creutzfeldt-Jakob. C) Enfermedad de Huntington. D) Hidrocefalia a presión normal. E) Enfermedad de Parkinson. 37. ¿Cuál de las lesiones cutáneas siguientes se asocia más frecuentemente con neoplasia oculta en un paciente anciano?: A) Penfigoide bulloso. B) Dermatomiositis. C) Eritema multiforme. D) Herpes zoster. E) Pénfigo vulgar. 38. Anciano de 70 años con un melanoma maligno de reciente diagnóstico y sin otra patología médica. Su pronóstico vital estará más estrechamente relacionado con una de las siguientes características: A) Ausencia de regresión. B) Tipo histológico de la lesión. C) Grado de invasión. D) Presencia de ulceración. E) Lugar de la lesión. 39. Una mujer de 75 años con historia de diabetes mellitus no insulindependiente y epilepsía secundaria a enfermedad cerebro- vascular padece una inflamación con retracción gingival. Se encuentra en tratamiento con glipizida y fenobarbital. La causa más probable de su proceso gingival será: A) Caries dental. B) Edentulismo. C) Tratamiento con fenobarbital. D) Déficit de cinc. E) Pobre higiene oral. 40. Un varón de 76 años con historia de diabetes mellitus de larga evolución y con datos clínicos de polineuropatía periférica, empieza a tomar amitriptilina por prescripción médica para las parestesias en miembros inferiores. De forma rogresiva nota disminución del volumen de diuresis y ocasionalmente escapes involuntarios de orina. En la revisión médica siguiente se objetiva deterioro del estado general con insuficiencia renal. ¿Cuál es la causa más lógica de su deterioro clínico?: A) Nefrotoxicidad por amitriptilina. B) Pielonefritis aguda. C) Infección urinaria de vías bajas. D) Retención urinaria con fracaso renal secundario. E) Glomeruloesclerosisdiabética.
  • 18. [17] Mg. Dany Colca Lic. Carolina Ayala RESIDENTADO DE ENFERMERÍA. Compendio Banco de Preguntas. 41. Varón de 45 años con otalgia derecha y sensación de taponamiento auditivo, sin otorrea. A los dos días presenta aumento del dolor, que se hace retroauricular, y fiebre en agujas. El Rinne es negativo en oído derecho y el Weber lo lateraliza a la derecha. El diagnóstico más probable es: A) Colesteatoma antral invasivo. B) Petrositis. C) Tromboflebitis del seno lateral. D) Otitis externa maligna. E) Carcinoma de oído derecho. 42. Mujer de 22 años que presenta parálisis facial periférica derecha. A la exploración presenta otoscopia normal y lengua geográfica fisurada. La paciente comenta haber tenido otro episodio anteriormente, aquella vez asociado a edema de labio inferior. El diagnóstico más probable es: A) Síndrome de Guillain-Barré. B) Síndrome de Heerfordt-Waaldenström. C) Déficit de C1-inhibidor. D) Síndrome de Melkerson-Rosenthal. E) Parálisis facial de Bell. 43. Paciente que presenta hipoacusia neurosensorial izquierda de larga evolución con caída en agudo, y acúfeno persistente. La actitud correcta en este caso sería: A) No hacer nada, pues se trata de un traumatismo acústico crónico. B) Potenciales evocados para descartar neurinoma del VIII. C) Instaurar terapia vasodilatadora endovenosa, pues se trata de una hipoacusia súbita. D) Intervenir el posible colesteatoma. E) Administrar sedantes vestibulares ante la posibilidad de vértigo de Ménière. 44. Paciente de 65 años con rinorrea unilateral purulenta, dolor hemifacial y epistaxis ocasionales. En Rx se observan lesiones osteolíticas en maxilar. Probablemente se trate de: A) Cuerpo extraño nasal. B) Ocena. C) Granuloma sangrante de tabique. D) Carcinoma de fosa nasal. E) Coriza. 45. Un varón de 60 años consulta por tumoración indolora en raíz nasal, que desplaza la órbita produciéndole diplopia. El cuadro se debe probablemente a: A) Mucocele etmoidal. B) Quiste de retención en seno frontal. C) Enfermedad de Woakes. D) Papiloma invertido en techo de fosa nasal. E) Plasmocitoma solitario en seno frontal. 46. Una mujer de 45 años acude a la urgencia con un síndrome de disnea y estridor importantes, con tiraje supraclavicular. A la exploración presenta taquicardia y leve cianosis, un tiroides agrandado y, en la laringoscopia, parálisis de ambas cuerdas vocales en posición paramediana. La actitud que debe seguirse es: A) Bolo de corticoides endovenosos, ante la posibilidad de carcinoma subglótico. B) Tiroidectomía de urgencia, pues probablemente el tiroides agrandado comprime la tráquea. C) Intubación y observación. D) Traqueostomía de urgencia, pues se trata de una parálisis recurrencial bilateral secundaria a patología tiroidea. E) Administración endovenosa de espasmolíticos. 47. Un niño de 3 años es traído a urgencias con un cuadro de fiebre y tos irritativa "perruna", a lo que se asocia disfonía y cierto grado de disnea. El cuadro es compatible con todas menos: A) Cuerpo extraño en vías aéreas. B) Laringotraqueítis aguda. C) Epiglotitis aguda. D) Adenoamigdalitis aguda obstructiva. E) Edema alérgico. 48. Paciente mujer de 25 años que consulta por pérdida de audición de comienzo insidioso. Su madre era sorda y ganó audición tras operarse del oído. Otoscopia normal. Rinne negativo oído izdo., positivo oído dcho. Weber a la izda. Audiometría: hipoacusia transmisiva izquierda. Reflejo estapedial abolido en oído izdo. y presente en derecho. Timpanometría normal. El diagnóstico probable es: A) Otosclerosis oído izquierdo. B) Colesteatoma izquierdo. C) Fijación de cadena osicular derecha. D) Timpanosclerosis izquierda. E) Luxación de cadena osicular izquierda. 49. Un varón de 30 años acude a urgencias por vértigo periférico intenso con nistagmus a la derecha y acúfeno en oído izquierdo. Recuerda haber tenido déficit auditivo izquierdo previo, que le desapareció. Audiometría: hipoacusia neurosensorial izda. leve. Reflejo estapedial: derecho en 70 dB; izquierdo en 50 dB. Diagnóstico probable: A) Neuronitis vestibular. B) Neurinoma del VIII par con reclutamiento positivo. C) Tumor de tronco cerebral. D) Otitis media secretora con fístula perilinfática. E) Síndrome de Ménière. 50. Un niños de 2 años presenta rinorrea purulenta de larga evolución por fosa nasal derecha, con mala ventilación nasal. Son diagnósticos posibles todos menos: A) Tuberculosis nasal. B) Cuerpo extraño intranasal. C) Sinusitis maxilar. D) Coriza común. E) Rinitis alérgica sobreinfectada.
  • 19. [18] Mg. Dany Colca Lic. Carolina Ayala RESIDENTADO DE ENFERMERÍA. Compendio Banco de Preguntas. 51. Paciente de 30 años que acude a urgencias presentando una ulceración amigdalar unilateral. Son diagnósticos probables todos menos: A) Amigdalitis tifoidea de Duguet. B) Angina de Ludwig. C) Carcinoma escamoso de amígdala. D) Angina de Plaut-Vincent. E) Agranulocitosis. 52. Un niños de 12 años acude a urgencias por obstrucción nasal bilateral crónica y otitis seromucosa bilateral. Recientemente ha tenido epistaxis importante por ambas fosas nasales. El diagnóstico más probable es: A) Hipertrofia adenoidea. B) Ototubaritis asociada a rinitis alérgica. C) Poliposis nasal bilateral. D) Adenocarcinoma de etmoides. E) Angiofibroma nasofaríngeo. 53. Mujer de 75 años con historia de cefalea presenta pérdida súbita de visión unilateral con edema de papila ipsilateral. ¿Qué medida tomaríamos en primer lugar?: A) TAC. B) VSG. C) Ingreso hospitalario paratratamiento antibiótico iv. D) Observación. E) Radiografía centrada en el agujero óptico. 54. Mujer de 20 años con ojo rojo bilateral, acompañado de quemosis, folículos conjuntivales tarsales, adenopatía preauricular, sin pérdida de visión. La etiología más frecuente será: A) Adenovirus. B) S. aureus. C) H. influenzae. D) Queratitis herpética. E) Parainfluenzae virus. 55. Mujer de 23 años que presenta pérdida brusca de agudeza visual indolora en ojo derecho, en la exploración se observa defecto pupilar aferente relativo en ojo derecho, segmento anterior normal y F.O. normal. El diagnóstico más probable es: A) Neuritis óptica retrobulbar. B) Histeria. C) Compresión quiasmática por tumor hipofisario. D) Compresión del globo ocular por tumor orbitario. E) Glaucoma agudo de ángulo cerrado. 56. Varón de 50 años con ojo rojo y doloroso, midriasis arreactiva y cámara anterior estrecha. ¿Cuál de las siguientes respuestas es falsa?: A) No sería raro que fuera hipermétrope. B) Puede presentar cefalea con náuseas y vómitos. C) Evitar tomar la PIO por posible etiología infecciosa. D) El tratamiento definitivo es láser o cirugía. E) Puede presentar visión en halos de colores. 57. Ante un varón de 32 años con síndrome febril y pérdida visual unilateral que presenta en fondo de ojo lesiones en queso y tomate. ¿Cuál de las siguientes afirmaciones es falsa?: A) Sería aconsejable realizar serología HIV. B) El pronóstico vital del enfermo es malo. C) No necesita tratamiento por ser con frecuencia una alteración transitoria. D) Con bastante probabilidad se deberá a CMV. E) La afectación suele ser bilateral. 58. Paciente de 70 años que presenta metamorfosias, micropsia y disminución de la agudeza visual de varias semanas de evolución, el diagnóstico más probable es: A) Catarata nuclear. B) Catarata subcapsular posterior. C) Degeneración macular senil. D) Pars planitis. E) Hemorragia vítrea. 59. Ante un niño de 5 años con endotropía que ha seguido tratamiento con corrección óptica y colusiones y cuya agudeza visual aún no es normal. La pauta a seguir será: A) Corrección quirúrgica de la endotropía. B) Continuar las oclusiones sobre ojo con mejor visión. C) Continuar las oclusiones sobre ojo con peor visión. D) No ocluir más y pasar a otro tratamiento. E) Observación. 60. Varón de 70 años que presenta pérdida visual progresiva unilateral, que precisa cambios sucesivos de corrección óptica miópica. La causa más probable es: A) Degeneración macular senil. B) Glaucoma crónico simple. C) Vitritis senil. D) Coriorretinosis senil. E) Catarata nuclear. 61. Varón de 51 años que sufre súbita pérdida visual total e indolora en ojo derecho, apreciándose en el F.O. una retina de color blanquecino con mácula rojo-cereza. El diagnóstico más probable es: A) Desprendimiento de retina con afectación macular. B) Enfermedad de Tay-Sacks. C) Enfermedad de Newman-Pick. D) Obstrucción de arteria central de la retina. E) Obstrucción de vena central de la retina. 62. Varón de 10 años de edad que presenta estrabismo, disminución de agudeza visual y leucocoria en ojo derecho, sin malformaciones oculares. ¿Qué patología debemos descartar como más probable?: A) Retinoblastoma. B) Catarata congénita. C) Vítreo primario hiperplásico persistente. D) Enfermedad de Coats. E) Fibroplasia retrolental.
  • 20. [19] Mg. Dany Colca Lic. Carolina Ayala RESIDENTADO DE ENFERMERÍA. Compendio Banco de Preguntas. 63. Paciente de 65 años con Diabetes mellitus tipo II de 10 años de evolución con mal control metabólico presenta disminución de visión brusca unilateral. La causa más probable será: A) Isquemia macular. B) Desprendimiento de retina traccional. C) Hemorragia vítrea. D) Edema macular. E) Obstrucción de arteria central de la retina. 64. Varón de 35 años sano de carácter nervioso comienza con metamorfopsias y escotoma central unilateral, disminución de visión moderada y patrón angiográfico en chimenea. El diagnóstico más probable será: A) Distrofia viteliforme de Best. B) Retinosis pigmentaria. C) Degeneración macular ligada a la edad. D) Enfermedad de Coats. E) Coriorretinopatía central serosa. 65. Una mujer de 35 años, no fumadora, atleta de fondo y sin antecedentes personales de interés refiere en los últimos 3 meses una menor respuesta al ejercicio habitual, con disnea de moderados esfuerzos. Tos no productiva pero niega fiebre. En una ocasión reciente presentó la emisión de varios esputos hemoptoicos, que no se han vuelto a repetir. La Rx de tórax muestra un patrón reticular fino difuso bilateral y un mínimo derrame pleural derecho. En las pruebas funcionales llama la atención el incremento de los volúmenes pulmonares. De los siguientes, ¿cuál le parece el diagnóstico más probable?: A) Lupus eritematoso sistémico. B) Tuberculosis. C) Sarcoidosis. D) Linfangioleiomatosis. E) Enfermedad de Hamman-Rich. 66. Varón de 50 años, no fumador, que refiere disnea de moderados esfuerzos de unos 4 meses de evolución con tos no productiva. Ha recibido tratamiento con diuréticos de asa (furosemida) en el último mes tras realizarse una Rx de tórax (que no aporta). Acude a urgencias por incremento de la disnea y expectoración de esputos claros. Refiere febrícula de predominio vespertino y pérdida de unos 6 Kg de peso. Presenta hipoxemia con hipocapnia (insuficiencia respiratoria parcial) y la Rx de tórax muestra densidades difusas bilaterales, confluentes, mal definidas de predominio parahiliar con un índice cardiotorácico en el límite de la normalidad. FVC: 65%, FEV1: 70%, FEV1/FVC: 75%. DLCO: 60%. A) Insuficiencia cardiaca. Edema agudo de pulmón. B) Enfermedad de Hamman-Rich. C) Hemorragia pulmonar. D) Neumonía por CMV. E) Carcinoma bronquioalveolar. 67. Varón de 64 años, veterinario de profesión, bebedor esporádico que presenta fiebre, cefalea y artromialgias desde hace 7 días. Durante las últimas 48 horas refiere tos productiva, dolor pleurítico en el costado derecho y disnea progresiva. Ha sido tratado con eritromicina, pese a lo que se encuentra febril y desorientado. Se evidencian estertores crepitantes en la base derecha y una hepatomegalia a 4 cm del reborde costal. En las pruebas complementarias efectuadas, destaca Hb de 14.8 mg/dL, leucocitosis (17.000/μL) con desviación izquierda, GOT (AST): 106, GPT (ALT): 82. En la Rx de tórax se aprecia un infiltrado intersticial en lóbulo inferior derecho. Su diagnóstico de presunción debe ser: A) Neumonía neumocócica. B) Neumonitis por hipersensibilidad. C) Fiebre Q. D) Legionella. E) Tuberculosis. 68. Varón de 37 años que viene presentando durante los tres últimos meses astenia, esputos hemoptoicos y disnea progresiva hasta hacerse de mínimos esfuerzos, con intolerancia al ejercicio. Salvo una TA de 90/60, los datos exploratorios son anodinos. Sin embargo, los estudios complementarios nos sorprenden: pH: 7.37, PaO2: 62, PaCO2: 37, HCO3: 27. Hb: 9.2 y creatinina de 2.3. La Rx de tórax muestra infiltrados difusos parahiliares bilaterales. Ante los hallazgos reseñados, se añade la petición de un sedimento de orina, que muestra microhematuria y proteinuria. De las siguientes, ¿qué prueba diagnóstica le parece prioritaria en la evaluación del enfermo?: A) Test de difusión (DLCO). B) Examen citológico (esputo o lavado broncoalveolar). C) Anticuerpos anti membrana basal glomerular, c- ANCA. D) Biopsia pulmonar. E) Biopsia renal. 69. En el caso anterior, la capacidad de difusión del CO está aumentada y el título de anticuerpos antimembrana basal glomerular es de 1:128. ¿Cuál le parece el diagnóstico más probable?: A) Granulomatosis de Wegener. B) Síndrome de Goodpasture. C) Tuberculosis. D) Tromboembolismopulmonar. E) Granulomatosis de Churg-Strauss. 70. Mujer de 78 años que en el curso de un postoperatorio por fractura de cadera comienza con un cuadro brusco de disnea y febrícula. Exploración: taquipnea a 30 r.p.m., taquicardia a 130 l.p.m., refuerzo del segundo tono, abolición del murmullo vesicular en base de pulmón derecho y extremidades sin edemas, no dolorosas, sin signos flogóticos. Complementarios: GAB: pH: 7.52, PaO2: 56, PaCO2: 30, HCO3: 25. 13.000 leucocitos con desviación izquierda. Rx de tórax: pinzamiento del seno costodiafragmático derecho. ECG: Taquicardia sinusal con bloqueo
  • 21. [20] Mg. Dany Colca Lic. Carolina Ayala RESIDENTADO DE ENFERMERÍA. Compendio Banco de Preguntas. incompleto de rama derecha. Señale la actitud más adecuada: A) Diuréticos. B) Toracocentesis. C) Antibióticos. D) Corticoides. E) Heparina. 71. Mujer de 37 años, fumadora, que consulta por presentar durante el último mes fiebre, malestar general, artralgias y una erupción cutánea dolorosa en ambas piernas. Complementarios. Rx de tórax: adenopatías hiliares bilaterales, sin afectación del parénquima pulmonar. Mantoux negativo. Se realizó una FBC con lavado broncoalveolar (LBA). Líquido del LBA: 22% de linfocitos con cociente CD4/CD8 de 5.2. El diagnóstico más probable es: A) Tuberculosis. B) Sarcoidosis. C) Linfoma. D) Cáncer de pulmón . E) Asbestosis. 72. Varón de 47 años que acude a urgencias por fiebre, tos, artralgias y rinorrea purulenta con ulceraciones de la mucosa nasal de dos semanas de evolución. Inició tratamiento antibiótico 7 días antes, al ser diagnosticado por su médico de cabecera de sinisitis (opacificación de ambos senos maxilares), sin obtener una mejoría clínica. La Rx de tórax presenta múltiples nódulos pulmonares bilaterales, algunos de ellos cavitados. En los análisis efectuados destaca un sedimento de orina con 8 hematíes por campo con algún cilindro eritrocitario. La biopsia de la mucosa nasal mostró inflamación granulomatosa con necrosis. El diagnóstico más probable es: A) Granulomatosis de Wegener. B) Granulomatosis de Churg-Strauss. C) Cáncer de cavum con metástasis pulmonares. D) Granulomatosis linfomatoide. E) Tuberculosis. 73. Un grave problema de las unidades de cuidados intensivos son las infecciones (neumonías y sepsis) por gérmenes gramnegativos multirresistentes (pseudomonas, serratias, citrobacter, morganella, acinetobacter, etc.). Ya se han identificado previamente cepas multirresistentes en nuestra UCI. A falta de un antibiograma, ¿cuál sería el tratamiento empírico de elección? A) Ceftazidima, amicamicina y vancomicina. B) Ceftriaxona y tobramicina. C) Imipenem o ciprofloxacino. D) Imipenem y amikamicina. E) Esperar hasta los resultados del antibiograma. 74. Un varón de 30 años, fumador de 20 cigarrillos/día desde los 20 a los 25 años y ex fumador desde entonces, presenta, en un reconocimiento laboral, un nódulo pulmonar solitario (NPS) de unos 2 cm. de diametro en la periferia del LSD. La Rx de tórax muestra un mediastino normal y no permite identificar calcificaciones en el NPS. El paciente se encuentra asintomático y niega la posibilidad de recuperar radiografías anteriores antes de 6 meses (por cambio de domicilio). ¿Qué actitud mantendría ante este enfermo? A) Informar de la baja probabilidad de malignidad y Rx de tórax en 3 meses. B) Fibrobroncoscopia. C) Realización de una TAC torácica. D) Realización preferente de una PAAF con control de TAC. E) Insistir en la recuperación de las Rx previas y nueva cita en la consulta entonces (6 meses). 75. En el caso anterior, la TAC no muestra nuevos datos (confirma la ausencia de calcificaciones, no adenopatías ni afectación mediastínica y no existe afectación pleural). La familia del enfermo ha localizado las Rx de tórax previas (14 meses antes) en la que se identifica el mismo NPS con un diámetro de 1,4 cm. ¿Cuál sería la actitud más adecuada? A) Actitud expectante y repetir pruebas de imagen en 2 meses. B) Fibrobroncoscopia con citología en las muestras obtenidas. C) Fibrobroncoscopia y biopsia transbronquial (BTB). D) PAAF con control de TAC. E) Toracotomía. 76. En caso de que la PAAF obtenga material suficiente y el resultado sea de malignidad, ¿qué tipo histológico le parece el más probable? A) Ca. epidermoide. B) Adenocarcinoma. C) Ca células pequeñas (CCP). D) Ca. bronquioalveolar. E) Carcinoide. 77. Paciente de 36 años con amenorrea de 10 semanas. Tiene un antecedente de infertilidad por factor tubárico. Refiere episodios de dolor cólico hipogástrico desde hace aproximadamente un mes. El test de embarazo en orina es positivo. La prueba diagnóstica que solicita a continuación es: A) Amniocentesis precoz. B) Ecografía. C) Laparoscopia. D) Triple screening. E) Biopsia de vellosidades coriónicas.
  • 22. [21] Mg. Dany Colca Lic. Carolina Ayala RESIDENTADO DE ENFERMERÍA. Compendio Banco de Preguntas. 78. Gestante de 16 semanas sin antecedentes de interés que presenta los siguientes resultados en la analítica de triple screening: alfafetoproteína 0,3 MM (disminuido), betahcg 1,7 MM (normal), riesgo estimado de T21 1/43. A continuación se le realiza: A) Ecografía. B) Amniocentesis. C) Biopsia de vellosidades coriónicas. D) Funiculocentesis. E) Fetoscopia. 79. Gestante de 26 semanas que consulta por fiebre de 39° C y dolor lumbar unilateral. La analítica de sangre presenta 16.000 leucocitos y desviación izquierda. El tratamiento indicado es: A) Abundante ingesta de líquidos. B) Analgésicos orales y abundante ingesta de líquidos. C) Analgésicos endovenosos y forzar diuresis. D) Antibióticos orales y reposo domiciliario. E) Antibióticos endovenosos intrahospitalarios. 80. Gestante de 30 semanas con aumento excesivo de peso (ganancia de 18 kg. hasta la actualidad) a la que se realiza un test de O’Sullivan que resulta patológico. La actitud médica ha de ser a continuación: A) Vigilancia fetal estricta, con registros semanales de la frecuencia cardíaca fetal. B) Controles de glucemia capilar (BMtest) en desayuno, comida y cena. C) Ecografías seriadas para diagnosticar a la mayor brevedad posible un hidramnios o macrosomía fetal. D) Confirmar el diagnóstico mediante una prueba de tolerancia oral a la glucosa. E) Tratar con insulina rápida según los resultados del test de O’Sullivan. 81. Gestante de 9 semanas que consulta por metrorragia menor que una regla y dolor abdominal. Todavía no ha acudido a ninguna visita de control por su tocólogo. En la exploración se obseva un útero de aproximadamente 8 semanas de gestación, abdomen blando y depresible y cérvix permeable a un dedo. Poco después la paciente empieza a sangrar abundantemente, mucho más que una regla. El tratamiento indicado es: A) Ingreso y observación. B) Laparoscopia. C) Legrado. D) Laparotomía. E) Venoclisis de oxitocina. 82. Primigesta de 27 años, sin antecedentes médicos de interés, que consulta por metrorragia insidiosa y recurrente en semana 36 de embarazo. No presenta dolor abdominal, el útero está relajado y el latido fetal se escucha vigoroso. El diagnóstico más probable será: A) Vasa praevia. B) Desprendimiento prematuro de placenta. C) Expulsión del tapón mucoso. D) Placenta previa. E) Pérdida de líquido amniótico hemático. 83. Tercigesta isoinmunizada que presenta test de Coombs indirecto de 1/10 en semana 30 de embarazo. Se realiza un amniocentesis en que se determina la madurez fetal (se confirma la presencia de fosfatidil glicerol en líquido amniótico) y el nomograma de Liley, que se encuentra en la zona II. La actitud indicada es: A) Nueva amniocentesis en una semana. B) Seguimiento ecográfico con la paciente hospitalizada. C) Administración de corticoides para inducir la madurez fetal. D) Extracción fetal. E) Administración endovenosa materna de IgG contra Ac anti D. 84. Gestante de 33 semanas que consulta por dinámica uterina, disminución de movimientos fetales y febrícula. Al ingreso presenta una analítica de sangre con leucocitosis moderada y el resto de los parámetros normales. En la ecografìa se observa un oligoamnios y un perfil biofísico fetal de 9. El tratamiento consiste en: A) Antibióticos endovenosos. B) Antibióticos intracavitarios. C) Inducción del parto. D) Cultivo del líquido amniótico y tratamiento según antibiograma. E) Administración de corticoides para favorecer la madurez fetal e inducción del parto en semana 36. 85. Gestante de embarazo gemelar que presenta dinámica espontánea en semana 36. Ambos gemelos se encuentran en presentación cefálica. El parto del primero se produce sin complicaciones, pero el segundo se encuentra en posición occipito iliaca derecha transversa al cabo de 25 minutos desde el nacimiento del primero, sin progresar desde un III plano de Hodge. Para finalizar el parto está indicada la realización de: A) Vacuum. B) Fórceps. C) Cesarea. D) Maniobra de Kristeller. E) Cualquiera de las anteriores según el estado fetal. 86. Primigesta de 36 años que presenta un aumento de la tensión diastólica de 30 mmHg respecto a tomas iniciales, y albuminuria con edemas generalizados. Actualmente se encuentra en la 34 semana de embarazo. Súbitamente inicia un cuadro de cefalea y transtornos visuales. Vd. le inicia tratamiento médico con: A) Nifedipina. B) Dihidralacina. C) Alfametildopa. D) Sulfato de magnesio. E) Labetalol.
  • 23. [22] Mg. Dany Colca Lic. Carolina Ayala RESIDENTADO DE ENFERMERÍA. Compendio Banco de Preguntas. 87. En la paciente del caso anterior el estudio fetal descubre un feto afecto de crecimiento intrauterino retardado, en el que la relación entre el área cefálica y el área abdominal es mayor de uno. El feto se encuentra en situación cefálica. La conducta obstétrica adecuada es: A) Expectante, con parto vaginal como via de elección. B) Controles de bienestar fetal (perfil biofísico) semanales hasta la semana 40. C) Valoración de la madurez fetal e inducción del parto, si el feto es maduro. D) Cesárea inmediata. E) Evaluación de la funcionalidad placentaria mediante ecografía Doppler. 88. Puérpera que acaba de parir mediante parto eutócico un varón de 3,450 kg. a los 30 minutos no ha alumbrado, por lo que se realiza una maniobra de Credé para extraer la placenta. Transcurridos unos minutos presenta un cuadro de disnea y hemorragia profusa. El útero está bien contraído, pero el sangrado no cesa. El diagnóstico más probable es: A) Ruptura uterina. B) Desgarro de cérvix. C) Hipotonía uterina. D) Retención de restos placentarios. E) Coagulación intravascular diseminada. 89. Un paciente de 26 años de edad, adicto a drogas por vía parenteral, consulta por malestar general, fiebre de 39°con tiritona, escalofríos y dolor y tumefacción en rodilla derecha. En la exploración llama la atención un soplo cardíco panfocal que previamente no estaba en la historia del enfermo y artritis de rodilla derecha. No se pudo realizar artrocentesis diagnóstica. La cobertura empírica antibiótica más segura sería: A) Vancomicina. B) Vancomicina y Gentamicina. C) Ciprofloxacina. D) Eritromicina y cefuloxima. E) Ceftriaxona. 90. Un joven de 18 años sin antecedentes de interés, consulta por tos, fiebre y otalgia. En la Rx de tórax presenta infiltrado intersticial derecho. La exploración ORL demuestra miringitis ampollosa. Iniciaríamos tratamiento con: A) Ceftriaxona. B) Vancomicina. C) Norfloxacina. D) Isoniazida, rifampicina y pirazinamida. E) Eritromicina. 91. Un paciente al que le detecta una infección urinaria por Pseudomona inicia tratamiento con ceftacidima. Tras 2 días de tratamiento el paciente comienza a encontrarse peor, la fiebre aumenta hasta los 39° y desarrolla hipotensión. En el hemograma destaca leucopenia e importante trombopenia que previamente no estaban. Bioquímicamente presenta hiperglucemia de 198 g./dl. Probablementenos encontramos ante: A) Efecto secundario de la ceftacidima. B) Evolución natural del proceso. C) Situación de shock séptico. D) Probable asociación de una hemopatía. E) Cetoacidosis diabética. 92. Un varón de 27 años consulta por presenta en la región balanoprepucial una úlcera de 1 cm. de tamaño de borde indurado, no doloroso y que secreta serosidad. Presenta adicionalmente adenopatías inguinales bilaterales y fiebre. Estaríamos obligados a solicitarle en el estudio: A) Aglutinaciones a salmonella. B) Aglutinaciones a brucella. C) Serología de VIH. D) Serología luética. E) Serología de micoplasma. 93. Una paciente de 19 años ha sido diagnosticada de mononucleosis infecciosa, confirmada mediante Paul- Bunnel. Ha sido tratada con paracetamol a dosis de 2,5 g./día. A los cinco días del diagnóstico comienza de forma brusca con un cuadro de abdomen agudo y shock. El proceso más probable es: A) Rotura espontánea del bazo. B) Hepatitis por virus de Epstein-Barr. C) Complicación infecciosa intraabdominal. D) Complicación del tratamiento. E) Otra cualquier causa de abdomen agudo: p. ej. apendicitis aguda. 94. Un lactante de 7 meses de edad, con Tetralogía de Fallot intervenida, desarrolla un cuadro de dificultad respiratoria, sibilancias y tos. No se termometra fiebre. Además de las medidas de soporte, el tratamiento de elección sería: A) Reintervención quirúrgica de su cardiopatía. B) Esteroides a dosis plenas. C) Ribavirina en aerosol. D) Eritromicina intravenosa. E) Ceftriaxona intravenosa. 95. Una paciente de 49 años consulta en un Servicio de Urgencias por dolor, enrojecimiento y tumefacción de la parte distal de su miembro inferior derecho. Por sospecha de trombosis venosa profunda se le realiza flebografía que resulta ser negativa. La actitud terapéutica a seguir es: A) Tratamiento antiinflamatorio. B) Heparina de bajo peso molecular. C) Cobertura empírica antibiótica con Oxacilina. D) Reposo del miembro sin más. E) Repetir flebografía pasadas 48 horas.
  • 24. [23] Mg. Dany Colca Lic. Carolina Ayala RESIDENTADO DE ENFERMERÍA. Compendio Banco de Preguntas. 96. Un adicto a drogas por vía parenteral consulta por tumoración fluctuante y con signos inflamatorios en la flexura del codo donde ha realizado inyecciones intravenosas. Se debe realizar: A) Tratamiento quirúrgico de drenaje. B) Cobertura con ceftriaxona. C) Cobertura con vancomicina. D) Cobertura con Oxacilina. E) Medidas antiinflamatorias. 97. Una pareja de turistas que han regresado a Lima, luego de estar en la Selva Central del Perú, comienzan a presentar deposiciones en cantidad abundante de características líquidas, similares al agua, en número de 20-25 al día. No se acompaña de dolor abdominal ni fiebre. El cuadro obliga a una rehidratación y ésta se consigue con dificultad dada la gran pérdida de agua y electrólitos. El cuadro clínico más probable es: A) Gastroenteritis por Salmonella. B) Disentería bacilar. C) Disentería amebiana. D) Cólera. E) Gastroenteritis viral. 98. Un enfermo leucémico, muy inmunodeprimido, desarrolla tras un tratamiento antibacteriano de amplio espectro, cuadro importante de insuficiencia respiratoria y fiebre. En la Rx. de tórax se aprecia una masa densa, cubierta por un menisco delgado de aire en el interior de una cavidad. El diagnóstico más probable es: A) Tuberculosis. B) Neumonía bacteriana. C) Masa tumoral sobreinfectada. D) Neumonía por cándidas. E) Aspergiloma. 99. Un niño de 3 años desarrolla un cuadro de infección respiratoria de vías altas con fiebre elevada. Posteriormente aparece tos paroxística con gallo inspiratorio. El tratamiento de elección sería: A) Ceftriaxona. B) Cefonicid. C) Cefalotina. D) Eritromicina. E) Amoxicilina-clavulánico. 100. Un paciente de 56 años acude a su médico de cabecera por presentar en el último mes fiebre diaria, con una distribución de dos picos, matutino y vespertino. A la exploración llama la atención una hepatoesplenomegalia muy importante. No se objetivan adenopatías a ningún nivel. El hemograma muestra pancitopenia. La prueba diagnóstica que se debería realizar sería: A) Estudio de médula ósea. B) Ecografía abdominal. C) Marcadores tumorales. D) Biopsia hepática. E) Marcadores de hepatitis.